Anda di halaman 1dari 281

SHMAEL M, KHAN & SONS

I I,::::

a1 Sie,u

Copyright Reserved

CARIBBEAN BOOK DISTRIBUTORS (1996) LTD

No part of this book may be


ALL RIGHTS RESERVED.
reproduced, stored in a retrieval system, or transmitted in any
electronic, mechanical, photocopying,
form or by any means
without the prior permission of the
recording or otherwise
copyright owner.

First Edition 2002

ACKNOWLEDGEMENT
Questions from past examinations are reproduced by permission
of the University of Cambridge Local Examination Syndicate &
SNP Publishing Pte. Ltd.

PUBLISHED BY
Z VS7.2717S (/996) 47
#W oda ad
Saq2t4a

r6r-674-47o
paz; gog-674-0497

7
e

The

BCJJ

103133

PREFACE
Assessment and Revision Course in Chemistry for CXC consists of a
compilation of questions specifically designed for use by students attempting the
CXC 0 Level examination in Chemistry.
The objective of the book is to provide practice in answering questions and to
extend knowledge gained during the Course. The wide range of questions that
span the entire syllabus will help students to build the confidence and ability
necessary to master examination techniques.
Accordingly, the book is set to provide practice in Paper 01
Multiple Choice,
Paper 02 Structured Questions, and Paper 03 Essay Type Questions in
the same format and level as CXC.

Particular attention has been given to Question I of the structured paper in which
actual experiments performed by students are used as examples to cover the
data response questions. Questions on Section C, Paper 03, of the Syllabus are
included so that students have adequate practice.
The student is therefore challenged to attempt 600 Multiple Choice items, 50
Structured Questions and 60 Essay Type Questions. It is envisaged that with
such practice in an array of Questions, success in CXC Chemistry is guaranteed.

cONTENTS
of:
There are ten (10) sets of examinations. Each examination consists

Paper 01
Paper 02
Paper 03

60 Multiple Choice Questions


5 Structured Questions

6 Essay Type Questions

SUGGESTiONS
Suggested Time and use
attempt of 1
The test is intended for use over a ten week period with an
examination (Paper 01, Paper 02 and Paper 03) per week.
ts working at
Teachers can use the book for class revision as well as studen
d to use their
home in preparation for the examination. Teachers are advise
allocation for
individual marking scheme for questions but to remain within the
each paper.

ACKNOWLEDGEMENT
of the University
Questions from past examinations are reproduced by permission
of Cambridge Local Examinations syndicate.

Published by:
e66ea oo Deze6aeow (1996) &d.
Sodzz ad
c
#10 9
Sa4q1c44

7d,: g6g-674-?72o
674o
97
g6g
t
paz: 4

CXC 0 Level Chemistry


SCHEME OF ASSESSMENT
Candidates are required to enter for Papers 01, 02, 03, 04.

Paper

Type of Paper

Duration

Marks

01

Multiple Choice

1 hour 15 minutes

60

02

Theory Structured Questions

1 hour 30 minutes

90

03

Essay Type Questions

1 hour 30 minutes

60

04

S.B.A. Practical Work

Over 2 academic years

80

Paper 01
1 hour 15 minutes 60 marks
Sixty (60) compulsory Multiple Choice items.

Paper 02
1 hour 30 minutes 90 marks
A series of five structured questions with Question 1 (Data response) related to
Practicals completed during the two academic years.

Paper 03
1 hour 30 minutes 60 marks
The paper is divided into Three (3) sections with 2 questions in each section.
Candidates must answer one (1) question from each section.

Paper 04
School Based Assessment (S.B.A.)
over their last two years.

This component is completed by students

CXC 0 level Chemistry


Assessment Practice
Paper 01 Test I

TIME:

1 hour 15 minutes.

INSTRUCTIONS TO CANDIDATES
There are sixty (60) questions in this paper. Answer all questions. For each question,
there are four possible answers A, B, C and D. Choose the one you consider correct and
record your choice in soft pencil on the separate answer sheet.
Read the Instructions on the answer sheet very carefully.

INFORMATION FOR CANDIDATES


Each correct answer will score one mark.
answer.

A mark will not be deducted for a wrong

Any rough working should be done in this booklet.

3 of water.
3 sample of alcohol is mixed with 50cm
A 50cm
3
50 cm
of alcohol

50cm
3
of water
.
3
The volume of the mixed alcohol and water is found to be 97cm
Which of the following is the best explanation?

Some alcohol molecules evaporate.

The alcohol molecules fit into gaps between water molecules.

Water and alcohol react to form a gas which escapes.

Water and alcohol react to produce a salt which then dissolves.

The diagram shows a chromatogram which was prepared using spots of five different
inks.

starting lIne
H

solvent
flow

How many different dyes were used to make the five inks?
A

12

A filter tip of a cigarette acts as both a filter and a condenser.


Which of the following cannot be removed, assuming that the filter tip is 100% efficient?
substance

boiling point[C

carbon monoxide

191

nicotine

247

tar

350 to 400

water

100

A gas X is soluble in water and less dense than air. An impure supply of X contains water
vapour and a water-soluble impurity.

1j
12
LiY

water

--

W18L

drying-

agent

In which order should the pieces of apparatus be joined together to collect a pure, dry
sample of X?
1,32,4
D
1,32,5
C
B
1,23,5
1,2,3,4
A
5

What is the formula of uranium (VI) oxide?


A

3
0
2
U

3
U0

6
U0

Which ion has a correct set of numbers for its structure?


neutrons electrons

ion

protons

Li

3
Al

13

14

10

10

17

20

16

Cr

0
6
U

The diagram shows the arrangement of electrons in a particle.


eectron

_-_%

/
4

7
(\

__nucleus
conLning
\____--
7 protons and
,l

8 neutrons

What is the symbol for the particle?


A

3
N

2
Q

Ne

iA

The diagram shows the structure of silicon carbide.

key

carbon atom

a silicon atom

Which set of properties does silicon carbide have?

physical state

when strongly heated in oxygen

soft solid

burns, giving a solid residue only

soft solid

burns, leaving no solid residue

hard solid

burns, giving a solid residue and a colourless gas

hard solid

burns, giving a solid residue only

A 240g sample of hydrated sodium suiphide contains 162g of water of crystallisation.


What is the correct molecular formula of this compound?
A 2
S.3H
Na
0
B
S.5H
Na
0
2
C
S.7H
Na
0
2
D 2
S.9H
Na
0

10 A compound P is the only substance formed when two volumes of ammonia gas react
with one volume of carbon dioxide (both volumes being measured at r.t.p.).
What is the most likely formula of P?
A

Ii

3
C
2
)
4
(NH
0

4
C
NH
N
2
O
H

(NH
C
)
2
0

00
4
NH
N
2
H

On complete combustion, 4.Og of a hydrocarbon give 11 .Og of carbon dioxide and


9.Og of water.
What is the formula of the hydrocarbon?
A CH
4
B C
4
H
2
C C
6
H
2
D 3
C
8
H

12 Apparatus was set up as shown in the diagram.


What occurred at electrode X?
A

Chloride ions were oxidised.

Chloride ions were reduced.

Nickel ions were oxidised.

Nickel was deposited.


carbon
otectrode

cat
etectrode

concentrated
-

aqueous
nickel(It)
cNoride

13 An electric current was passed through molten lead (II) bromide.


Which particles moved towards the positive electrode and which ones towards the
negative electrode?
towards the positive electrode

towards the negative electrode

bromide i s only

lead ions only

bromide ions and electrons

lead ions only

electrons only

protons only

lead ions only

bromide ions only

14 Which equation represents the chemical change caused


photographic film to sunlight?
A

AgAg+e

Ag+eAg

2AgBr
2
2Ag+Br

AgBrAg+Br

by exposing a

15 Which of the following industrial manufacturing processes does not use a catalyst?
A ethanol from ethene and steam
B

hydrogen from methane and steam

iron from iron (Ill) oxide and carbon

sulphur tnoxide from sulphur dioxide and oxygen

16

In the graph, curve X represents the result of the reaction between 1 Og of granulated zinc
and an excess of acid at 30C.

total
volume ot gas
produced
(measured
at r.t.p.)

Which change could produce curve Y?

17

1 .Og of powdered zinc at 20C

1 .Og of granulated zinc at 20C

0.5g of granulated zinc at 40C

0.5g of granulated zinc at 20C

time

In two separate experiments, a substance was decQmposed and the gas evolved was
collected.
The graph shows the total volume of gas collected against time for each experiment.
experIment 2

volume
of gas
coflected

Which graph shows how the speed of reaction varied with time in each experiment?
B

C
Spo&d
of

18

time

time

time

Which reactions below are oxidation reactions?

Xonly

XandY

C
6

Yonly

YandZ

19

Two solutions were mixed in a beaker and the mass of the beaker and contents was then
recorded at various times. The graph shows the results.

time

What could the two solutions have been?

20

21

aqueous copper (II) sulphate and aqueous ammonia

aqueous sodium carbonate and dilute nitric acid

aqueous sodium hydroxide and aqueous zinc sulphate

dilute hydrochloric acid and aqueous sodium sulphate

Which property does hydrochloric acid have?


A

It liberates ammonia from ammonium salts.

It reacts with any base to give a salt.

It reacts with any metal to give hydrogen.

It turns red litmus paper blue.

The table gives information about the sofubilities of the hydroxides, carbonates, and
sulphates of calcium, sodium and zinc.
hydroxide

carbonate

sulphate

calcium

slightly soluble

insoluble

slightly soluble

sodium

soluble

soluble

soluble

zinc

insoluble

insoluble

soluble

What is the best way of making zinc carbonate?


A

Shake solid zinc hydroxide with aqueous sodium hydroxide and pass in carbon dioxide.

Shake solid zinc sulphate and solid calcium carbonate with water.

Shake solid zinc sulphate .ith water and add aqueous sodium carbonate.

Shake aqueous zinc sulphate with solid calcium hydroxide and pass in carbon dioxide.

22

Aqueous ammonia is added to a solution of a salt. A white precipitate is formed which


dissolves in an excess of aqueous ammonia.
Which metal ion could the salt contain?
A

23

3
A1

2
Ca

2
Cu

2
Zn

sodium

Astatine (At) is in Group VII of the Periodic Table.


Which of the following is a property of astatine?

24

It forms a basic oxide.

It is a good conductor of electricity.

It is displaced by chlorine from aqueous potassium astatide.

It displaces iodine from aqueous potassium iodide.

Which of the following is less dense than water?


A

25

iron

lead

Which of the following is a property of aqueous potassium iodide?


A

26

calcium

It reacts with aqueous lead (II) nitrate to form a white precipitate.


It reacts with aqueous bromine to form iodine.

It does not conduct electricity.

It is decolourised by chlorine.

Element X occurs naturally as the uncombined metal; element Y is obtained by heating its
oxide with carbon; element Z is obtained by electrolysing its molten chloride.
What is the order of reactivity of X, Y and Z?
most reactive

least reactive

BY

27

The diagram shows a ball of steel wool


placed inside the end of a small test
tube. The test-tube is inverted in a
beaker of water, trapping air inside.

steel wool

air

water
level

What was the level of the water after several days?


8

at

bennir
o experiment

25

An element X has the following properties:


it corrodes very slowly in air;

it forms a hydroxide which dissolves both in acids and in alkalis.


What could X be?
A
29

aluminium

calcium

copper

iron

DCI

Deuterium (symbol D) is an isotope of hydrogen.


Which formula is incorrect for a deuterium compound?
A

30

4
CD

OD
5
D
2
C

CaOD

Methods used to stop iron from rusting include


coating with tin;
coating with zinc;
connection to magnesium rods.
Which metal is most often used to protect iron in food containers, in oil pipelines and in
roofing sheets?

31

32

33

food containers

oil pipelines

roofing sheets

magnesium

tin

zinc

tin

zinc

magnesium

tin

magnesium

zinc

zinc

magnesium

tin

Which of the following methods would not produce ammonia?


A

heating concentrated aqueous ammonia

heating ammonium chloride with calcium hydroxide

heating ammonium sulphate with sodium hydroxide

heating ammonium sulphate with dilute hydrochloric acid

Which reaction will not occur using cold, dilute sulphuric acid?
A

formation of copper (II) sulphate from copper (II) oxide

formation of copper (II) sulphate from copper

formation of hydrogen with magnesium metal

formation of carbon dioxide from sodium carbonate

Which polymer contains the element chionne?

nylon

poly (ethene)

P.V.C.

Terylene

______

34

Silicon is in Group IV of the Periodic Table.


Which is a correct formula of a silicon compound?
A

35

Si
C
4
I

4
S10

6
H
2
Si

Graphite conducts electricity, whereas diamond does not


Under suitable conditions, graphite can be converted into diamond.
Complete combustion of equal masses of each produces equal masses of carbon
dioxide as the only product.

C
D

37

4
SiN

Which statement indicates that diamond and graphite are allotropic forms of carbon?
A
Both have giant molecular structures.
B

36

In the polymerisation of ethene to form poly (ethene), there is no change in


A
boiling point.
B

density.

mass.

empirical formula.

The diagram shows four stages in a reaction scheme.


stage A
stage B
stage C
petroeum

stage 0
ethanoic

ethene

Which stage involves an addition reaction?


38

In the alkane series of hydrocarbons CH


, the boiling point (b. p.) of the compound
22
increases as n increases.
Which graph correctly represents this effect?

b.

39

b.p.

b.p.

b.p.

A carbohydrate such as starch can be represented as

-x_-x--x _x_
What is X?
A

carbon

hydrogen

C
10

nitrogen

oxygen

40 Which diagram represents the structure of Terylene?

B
4

41

Which one of the following is the best pair of reagents for preparing copper (II)
sulphate crystals?
A

copper oxide and dilute sulphuric acid

copper metal and dilute sulphuric acid

copper carbonate and aqueous ammonium sulphate

copper (II) nitrate and aqueous sodium sulphate

42 Which of the following is most likely to be the melting point of an ionic solid?
A

12C

78C

35C

534C

11

43

One mole of a compound was dissolved in water to make a 1dm


3 solution. Which one
of the following compounds would produce a pH of 2 when dissolved?
A

potassium hydroxide

sodium chloride

sulphuric acid

calcium hydroxide

44 What are the products of electrolysis of concentrated aqueous sodium chloride using
inert electrodes?

45

cathode (-ye)

anode (+ve)

hydrogen

chlorine

sodium

chlorine

sodium

oxygen

hydrogen

oxygen

Lead iodide can be prepared by the reaction of lead nitrate and potassium iodide
according to the following equation:

Pb(N0
(
2
)
3
aq)

2K1(aq)

2KNO
a
3
q

Pbl
(
2
S)

Which of the following methods can be used to separate the lead iodide from the
mixture?

46

distillation

evaporation

filtration

crystallization

A crystal of magnesium chloride is held together by


A

shared paired of electrons

strong attractive forces between oppositely charged ions

covalent bonds

positive ions in a sea of electrons

12

ity by the flow of electrons?


47 Which one of the following substances conducts electric
B sulphur
molten sodium chloride
A
0 aqueous sodium chloride
graphite
C
ht of the product liberated at the cathode in
48 When a solution of a salt is electrolyzed the weig
a given time depends on:
the nature of the electrodes
(i)
(ii)
(iii)
A

the strength of the current


the charge on the metal ion
i only

i & ii only

ii & iii only

i, ii & iii

?
49 Which liquid is the best conductor of electricity
ethanoic acid
B
ethanol
A
C

dilute sulphuric acid

sugar solution

t is its molecular formula if the RMM


50 A hydrocarbon contains 85.6% carbon by mass. Wha
is 84? [RAM C=12, H1]
A

51

4
H
2
C

8
H
3
C

0
C
1
H
4

1
H
6
C

ised in the Contact Process, how


If four (4) moles of sulphur dioxide were completely oxid
many moles of oxygen would be required
A

0.5

1.0

1.5

electrolysing of acidified water in the


Items 52-54 refer to the following diagram used for the
laboratory.
Water with a small
4
amount dilute H,S0

+ve

-ye

all change occurring in this electrolysis?


52 Which one of the following represents the over
+0
2
2H
0
2
2H
B
2 2H
0
2
H
A
4H+O
2
0
2
2H
0
+2O
4
4H
0
2
2H
C
,

13

53 Which ions are present in the liquid being electrolysed?


A
H S0
2 0H
4
2
B
H SO
4 2- 0H
C
H SO
4 0H
2
D
2 so
H
4 2- 0H
2

54 Why are a few drops of dilute sulphuric acid added at the beginning of the experim
ent?
A
To neutralize the water
B

To prevent corrosion of the electrodes

To provide 0H ions for the production of hydrogen atoms


To improve conductivity of the liquid

55 In the fermentation process, enzymes in yeast convert


A
Glucose into sugars
B
C

Cellulose and starch into methanol


Water and carbon dioxide into glucose

Glucose into ethanol and carbon-dioxide

56 Which of the following refers to the excessive consumption of alcoho


lic drinks?
A
Cirrhosis of the liver
B
Brain cell deterioration
C
Retention of body fluids
0
Impaired circulation
57 The oxidation number of chlorine in 4
KCI0 is:
A
-1
B
-3
C
-7

+7

58 Which of the following oxides dissolves readily in water to form


a strongly alkaline solution?
A
Carbon dioxide
B
Sulphur dioxide
C
Potassium oxide
0
Iron oxide

14

metal?
59 In which of the following reactions is there a decrease in the oxidation state of the
)
9
(
2
2 + H
Zn
2 2H
Zn
A
B

3
Fe

3e

Fe(s)

2
Fe

20H

)
s)
2
Fe(OH
(

2C1+2e

2
Cl

60 Which of the following halogens is a solid at room temperature?


Bromine
D
Iodine
C
Chlorine
B
Fluorine
A

15

CXC 0 level Chemistry


Assessment Practice
Paper 02 Test I
TIME:

1 hour 30 minutes.

INSTRUCTIONS TO CANDIDATES

Read the following directions carefully.

1. You must use this booklet when responding to the questions. For each question,
write your answer in the space provided.
2. All working must be shown.
3. There are five questions in this booklet. Answer all five questions.

IM

1th1A-

You should NOT spend more than 30 minutes on Question 1.


1.

3
3 of an aqueous solution of sodium hydroxide containing 8.05g dm
(a) 20 cm
X
2
3 of an aqueous solution of an acid H
are neutralized exactly b 25.0 am
containing 0.080 mol dm

(i)

3 of aqueous solution?
X are present in 25.0 cm
2
How many moles of the acid H
-

or

(ii)

o w
i tlvv

3 of the
How many moles of sodium hydroxide, NaOH, are present in 20 cm
solution?
,--

0)

(iii) Using your answers from (i) and (ii) deduce

(a) How many moles of sodium hydroxide, NaOH, react with one mole of the
X?
2
acid H

(b) The equation for the reaction.

+--

*)

, calculate its
3
H, contains 10.05 gdm
X
(c) If the aqueous solution of the acid, 2
molecular
mass.
relative

(d) Name a suitable indicator, and state the change that occurs with your choice
of indicator. The indicator was added to the acid.

(iv) Use your resufts from (iiic) to show whether or not the acid, H
X, is sulphunc
2
acid, 4
S0
2
H
.

(v)

Give the formulae of the two calcium salts of the acid H


X using X in the
2
formulae.

(b) The following tests were carried out on substance Y which is composed of two
oxides of the same metal. The observations were recorded in the table. Based
on these observations, deduce what you can about these two oxides and the
identity of any gas or gases evolved and name the metal ion present.
TEST

OBSERVATIONS

Dilute nitric acid was added


to X to separate the two
oxides.
The mixture was
stirred and then filtered.

Dark brown residue on filter


paper. Filtrate colourless.

CONCLUSTIONS

The residue from (i) was Green and yellow gas


placed in a clear test tube formed which
bleaches
and concentrated hydro- litmus. A white discoloured
chloric acid added and then solid on the side of the test
heated gently.
tube.

iiia

To separate portion of the


filtrate from (i) a little sodium
hydroxide was added until in
excess.

White precipitate formed


which was soluble
in
excess.

iiib

To a second portion dilute


hydrochloric and was added.

A white
formed.

precipitate

was

iiic

To a third portion dilute


sulphuric acid was added.

A white
formed.

precipitate

was

1 (c) The following experiments were carried out to find out the conditions which
caused the rusting of iron.

pieces ot
anhydrous
calcium chloride

oiI
cotton wool

...

5 boiled
water
to remove
dissolved air

iron nali

Tube I

Experiment I

water

Tube II

Tube Ill

Experiment 2

Experiment 3

Rusting only took place in Tube Ill.


(i)

Explain why the nail in Tube I did not rust.

(ii)

zrL

Explain why the nail in Tube Il did not rust.

.JJ

(iii) What was the purpose of the layer of oil in Tube II?
.

--

O4
-

--

0
L
1
-I

L.

Lk

(iv) How could the rate of rusting be increased in Tube Ill?

(L

(
S

(v) What could be done to the nail in Tube Ill to slow down the rate of rusting?

SS

55

%JJ

L.

(a)

;.

Describe what is observed in each of the following reactions:


(I) aqueous sodium hydroxide is added to aqueous iron (Ill) sulphate;

(ii) dilute hydrochloric acid is added to solid sodium carbonate,

eL!

aqueous barium chloride is added to dilute sulphuric acid;

(ii)

(iv) aqueous silver nitrate is added to aqueous sodium chloride.


[4]
2

(b) The following equations represent changes that occur during electrolysis. Use only these
equations to answer the questions.
2Cl
Cu
+
2
2e
Cu
2H+2e40H
(i)

C1
+
2
2eCu
2e
2
Cu
2
H
04e2H

2
0

Write one equation that represents a reduction.


Explain your answer

(ii)

Write the equation that shows the process which must occur when an object is
electroplated with copper.

(iii)

Write two equation which show the processes that occur when dilute sulphuric acid
is electrolysed.
1
2

(a)

[4]

Uranium-235 and uranium-238 are two isotopes of uranium The table below shows the
numbers of the particles in one atom of uranium-235.

(i)

particle

number

electron

92

neutron

143

proton

92

Suest a reason why this is known as uranium-233

n:..

____
________
______
____
_______

(ii)

In the table below, give the numbers of particles found in an atom of uranium-238.
number

particle
electron
neutron

_L

proton
(iii)

(b)

Uranium-235 is radioactive. It is used in the manufacture of nuclear weapons. Give


one other use of uranium-235.
[3]

3 portions of an acid
A student added lqm
3 potassium
3 of 0.10 mol/dm
HA to 25cm
hydroxide. After each addition of thern
acid, the student measured the pH of the
mixture, using a pH meter. From his
results, the student drew the graph shown
below.
(a) What is the pH of a neutral solution?

WJIH

86 i.ni
IIIIII;4

pH

1t11

11;Ir

rjrr111r IH

1:1.111

I I 1111111 iiI

H I I

.:

(b)

(I)

(ii)

What volume of acid was


needed to neutralise the
alkali?

j....j

III

I H

I I, III I I

I III 11111
I

HP

IM

Acid HA reacts with potassium hydroxide as shown in the equation below.


KOHHAKA+H
O
2
, of acid HA.
3
Calculate the concentration, in mol/dm

(c)

(a)

Suggest a way, other than using a pH meter, by which the student could follow the
pH changes during this reaction
[6)

Fats contain the ester linkage. The structure of a tat can be represented by the diagram
(a) Fat is one of the three main energy sources present
NaO

tw

(b)

An ester such as ethyl ethanoate reacts with aqueous sodium hydroxide as shown
by the diagram below.
HO

H
+

NaOH

HO
Na

HOCCH
1I4

Draw the structures of two products of the reaction of a fat with aqueous sodium
hydroxide.

(c)

Terylene is a polyester.
Draw the structure of Terylene.

j9
[6]

(b)

In an experiment, copper (II) oxide was reduced by heating it with powdered chromium.
Chromium (Ill) oxide and copper were formed.
The table below gives some information about the substances involved.
substance

action of water

action of hot dilute sulphuric acid

chromium, Cr

insoluble

reacts to give a green solution

chromium (Ill) oxide, Cr


3
0
2

insoluble

reacts to give a green solution

copper, Cu

insoluble

does not react

copper (II) oxide, CuD

insoluble

reacts to give a blue solution

(a)

Construct an equation to show the reduction of copper (II) oxide by chromium.


f

(ii) At the end of the heating experiment, all four substances were present in the reaction
mixture.
Suggest how a pure dry sample of copper can be obtained from this mixture.

(iii)

Name a gas which reduces copper (II) oxide on heating.

(iv)

Similar experiments between other metals and metal oxides were carried out. The
findings from these experiments are shown in the table below.

metal used

metal oxide used

was the metal oxide reduced?

chromium

copper (II) oxide

yes

copper

lead (II) oxide

no

lead

chromium (Ill) oxide

no

sodium

chromium (Ill) oxide

yes

(.1)

Place the metals chromium, copper, lead and sodium in order of decreasing
reactivity.
Most reactive
Least reactive

(ii)

Hence predict what would be observed if a piece of chromium is dipped into


aqueous lead (II) nitrate.
[9]

(a)

The following questions refer to compounds with the structures drawn below.

H--coH
H

1-4

D
H

I-

C CC H
H

HCCH

Using the letters A to F as appropriate, answer the following


(i)

Which compound is an unsaturated hydrocarbon


9

(ii)

Which compounds are members uf the same homologous series


7

L..
.
1
?

(iii) Which compound reacts with sod.m carbonate


9

(iv) Which compounds are isomers ot each other


8

[4]

(b

Complete the following table which refers to the uses of some gaseous elements.
use

element

chlorine
hydrogen
nitrogen
[4]

oxygen

(C) Fluorine can form either covalent or ionic bonds.

(i)

Draw a dot-and-cross diagram to show the bonding in


(i)

sodium fluoride, NaF,

(ii)

.
2
fluorine, F

Your diagrams must show all the electrons.


sodium fluoride

fluorine
(ii)

Explain why sodium fluoride has a higher melting point than fluorine.
.1XL?

ot

srr3

%4

(r

M
0
L

[5]

CXC 0 Level Chemistry


Assessment Practice
Paper 03 Test I

Time:

1 hour 10 minutes.

INSTRUCTIONS TO CANDIDATES

Read the Instructions on the answer sheet very carefully.

Candidates MUST answer THREE questions on this paper. They must choose ONE from
Section 1, ONE from Section 2, and ONE from Section 3.
All working MUST be shown for calculations.

Answer one (1) question from this section.


Section I
I

Hydrogen chloride can be manufactured by reacting hydrogen and chlorine


together. The reaction is exothermic.
HH+CICh H-CI+H-Cl
(a) (i) Calculate the volume of hydrogen chloride that can be manufactured
3 of chlorine (both volumes measured under the same
from 1000 dm
conditions).
(ii) Suggest why hydrogen chloride is stored under high pressure rather
[21
than at atmospheric pressure.
What is the meaning of the term exothermic?
(ii) Explain, in terms of the bonds broken and bonds formed, why the
[5]
reaction between hydrogen and chlorine is exothermic.

(b) (i)

(C)

The reaction between hydrogen and chlorine is an example of a redox


reaction.
Explain what is meant by a redox reaction and name the substance
[3]
oxidised and the substance reduced.

Concentrated sulphuric acid is a dehydrating agent.


elements of water from a substance.

It can remove the

Suggest the names of two gases formed when concentrated


. Give a chemical
4
0
C
2
sulphuric acid is added to ethanedioic acid, H
test for one of these gases.
, turn white when
5H
4
CuSO
O
(ii) Blue copper (II) sulphate crystals, 2
added to concentrated sulphuric acid. Write an equation for this
[4]
reaction.

(a) (i)

H, to give a
C
O
9
H
(b) Concentrated sulphuric acid reacts with butanol, 4
compound X.
Compound X decolourises aqueous bromine.
(i) What is the molecular formula of X?
(ii) Suggest two possible structures for X.

[3]

(c) Concentrated sulphuric acid is manufactured from sulphur.


(i) Give the names of the other raw materials needed.
(ii) Give one large-scale use of sulphuric acid.

[3]

Section II
Answer one (1) question from this section.
3

Silicon carbide, SiC, has a diamond-like structure and a very high melting point. It is
made by heating pure sand, SiC
, with carbon at a very high temperature. Carbon
2
monoxide is the other product.
-

(a) (i) Construct a balanced equation for this reaction.


(ii) Calculate the mass of sand required to produce 300 kg of silicon carbide. [4]
(b) (i) Draw a possible structure for silicon carbide and explain why this compound
has a high melting point.
(ii) Suggest two other physical properties that silicon carbide is likely to have.
[6]
4

The exhaust gases of a motor car contain several pollutants, among them carbon
monoxide and oxides of nitrogen. The flowchart below shows the arrangement of a
two-stage catalytic converter in a motor car engine and exhaust system.
air

petrol

engine

first catalytic
converter

I,
second catalytic
converter

1
exhaust
gases

In the first converter, carbon monoxide reacts with oxygen.


In the second converter, nitrogen oxides decompose into nitrogen and oxygen.
(a) (i) Explain how carbon monoxide is produced in a motor car engine.
(ii) Give the name of the gas that is produced in the first converter.
Write an equation to show its production.
(iii) Why is carbon monoxide a pollutant?
(b) Two oxides of nitrogen are nitrogen dioxide, 2
NO and nitrogen monoxide, NO.
,
(i) Write an equation to show the decomposition of one of these oxides.
(ii) Why are oxides of nitrogen pollutants?
[3]
(C) (i)
What is a catalyst?
(ii) Suggest why the catalysts in the first and second converters are different.

SECTION III
Answer ONE question from this section

5.

6.

[2]
(a) Give a definition of weighting of fibres.
[2]
(b) What benefits does weighting impart on fabrics?
necessary. [6]
(C) Describe how a given fabric is weighted, giving equations where
(d) Other than weighting, describe one process whereby the properties of natural
fibres may be adjusted by chemically modifying groups in the molecules of the
[6]
fibres.
[4]
(e) Why cant the properties of fibres be chemically modified?

7N

Plant Protein

7
N0

Nitrogen

Ammonia

Animal Protein

Humus

ant role in
The above figure represents part of the nitrogen cycle. Bacteria play an import
this cycle.
the name
(a) Name the two bacteria A and B which takes part in the cycle and state
of the process which converts elemental nitrogen into Y.
nitrogen (11) oxide
2
(b) The formation of Y involves a series of reactions N
nitrogen IV oxide nitric acid Y.
(i) Write formulae for nitrogen 11 oxide and nitrogen IV oxide.
2 nitrogen IV oxide.
(ii) Hence write chemical equations to represent N
(c) Calculate the oxidation number of nitrogen in
3
(ii) NH
2
(i) N0
(d) How do modern farming methods upset the natural nitrogen cycle?
and state
(e) Many gardeners follow a simple rotation of crops. Give an example
what are the disadvantages of crop rotation.
Name
(f) The mealy-bug pest can be controlled by the use of the lady bird beetle.
two advantages and one disadvantage for the use of this type of control.
4

CXC 0 Level Chemistry


Assessment Practice
Paper 01 Test 2
TIME:

1 hour 15 minutes.

INSTRUCTIONS TO CANDIDATES
There are sixty (60) questions in this paper. Answer all questions. For each question,
there are four possible answers A, B, C and D. Choose the one you consider correct and
record your choice in soft pencil on the separate answer sheet.
Read the Instructions on the answer sheet very carefully.

INFORMATION FOR CANDIDATES


Each correct answer will score one mark.
answer.

A mark will not be deducted for a wrong

Any rough working should be done in this booklet.

3 of water.
A 50cm
3 sample of alcohol is mixed with 30cm
50 cm
3
of alcohol

50cm
3
of water
.
3
The volume of the mixed alcohol and water is found to be 97cm
Which of the following is the best explanation?
Some alcohol molecules evaporate.

The alcohol molecules fit into gaps between water molecules.

Water and alcohol react to form a gas wnich escapes.

Water and alcohol react to produce a salt which then dissolves.

ne diagram shows a chromatogram which was prepared using spots of five different
inks.
I

I
I

starting line

solvent
flow

How many different dyes were used to make the five inks?

12

A filter tip of a cigarette acts as both a filter and a condenser.


Which of the following cannot be removed, assuming that the filter tip is 100% efficient?
substance

boiling point/C

carbon monoxide

191

nicotine

247

tar

350 to 400

water

100

An isotope of element X has 17 protons and 20 neutrons in its nucleus.


Which symbol is correct for the ion of X?

?X

CuS
.
4
0
5H
,
O changes
The reaction scheme shows how hydrated copper(lI) sulphate, 2
when heated.
250C
90C
p- CuSO
4
.H
4
CuSO
0
2
.5H
4
CuSO
0
2
-

A little water was accidentay spilled into a basin containing hydrated copper (II) sulphate.
0?
2
. 5H
4
What could be done to remove the water, leaving pure, dry CuSO

Heat the basin over a boiling water-bath.

Heat the basin to constant mass.

Heat the basin with a bunsen.

Let the basin stand in direct sunlight.

An element X has proton (atomic) number 3.


Which statement about X is correct?

It forms ions by gaining electrons.

It is a gas at room temperature and pressure.

It is in Group Ill of the Periodic Table.

It reacts with cold water.

In which of the following pairs do the elements form a compound by sharing electrons?.
A

carbon and chlorine

lithium and iodine

neon and oxygen

potassium and bromine

How does a magnesium atom form a bond with an oxygen atom?


A
by giving one pair of electrons to the oxygen atom
B

by sharing one pair of electrons, both electrons provided by the magnesium atom
by sharing two pair of electrons, both pairs provided by the oxygen atom

by sharing two pair of electrons, each atom donating one pair of electrons

What volume of sulphur dioxide (at r.t.p.) is giver off on heating 9.7 g of zinc sulphide,
ZnS?
2ZnS + 302
2ZnO + 2S0
2

3
1.2dm

3
24dm

3
3.6dm
3

3
4.8dm

10

In an electrolysis experiment, the same quantity of electricity deposited 1 6g of copper and


6g of titanium.
What was the charge on the titanium ion?
A

11

1+

2+

3+

4+

A sample of copper contained as impurity a metal below it in the reactivity series. The
diagram shows the apparatus used for refining the sample.
+

impure
copper
anode

pure
copper
cathode

aqueous
copper(H)
sulphate

The loss in mass of the anode (positive electrode) was 50g and the gain in mass of the
cathode (negative electrode) was 45g.
What was the percentage purity of this sample of copper?
A
12

10.0%

11.1%

90.0%

95.0%

Dilute sulphuric acid was eiectrolysed using inert electrodes.


What volumes (at r.t.p.) of hydrogen and oxygen were formed by the passage of one mole
of electrons?
volume of
/dm
2
H
3

volume of
/dm
2
0
3

24

12

12

12

12

24

13

14

15

What are the products when concentrated aqueous lithium chloride is electrolysed?

at the anode
(positive)

at the cathode
(negative)

chlorine

hydrogen

chlorine

lithium

oxygen

hydrogen

oxygen

lithium

Which statement is correct for all exothermic reactions?


I

A catalyst is needed for the reaction to take place.

Light is absorbed during the reaction.

The products of the reaction have less energy than the reactants.

They are reactions which require heat to start.

A chemical reaction between a solid and an acid is too fast for safe use by a class of
students.
Which set of changes is most likely to make the reaction safe for the students to use?

16

pieces ot solid

acid

temperature

larger

less concentrated

lower

larger

more concentrated

higher

smaller

less concentrated

lower

smaller

more concentrated

higher

Curve I was obtained by observing the decomposition of 100cm


3 of 1 mol/dm
3 aqueous
hydrogen peroxide, catalysed by manganese (IV) oxide.
2H
(
0
2
aq)

0(l)
2
2H

(g)
2
0

Which alteration to the conditions would produce curve II?

17

adding some 0.1 mOI/dm3 aqueous hydrogen peroxide

lowering the temperature

using a belier catalyst

using less manganese (IV) oxide

The graph shows how the total volume of hydrogen produced by reaction between
hydrochloric acid and an excess of magnesium varied with time.

total volume
of hydrogen
produced

time

Which statement about section XY of the curve is correct?

18

All the magnesium has reacted.

No more hydrogen is being produced.

The concentration of the acid is decreasing.

The speed of reaction is at a maximum.

Hydrogen peroxide, H
, reacts with an oxide, M
0
2
0.
2
0(s)
2
M

l)
H
(
0
2

2M(s)

0(l)
2
H

(g)
2
0

What is the function of the oxide, M


0, in this reaction?
2

19

a dehydrating agent

a reducing agent

an acid

an oxidising agent

Two bottles in a laboratory are unlabelled. One bottle is known to contain aqueous sodium
iodide and the other aqueous sodium chloride.
Whch :est would identify the scutions?

adlition of aqueous lead il) nitrate

addition of dilute sulphuric acid

addition of blue litmus paper

addition of aqueous sodum hydroxide


6

20

21

22

23

Which hydroxide dissolves in water to form an alkaline solution


9
A

aluminium hydroxide

calcium hydroxide

copper (II) hydroxide

iron (Ill) hydroxide

Which substance is not used to prepare zinc chloride by reaction with hydrochloric acid?
A

zinc carbonate

zinc hydroxide

zinc nitrate

zinc oxide

Which gas changes damp litmus paper from red to blue?


A

ammonia

carbon dioxide

chlorine

hydrogen chloride

The table shows the electron structures of five elements.


element

electron structure

2,4

2, 8, 2

2,8,5

2,8,6

2,8,8,1/

C,3,J

Which two elements form ionic chlorides?


A
24

P and S

Q and T

A and S

R and T

Element M has a high density and a high melting point. It forms two chlorides MCI and
.
2
MCI
What does this suggest about element M?
A
It is a Group If metal.
B
C

It is a Group IV metal.
It is an alkali metal.

It is a transition metal.

25

Astatine, At, is immediatey below iodine in the Periodic Table.


Which of the following statements about astatine is not correct?

26

It forms a hydride which ionises in water.

It forms a molecule At
.
2

It forms a soluble silver salt.

it is a solid at room temperature.

Which of the following oxides is most readily reduced to the metal by heating in a stream
of hydrogen?
A

27

calcium oxide

copper(ll) oxide

sodium oxide

zinc oxide

Which diagram represents the structure of an alloyA

ZGG

28

29

30

Which of the following reactions would you expect to take place?


A
(aq) + Mg(s)
2
Cu
(aq)
2
Mg
+ Cu(s)
B

Fe
(
2
aq)

Cu(s)

2H(aq)

Cu(s)

Mg
(
2
aq)

Zn(s)

Fe(s)

Cu
(
2
aq)

H(g)

Cu
(
2
aq)

Mg(s)

Zn
(
2
aq)

Diammonium hydrogenphosphate, ,
HP0 is a widely used tertiliser.
2
)
4
(NH
Which of the following could be added to this compound to produce a mixture providing
the three principal elements needed for plant growth?
A

ammonium sulphate

lime (calcium hydroxide)

magnesium nitrate

potassium chionde

Which of the following gases is most common in air?


A

argon

carbon dioxide

oxygen
water vapour
8

31

The table gives information about three indicators.


colour change
high pH
low pH

indicator
methyl orange

red

bromothymol blue

yellow

yellow

phenolphthalein

pH at which colour
change takes place

colourless

4.0

blue

6.5

pink

9.0

If equal volumes of these three indicators were mixed, which colour would be observed at
pH 5?
A

blue

green

orange

Inihe manufacture of sulphuric acid, the following reaction occurs:

32

2
2S0

+ 2

3
2S0

What conditions of pressure and temperature are used for this reaction?
pressure/atm

33

34

35

temperaturelC

200

20

200

500

20

500

Which of th? following is not a use of chlorine?


A

disinfecting swimming pool water

making domestic bleaches

making nylon

making pvc

What is the function of silica, Si0


, in the equation shown?
2
CaO + Si0
2
3
CaSiO
A
a basic oxide
B

a reducing agent

an acidic oxide

an oxidising agent

What are the molecular formula and empirical formula of ethanoic acid?
molecular
empirical
formula
formula
A
0
4
H
2
C
0
4
H
2
C
B

0
4
H
2
C

O
2
CH

0
4
H
2
C

0
4
H
2
C

0
6
H
2
C

0
6
H
2
C
9

yellow

36

Which set contains the correct process for converting substance S to the product(s)?

37

38

product (s)

substance S

process
A

addition polymerisation

an ester

Terylene

condensation polymerisation

starch

glucose (a sugar)

hydrolysis

protein

amino acids

reduction

ethanol

ethanoic acid

?
4
Which set contains all the possible combustion products of methane, CH
A

carbon, carbon dioxide, carbon monoxide and water

carbon, carbon monoxide and hydrogen

carbon dioxide, carbon monoxide, hydrogen and water

carbon monoxide and water

The diagram shows the structure of a compound X.

7
H
How can X be classified?
A

as an acid and as an alcohol


as an alkene and as an acid

39

as an alkene and as an alcohol

as an alkane and as an ester

The diagram shows the structure of a polymer.


H

ccCCCC

CL

Cl

Cl

Cl

Which monomer is used to manufacture the polymer?

ci

HCH

CI

CC

CI

H=CH

11

10

Cl

Cl

HCCH

40

0 reacts with sodium


4
H
2
An aqueous solution of a compound of molecular formula C
carbonate, liberating carbon dioxide.
What is the structural formula of the compound?

/H

soH

H\/0\/ H

\/

HCC

SOH
H

11

27

56

cs

Fr

59

La

Ba

1,,

Cs

C..w.r

Ac

Re

99

227

226

87

Ln

139

Yttsfl

Sr

37

733

99

96

Key

Zr

91

72

48,.n

Hf

75

Zvwnr
40

22

Ti

49

bwponnx17be

xato1T9cs1nbo

in4at,v aloer 71785.3

t90-103 Actinoid series

58-71 Lanlhanoid series

97

55

31

Rb

59

Rd&,,

79

Sc

Ca

Pan

Sk

45

20

72

Mg

Na

39

24

33

II

40

ii

ii

90

18

73

41

Ta

787

Ncon

Nb

93

Mo

98

Tc

Nd

744

9wa.r

93

93

U....4n

226

745d5n

75

Re

786

IflMfum
43

25

U9

Mn

56

Ru

107

Pm

0,n.n,

Os

II)

Np
93

7*8t

91

P75

76

44

Pw.,.sn

28

fron

Fe

58

--

Sm

790

.o.r

Ir

92

.un

Rh

703

94

Pts.,n

Pu

Snnn,
82

77

45

21

Cob.4

Co

56

759

Am

(s.s.,

Eu

152

Psr

Pt

59

kn..,,

83

78

44

109

Pd
PMbr

28

NI

Ni

56

Gd

757

044

Au

797

S*

Ag

708

Cosn.

98

Cats.,

Cm

Gr
64

79

47

29

84

Cu

59

49
207

Bk

1..

Th

756

Ms..y

Hg

97

96bs.

85

Cd

172

Z.n

Ca*vsir

30

59

Zn

II

49

31

73

74

Dy

762

Ths.,

TI

204

war

In

115

Gn

Ga

70

Cf

119

Es

s.6s...

Ho

1*

L.

Pb

201

Is.

Sn

(ss.l,.r
99

87

82

50

32

G.nn.n

Ge

73

21

Si

27

Al

C..SIons.,,,

59

72

II

iii

DDa,.s.,
66

The volume of one mole of any gas is 24 dm


3 at room temperature and pressure (r.t.p.)

97

Pa
Ps.,n

Th

fln

Yws.,n

232

Cw,s,n

741

Pr

1,.n

784

740

49

74

43

M&VtftJrn

24

CNnn

Cr

52

Ce

T,86,n

23

57

Group

DATA SHEET
The Periodic Table of the Elements

Fm

E9.n

Er

787

I.n.4w

Bi

200

AflaW

Sb

122

AC

103

Fw,.,

96

93

51

33

As

75

Md

t7.O4n

Tm

796

Po

I.s.nn

To

121

S.n

Ms.bssr
90)

96

94

52

34

Se

79

S9h.

79

Phi

32

31

76

vi

P
15

74

Yb

773

A.8,.

At

1.

727

Br

59

Cl

355

No
P86Hss.
101

70

Lwa

Lu

773

Rn

8ns

Xe

731

K99c.

Kr

94

A,n

j.r.ssr
703

77

98

54

39

18

Ar

40

7W
70

F4.o...

20

lIe

Na

Vlblsr

96

53

35

17

II

\ii

41

Which of the following can be represented by the ionic equation below in which X is the
symbol for a metallic element?
X(s)

42

45

iron

dilute nitric acid

sodium water

copper

steam

Which one of the following is extracted by the reduction of its oxide?


sodium

calcium

iron

potassium

Which of the following is most likely to form ions with charges of +2 and +3?

A
44

iron

2H(aq)

dilute hydrochloric acid

A
43

copper

aluminum

iron

calcium

Amino acids are converted to proteins by:


A

addition polymerization

condensation polymerization

deamination

hydrolysis

Which of the following properties would cause an element to be classified as a metal?


A

It can act as an oxidising agent

It is a gas at room temperature

It is usually brittle

It forms a basic oxide

Items 4648 refer to the following observations:


A

brown precipitate insoluble in excess

white precipitate soluble in excess

white precipitate insoluble in excess

yellow precipitate insoluble in excess

46

Aqueous sodium hydroxide is added to a solution containing zinc ions until no further
change takes place.

47

Aqueous potassium iodide is added to a solution containing lead ions.

48

Aqueous sodium hydroxide is added to a solution containing calcium ions.

13

49

50

The calcium salt responsible for temporary hardness of water is:


A

the hydrogen carbonate

the sulphate

the chloride

the carbonate

Which of the following properties is NOT typical of an acid?


A

sour taste

turns blue litmus to red

liberates carbon dioxide from carbonates

neutralizes bases with the evolution of hydrogen

Items 51 53 refer to the following:


A

Isomerism

Polymerization C

Hydrogenation

Esterification

Each option may be used once, more than once or not at all.
51

The following structures illustrate this property:


H H HH
II
II
H-C-C=C-C-H
H

H
HH
I
II
CtCCC-H
Ill
HHH
H

52

Starches are products of this type of process.

53

This type of reaction can occur across the double bond of compounds.

54

Dehydration of propanol will result in the formation of:


A

55

propane

propene

ethane

ethene

For two substances to be separated by paper chromatography, it is necessary that:

They are both liquids

They have different densities

They are soluble in the same solvent

They have different colours

14

56

Barium is in the same group of the Periodic Table as calcium. Which of the foflowing
statement is likely to be true of barium? it is..
A

easily reduced

high in the activity series

a non metal

normally found uncombined in the earth

Items 57

59 refer to the following data:

From the following list, select one method by which each of the following may be
separated from the mixtures. Each method may be used once, more than once, or not
at all:
A

filtration

fractional distillation

chromatography

sublimation

57

Amino acids from a mixture obtained by the hydrolysis of proteins.

58

Oxygen from liquid air.

59

Ammonium chloride from a mixture of ammonium chloride and sodium chloride.

60

Which of the following solutions will turn green when either sulphur dioxide or hydrogen
sulphide is bubbled through?
A

Potassium chloride

Potassium manganate (VII)

Potassium dichromate

Potassium nitrate

15

CXC 0 Level Chemistry


Assessment Practice
Paper 02 est 2
TIME:

1 hour 30 minutes.

INSTRUCTIONS TO CANDIDATES

Read the following directions carefully.

1. You must use this booklet when responding to the questions. For each question, write
your answer in the space provided.
2. All working must be shown.
3. There are five questions in this booklet. Answer all five questions.

You should NOT spend more than 30 minutes on Question 1.


1. (a) The following experiment was carried out to determine the end point of a
neutralization reaction by measuring temperature changes. The student was
provided with two solutions:
X is a 2.0 mol/dm
3 sodium hydroxide solution.
Y is a solution of hydrochloric acid of unknown concentration.
25.0cm of sodium hydroxide was put into a plastic cup and the temperature
3
noted. 5cm
3 of hydrochloric acid was added and the new temperature noted.
This was repeated until a total of 40cm
3 had been added. The results are
shown in the following table.
Total volume of acid added!cm
3
0
5
10
15
20
25
30
35
40

Temperature of mixturelC
33.5
35.0
36.5
38.0
39.0
38.0
36.5
35.0

(i)

Write an equation for the reaction between sodium hydroxide and hydro
chloric acid.
44O

(ii)

-C

L)

Plot a graph of temperature versus volume of acid added.

(iii) Extend your graph so that it crosses the temperature axis. What is the
temperature of the sodium hydroxide solution at the beginning of the
experiment?
5

(iv) Use the graph in (ii) above to determine the volume of acid needed to
neutralize 25cm
3 of sodium hydroxide used.

-w

C,

3,,

V
C

..

.L.44

-I ,

-:

ft -f-i.

.-4 .4

4..

4-

41..

4 .4...

-+

.4---

.4.

.4.

.4

...

.4.

4.
4

4.

-.- 1

fi

-a--

.4

4..

..

..E
.
4

I-

q_

...-

4...

L 4

4
t

.4

._.._

4.

--

.4.

..

..

....,...... .,..

.5

i-!-.w

.4

4-

.4

4-

4-

. 3..

4...L

4-H

,
.

4..

.4.

-.f ..4

4--

.4

...3

4r-

.
.

4.-

4-,.

.,.

.4.

:-

..-q.

4.-,..

-.-.-.

.4-

,....,

,..,. ...

.
..,..,.,...

.
----i-

.---_

.3

4.

..

.,.,

..4..4.4.

.4---

..

I
-4,
.L..
.......,

..t
,

. ..

..,....,,.

-4-h-

.4.

-..

.....L

..

..-.

-i

4q.4_4..._..4.4.444

4.

--

j Jt

.4.

._.,

3..
4._.4_.

_.

--

._

.4

-.

.
.

44.

4.

.14.1

..3

.-

.4.

.-:

- .

.4

4..

.-----

4....

4......,

:,..

4...

.4

4.

-.

4-

3.4...

4-1

.4.

j..

4.

..

4..

..,

,.

L.

.,

.4-3

.,

.4..L

1 -
...

.4...

-:

..r..

- 444-.4

4_4_44-

-i

r-

L-q-

.., ..

-,

.
4...L .4

..4..4

. .3

.4

ZE2E
4--

- 4-.4444
-.-

--.

..

.,-..,..

4.4.
. . . .

-.-...

--;

-1-i+4

..,

. . .-...,
.
..

3.

...

.4. .4.
3... , .3.4

......

.4.4....
.

-4--f

_4+

.-.4-..

.....,

4.

-.4

4.4.

...

1
Z.-

...

...,.._.1..

;
LZZ

.3.

.,
.4.4.
3..

:-E

---

.4

4
r-

....j

:-

r4

,-.

...

4
---.I
.
3--.-.444j---4-.I
-3--

C
b

.j

1
i
iEEEEEEE

r-

..

-.4,

...!

Te

(v) Calculate the number of moles of acid in the volume of acid obtained in (iii)
above.

moles of acid in

cm
Y
3
.

(vi) From your answer in (v) above, calculate the concentration in mol/dm
3 of
hydrochloric acid in solution Y.

The following tests were carried out on a sample salt N. Complete the following table.
TEST

OBSERVATIONS

A small amount of N was heated in a


dry test tube.

Solid turned black, gas


given off

ii

Gas was tested with moist red and blue


litmus.

Blue litmus turned pink

iii

Gas collected and bubbled through lime


water,

Lime water
cloudy

iv

Diluted sulphuric acid was added to the


black residue from (i) and gently heated
and following reagents added.

Sodium hydroxide solution was added


and heated gently.

Blue precipitate formed

vi

Acqueous ammonia added.

Deep blue
formed

CONCLUSIONS

turned

complex

r L4
k

1,

2,

4,

5,

6,

9,

20,

80.

2(a) Choose one of the numbers above to complete each of the sentences (a) to (g).
Each number may be used once, more than once, or not at all.
(a) The percentage by volume of oxygen in the atmosphere is
(b) The total number of atoms in one molecule of methanol is
(C)

The number of elements in the group known as the halogen is

(d) The pH of aqueous ammonia is


(e) The proton (atomic) number of calcium is
(f)

The number of carbon atoms in one molecule of ethane is

(g) The percentage by mass of carbon in ethane is

[7]

2(b) Olive oil contains oleic acid which is unsaturated.


(a) (i)

By naming the reagent and giving the observation, describe a simple test to
confirm that olive oil contains an acid.

Reagent:
Observation:

(ii)

Explain what is meant by unsaturated.

(iii) By naming the reagent and giving the observation describe a simple test tc
confirm that oleic acid is unsaturated.

Reagent:
Observation:
[5]
.
31
H
H
2
17 CO
(b) The molecular formula of oleic acid is C
How many double bonds between carbon atoms are present in one molecule of
oleic acid?
Explain your answer.

Number.
Explanation.
[2]

(a)

Potassium nitrate is a salt which can be prepared by reacting an acid with an alkali,
using the titration method.
(i)

Name an acid and an alkali which react to make potassium nitrate.


acid
alkali

(ii)

Explain why the titration method is suitable for the preparation of potassium
nitrate.

[4]

(b)

Lead (II) iodide is a salt which can be prepared by the precipitation method.
Name suitable reagents for the preparation of lead (II) iodide.
(i)
Reagents:

(ii)

Explain why the precipitation method is suitable for the preparation of lead (II)
iodide.

[4]
4

(A)

The diagrams below represent the flames on a burner using methane as the fuel.

;
V

FLL
E
I

..

(A) The equation for the complete combustion of methane in zone A is:
4 + 202
CH
2 + 2H
CO
0
2
(a) What colour would you expect the flame to be in
zone B
zone C
(b) (i)
(ii)

[2]

Explain what caUses the colour in zone C.


Give an equation for the combustion of methane in zone C.
Equation:
[3]

(C)

If the gas inlet provides 100cm


3 of methane per second, calculate the minimum
volume of air which should enter the air inlet, per second, to ensure complete
combustion of the methane in zone A.
Explain how you arrived at your answer.
[3]

4(B) (a) The structure of fluorochioroethene is shown below:


F

CI

C=C
H

Suggest a name for, and sketch the structure of, the macromolecule formed
when fluorochloroethene polymerises.
(ii) When 20 tonnes of fluorochioroethene polymerised, 18 tonnes of the polymer
were obtained. Calculate the percentage yield.
(iii) Name and give a use for a commercially available polymer containing a
[5]
halogen.
(i)

(b) Pure oxygen is obtained from air and is used as an aid to breathing in hospital
oxygen masks.
(i) Describe in outline how pure oxygen is obtained from air.
[3]
(ii) Give a commercial use for oxygen other than as an aid to breathing.
(C)

One problem with oxygen is that it is the essential element in causing corrosion
of metals.
One method of limiting corrosion is known as sacrificial protection.
[2]
Explain what is meant by sacrificial protection.

The incomplete diagram below represents the arrangement of the ions in crystals of
either sodium chloride or magnesium oxide.

N
2
a positive ion, Na or Mg

\
\

a negative ion, CI

or 02

l
(a) (I)
(ii)

Complete the diagram by adding the missing ions.


How does this crystal structure explain the fact that neither sodium chloride
nor magnesium oxide conduct an electric current when in the solid state?

(iii) What feature of the ions in the two compounds explains why the melting
point of magnesium oxide, 2800C, is much higher than that of sodium
chloride, 801C?

[6]
(b) Give a commercial use of magnesium oxide which depends on the high value for
its melting point.
[1]
5

(b) At 150C and a pressure 1 one atmosphere, the reversible reaction between gas
A and gas B reaches a dynamic equilibrium.
A(g) + 2B(g)
AB
(
2
g): 1H = 220 kJ/mol
(a) Is the formation of AB
2 exothermic or endothermic? Explain your answer.

[2]

(b) (i)

(ii)

What is meant by the phrase dynamic equilibrium?

Predict how the proportion of AB


2 at equilibrium changes if the pressure is
increased.

[4]
(C)

What effect will an increase in temperature have on the rate of formation of AB


?
2
Explain your answer in terms of the movement of the molecules.

[2]

CXC 0 Level Chemistry


Assessment Practice
Paper 03 Test 2
TIME:

1 hour 10 minutes.

INSTRUCTIONS TO CANDIDATES
Read the Instructions on the answer sheet very carefully.

Candidates MUST answer THREE questions on this paper. They must choose ONE from
Section 1, ONE from Section 2, and ONE from Section 3.
JI working MUST be shown for calculations.

Section I
Answer one (1) question from this section.
I

(a)

Cr are heated, the


2
)
4
(NH
,
7
When orange crystals of ammonium dichromate (VI), O
products are green chromium (Ill) oxide, nitrogen and water.
(I)
(II)

Construct the equation for this reaction.


By naming the reagents and giving the observations, explain how you would

confirm that
(1)

the orange crystals contain the ammonium ion,

(2)

the reaction products contain water.

(III) The action of heat on the crystals starts a redox reaction.


Has the chromium been oxidised or reduced?
[5]

Explain your answer.


(b)

Proteins are macromolecules and are a main constituent of food.


Name two other types of macromolecules present in food.
(I)
(II) Name the class of compounds formed when proteins are completely hydrolysed.
(UI) Describe, in outline, how the hydrolysis products from a protein can be separated
[5]
and identified.

(a)

The diagram represents an apparatus used to investigate the reaction between metals
and metallic oxides.

magnesium fuse

crucible

mixture of metal
and metallic oxide
sand

The results are given in the table


yes shows that a reaction took pIace

no shows that no reaction took place:


x shows that the mixture

was not tested.


2

metallic oxide

(I)
(II)

metal

aluminium
oxide

calcium
oxide

iron (Ill)
oxide

tin (IV)
oxide

aluminium

no

yes

yes

calcium

yes

yes

yes

iron

no

no

yes

tin

no

no

no

Why is the magnesium fuse included in the apparatus?


Place the four metals in order of decreasing reactivity.

(III) Construct the equation for the reaction between calcium and tin(IV) oxide. [3]
(b)

Describe how the action of heat on the carbonate of a metal depends on the position
[2]
of the metal in the reactivity series.

(C)

Transition metals and their compounds can be used as catalysts.

(d)

(I)

Explain what is meant by a catalyst.

(II)

By naming the reactants, the products and the catalyst, give an example of a
[3]
reaction which uses a catalyst.

Describe the bonding in metals. How does this bonding explain the ability of metals
[21
to conduct an electric current?
Section II
Answer one (1) question from this section.
How many electrons, protons and neutrons are present in one atom of the chlorine
.

(a)

isotope Cl?
(b)

When concentrated aqueous sodium chloride is electrolysed using graphite electrodes,


hydrogen is collected at the cathode and chlorine at the anode.
When concentrated aqueous sodium chloride is electrolysed using iron electrodes,
hydrogen is again collected at the cathode but much less chlorine is collected at the
anode.
i))

Give the equations for the eJectrode reactions by which hydrogen and chlorine
are formed.

(II)

Explain why much less chlorine is collected when iron electrodes are used.

(Iii) Name the product, other than hydrogen and chlorine, which is manufactured by
the electrolysis of concentrated aqueous sodium chloride. Give a major use of
[5]
this product.
(C)

Why is the electrolysis of concentrated hydrochloric acid not used for the manufacture
[2]
of chlorine?

(a) Explain the following observations.


(i)

Solid sodium chloride does not conduct electricity but an aqueous solution of
sodium chloride does.

(ii) Aluminium conducts electricity but sulphur does not.


(iii) Hydrogen chloride conducts electricity when dissolved in water but not when
dissolved in the organic solvent methylbenzene.

(b) A dilute solution of copper (II) sulphate is electrolysed using platinum electrodes.
A current of 0.025 A is maintained until 96.5 coulombs of electricity has passed
through the cell.
(I)

Explain in terms of ions and electrons the changes taking place at the anode
and the cathode in the solution.

(ii) What changes would you expect to observe at the anode, at the cathode
and in the solution?
(iii) For how many seconds must a current of 0.025 A be maintained to pass
96.5 coulombs of electricity?

(iv) Calculate the mass of copper deposited during the experiment.


(v) In a similar experiment it was found that 1.0 gm of a metal M and having at
R.A.M. of 200 was deposited by 96.5 coulombs. Calculate the mass of
metal deposited by 1 mole of electrons and so deduce the formula of the
ions of M present in the solution.

Section III
Answer ONE question from this section.
5. (a) Describe the differences between fibres, fabrics and textiles.
(b) List four desirable properties of textiles.
(C)

[31
[4]

Describe what is done when a fibre does not, possess all the desired properties of
a good textile.
[3]

(d) Describe how natural fibres can be dyed.

[6]

(e) Give a definition of a mordant and show, using equations how two different
mordant can be formed.
[4]

6. (a) What is meant by a culture solution?


(b) Describe how you would prepare a culture solution.

[2]
[4]

(c) Potassium, Phosphorous and Magnesium are three important elements needed
for plant growth. Plan and Design an investigation to show the effects of the
deficiency of there elements.
[8]
(d) At the end of the experiment in (c), list two measurements that can be taken. [4]
(e) What would be the effect of growth on this experiment?

[2]

CXC 0 Level Chemistry


Assessment Practice
Paper 01 Test 3
TIME:

1 hour 15 minutes.

INSTRUCTIONS TO CANDIDATES
There are sixty (60) questions in this paper. Answer all questions. For each questio
n,
there are four possible answers A, B, C and D. Choose the one you consider correct
and
record your choice in soft pencil on the separate answer sheet.
Read the Instructions on the answer sheet very carefully.

INFORMATION FOR CANDIDATES


Each correct answer will score one mark.
answer.

A mark will not be deducted for a wrong

Any rough working should be done in this booklet.

__
___

In the Haber Process, nitrogen and hydrogen react to form ammonia.


Which set of diagrams represents the molecules of nitrogen, hydrogen and ammonia?
nitrogen
A
1%

ammonia

hydrogen
0

I,J

Trichloroethane is a solvent used to remove grease from clothing.


How, after use, is the solvent separated from the grease?
A

by chromatography

by crystallisation

by distillation

by filtration

The table shows the melting and boiling points of four pure substances.
evaporate if left expbsed
Which substance is a liquid at room temperature and would rapidly
to the air?

substance

melting points 1C

boiling points IC

-100

-35

-7

58

-6

225

44

280

completely molten and the melt was


A sample of a pure compound was heated until it was
aga:n. The graph shows how the
then allowed to cool until it was completely solid
temperature of the compound changed with time.
When were liquid and solid both
present during cooling?
A

PtoQandRtoS

PtoQ

OtoR

RtoS

temperature

0
2

lime

The table gives the atomic structures of four particles.


particle
W_X

Protons

17

17

20

20

neutrons

20

18

20

21

electrons

17

18

18

20

Which particles combine with each other to give an ionics solid?


X and Y
C
W and Y
B
Wand X
A

It has an ionic lattice.

It has the same structure as diamond.


Its structure contains rings of carbon atoms.

Which of the following could describe a non-metallic element?


element conduction of
electricity

X and Z

Which statement about graphite is correct?


It conducts electricity becaus* graphite has ions which are free to move.
A

0
7

melting point

type of bonding
in its oxide

good

high

ionic

good

low

covalent

poor

high

ionic

poor

low

convalent

Which of the following is likely to be a pure compound?


A

a white powder which dissolves in water

green crystals which melt at 58C

blue crystals which melt over the range 55C to 60C

a liquid which gives two fractions when distilled

The gaseous element X is a diatomic. One volume of the element combines with one
volume of hydrogen to form two volumes of gaseous hydride.
Which is the formula for the hydride of X?
A

HX

2
HX

X
2
H

X
2
H

10

What is the ratio of the volume of 2g of hydrogen to the volume of 16g of


methane, both
olumes at r.t.p.?
A

11

12

13

lto8

lto2

What is the formula of nickel (II) hydroxide?


A
NiOH
B
2
N1OH

ltd

2tol

2
Ni(OH)

Ni
O
2
H

What are the products when concentrated aqueous sodium chloride is electrolysed using
inert electrodes?

anode (+)

cathode ()

chlorine

hydrogen

chlorine

sodium

oxygen

sodium

oxygen

hydrogen

The diagram shows the results of an electrolysis experiment using inert electrodes.
Which of the following could liquid X be?

._

aqueous copper (II) sulphate

concentrated hydrochloric acid

dilute sulphuric acid

ethanol

liquid X

III IIIIIIIIIIU IUUuI.LL.I.L.I.jL

-II,
14

62 into
The scheme shows four stages Ito IV in the conversion of solid candlewax CH
carbon dioxide and water.
II

I
CH(s)

-,

CH(l)

Ill
CH(g)

-,

3000
(
2
g)

3GCO(g)

31H
0
2
(g)

31H
0
2
(l)

Which stages axe exothermic?

landli

IlandlIl

lIlandlV

andlV

15

When an excess of calcium carbonate is added to dilute hydrochloric acid, the reaction
gradually becomes slower and finally stops.
Which statement best explains why this happens?
A
An insoluble layer of calcium chloride is formed on the calcium carbonate:

16

The pieces of calcium carbonate gradually become smaller.

The calcium carbonate is covered by bubbles of carbon dioxide.

The concentration of hydrochloric acid gradually reducesto zero.

Which of the following is a compound formed by photosynthesis?


A

17

carbon dioxide

glucose

nitrogen

water

An aqueous solution has a pH value of 1.


What information does this fact give about the ions present in the solution?
concentratioh of H ions

concentrations of 01-Lions

high

none

high

low

low

D
18

high

low

An element X burns in oxygen to produce a white oxide, which is sparingly soluble in


water, giving an alkaline solution.
Which of the following could be X?
A

19

20

calcium

copper

phosphorus

sulphur

Which of The following pairs consist of substances that produce a precipitate when their
aqueous solutions are mixed?
A

sodium chloride, barium nitrate

sodium nitrate, barium chloride

sodium nitrate, silver nitrate

sodium sulphate, barium chloride

Tests on a sample of polluted water from a factory gave the following results.
reagent added

result

hydrochloric acid and


aqueous barium chioride

white precipitate

aqueous ammonia

white precp;ate
insoluble in excess

Which compound could have been present in the water?


B

aluminium chloride
aluminium sulphate

zinc chloride

zinc sulphate

21

22

Which element is least likely to form an oxide?


C
helium
B
A
chlorine

hydrogen

nitrogen

The table gives the formulae of the catalysts used in some industrial processes.
catalyst

process

Haber process
Contact process

Fe

Mo

5
0
2
V

cracking of alkanes
Polymerisation of ethene

3
0
2
A1

manufacture of silicones

CuC!

2
Si0

C
2
A1(
3
)
5
H

4
TiCI

or as compounds) in the
How many different transition metals are included (as elements
list of catalysts?
06
5
C
84
3
A
23

bol Ha, which has seven outer


Which statement could be made about a new element, sym
electrons in its atoms?
It is monatomic.
A
It forms a covalent comeound with hydroge
B
C

D
24

It forms a positive ion Hat.


It forms covalent compound with Group I elements.

ity series for metals X, Y and Z. The


Experiments were carried out to construct a reactiv
table shows the results.
rnetalZ
metalY
metaiX
test
yes
no
yes
Does the metal liberate hydrogen
from dilute hydrochloric acid?
no
yes
yes
is the metal oxide reduced by
carbon
heating with 9
What is the order of reactivity of themetals?

most reactive

25

least rective

After copper (II) oxide h2s been reduced to copper as shown, the burner is turned off but
the hydrogen is kept Jwing until the tube is cold.

excess hydrogen

dry
I

hydrogen

burner

What is the main reason why the hydrogen is kept flowing


to lessen the risk of explosion in the hot tube
A

26

to make certain that the reaction is complete

to prevent the copper from reacting with the air

to remove any traces of water left in the tube

?
3
0
2
What is the mass of alUminium in 204 g of aluminium oxide, A1
A

27

26 g

27g

54 g

108 g

sodium

The diagram shows a metal X reacting with water


bubbles of hydrogen

water

metal X

Which of the following could X be?


A

calcium

copper
7

potassium

28

29

What is essential in the conversion of pig-iron into steel?


A
a decrease in the amount of carbon present
B

the addition of sulphur

the addition of zinc

the oxidation of iron (Ill) oxide

Air contains about 21% oxygen by volume. When a sample of river water was boiled the air
expelled was found to contain 30% oxygen.
What is the best explanation of this difference in oxygen content?
A
Carbon dioxide is more soluble in water than in oxygen.

30

Nitrogen reacts with water.

Oxygen is more soluble in water than in nitrogen.

The noble gases are insoluble in water.

An NPK tertiliser is made by mixing together one mole of each of three compounds.
Which mixture will contain equal numbers of atoms of nitrogen, phosphorus and potassium?

components

31

S0
2
K
4

(P0
3
Ca
2
)
4

3
KNO

NO
4
NH
3

4
Na2HPO

3
KNO

C0
2
)
4
(NH
3

HPO
2
K
4

3
NaNO

S0
2
)
(NH
4

(PO
3
Ca
)
4

S0
2
K
4

3 sulphuric acid?
What is the concentration of hydrogen ions in 0.05 mol/dm
A

32

mixture

3
0.025g/dm

3
0.05 g/dm

3
0 lOg/dm

3
0.2g/dm

Chlorine reacts with cold dilute aqueous sodium hydroxide.


Which of the following could be the balanced equation for this reaction?

2NaOHNaCli-H
C1

2
0

-i- 2NaOH
2
C1

NaCI

2Ct+NaOH

2
NACIHCI+0

2Ct+ 2NaOH

2NaCI

NaC1O

2HCI

0
2
H
2HCI

+ 02

33

Which diagram represents the structure of a silicone?


B
A
II

9i-0-Si--0
SiO--Si 0--

siDsifl
cc
0

fJ

11

EEi
0SiOSi0

DOSiD0Si

ES
34

35

Which pair of substances are allotropes?


A

C and C

carbon dioxide and carbon monoxide

diamond and graphite

6
H
3
4 and C
H
2
C

The structural formula of butenedioic acid is shown.


H

IIo

HC

0H

Which statement about butenedioic acid is not correct?

36

it decolourises bromine.

B
C

Its aqueous solution reacts with sodium carbonate.


Its empirical tormula is the same as its molecular formula.

Its relative molecular mass is 116.

Ethanol was oxidised to ethanoic


acid using the apparatus shown.

water out

water in

condenser

heat
9

ethanol

reforming agent

The purpose of the condenser was to prevent the


A
conversion of the ethanol to ethene.
B

37

escape of any unchanged ethanol.


reaction of the ethanoic acid with ethanol.

reforming of ethanol from the ethanoic acid.

The macromolecules of proteins, fats and carbohydrates can all be broken down into their
simple units by a similar process.
What is the process called?
A

38

39

hydrolysis

oxidation

reduction

Which of the following is an example of a polyester?


A
nylon
B

poly(ethene)

poly(chloroethene), pvc

Terylene

Which statment about the members of a homologous series is correct?


A
They contain the same number of bonds per molecule.
B

They have the same empirical formula.

They have the same number of carbon atoms per molecule.


They undergo similar reactions.

D
40

esterification

A 10 cm
3 sample of a gaseous hydrocarbon was completely burnt in oxygen. The total
volume of the products was 70 cm
.
3
Which equation represents the combustion of the hydrocarbon?
A
(g) + 20
4
CH
(g) C0
2
(g) + 2H
2
0(g)
2
B

C
(
4
H
2
g)

30
(
2
g)

2C0
(
2
g)

2H
0
2
(g)

C
(
8
H
3
g)

50
(
2
g)

3C0
(
2
g)

4H
0
2
(g)

2C
(
6
H
2
g)

70
(
2
g)

4C0
(
2
g)

6H
0
2
(g)

10

V
24

Cr

Fr

99

776

89

57

Ac

tnsn

Key

x aonic so

bpm(on(alm*)ntxnbe

1= r.Iiv. at1

mill

72

9.

c..,..,,,

54

179

Hf

136

La

137

Ba

133

Cs

90

89

73

41
III

29

Mn

Fe
27

Co
26

Ni
29

30

222

93

Pj,,
91
93

Np
te4,

U
*J4.

Pa

Pu
94

P*.n

150

Sm

99

Am
16

97

Bk
b1b4.

Cm
Cm

99

Cf
90

Es

10?

Fm

No
l4
10?

Md
Mb.im
101

LWa
*0?

Lr

U,n

11

91I.

70

1)4.n
E,*.n
Dr.o..W

46

ti,.

175

Lu

173

Yb

189

Tm

167

Er

165

Ho

152

89

Dy

99

94

159

99

52

Rn
P.

At
99

54

A.b*.

i.

131

Xe

Kr
127

36

II

c.

Th

97

62

53

39

17

40

Ar
Cl

35.5

lbo

157

91

Po

Te

12*

Se

32

10

Gd
65

93

152

66

79

52

34

16

20

Ne

F
0

II

VII

16

VI

Eu
(,n,..n,
93

70

3 at room temperature and pressure (r.t.p.)


The volume of one mole of any gas is 24 dm

Th

236

Pln*t..,.
61

N,,...,
03

Cu.,,

Pm

144

Nd

141

77

InIl

Bi
L

G4d

P*,

Uy

Ti

9,n

200
207

Pb

204
201

Hg

197

51

Au

199

80

Sb

122

As

Pt

..

33

Jr

49

118

Sn

In

32
115

Ge

192

49

Ga
G..,n

190

47

31

15

A._.._

14

13

31

29

Si

27

Al

Os

46

5E.

P.n

45

Cd

Ag

Pd

Rh

Ru

Pr

76

44

112

109

Z.

106

Zn

Cu
Co

103

N.I

101

140

75

9?...m

wn

50

74

199

Re

194

TWW
43

Tc

26

Ce

Ta

42

rb

Zr
40

Sr

SI

Rb

6.*

31

Mo

Nb

36

96

93

23

V.,,,

91

58-71 Lanthanoid series


t90-1 03 Actinoid series

87

22

11

99

55

37

21

Sc,,

99

96

20

KC

I2

IQr

24

Mg

23

Na

Po,

Sc

Ie1

t__,
5

Be

14

12

Ii

.L_______

Group

DATA SHEET
The Perfoc Table of the Elements

41

42

43

44

45

Which one the following is not an alkali?


A
sodium hydroxide
B

calcium hydroxide

copper hydroxide

ammonium hydroxide

Which of the following methods is NOT suitable for softening water that is permanently
hard?
A

distillation

iron-exchange

boiling

addition of slaked lime

Ethanoic acid is a weak acid because:


A

It will react only with very active metals

It is not a good conductor of electricity

It is not very soluble in water

It forms only few hydrogen ions in solution

In the reaction: X + 2H
X+ H,
copper and sulphuric acid
A

silver and nitric acid

sodium and sulphuric acid

magnesium and hydrochloric acid

which is the most suitable pair of substances?

Metals high in the electrochemical series are usually extracted from their ores by:
A
Reduction of the oxide with carbon
B

Electrolysis of the fused hydroxide

Electrolysis of the fused chloride

Decomposition of the carbonate in an electric furnace

12

46

47

A tap funnel is used to separate:


A
a mixture of two solids
B

a solute and a solvent

immiscible liquids

miscible liquids

What volume of propane is burnt for every 20 cm


3 of oxygen used in the following
reaction?
8
H
3
C
A

48

3C0

502

3
2cm

0
2
4H

3
4cm

3
8cm

The number of coulombs equivalent to 2.5 amp flowing for 8 mins is:
C
A
60
600
B
1200
D

10 cm
3

1800

Items 4951 refer to the following data:


Which of these terms best fit the reactions listed in 49
A

Polymerization

Addition

51

Hydrolysis

Dehydration

49

Protein boiled with dilute mineral acid

50

Ethene changing to 1, 2-dibromoethane

51

Ethanol changing to ethene

52

In which of the following does no chemical change occur?


A
Zn and CuSO
4 solution
B
Fe and CuSO
4 solution
C
Mg and ZriSO
4 solution
Pb and ZnSO
D
4 solution

53

If the following ions are in solution in equivalent amounts, then the one most likely to be
discharged at the anode on the passage of an electric current would be:
A

2
4
S0

OW

13

CF

54

Caesium is an element similar to that of potassium. The equation for the decomposition
by heat, of caesium nitrate is most likely to be:
2 +02
2CsNO
3
2CsNO
2 Q2
B
2Cs0 + 4N0
A
3
4CN0

C
55

B
C
D

2
2N0

+02

2
)
3
2Cs(N0

CsO

2
4N0

differing numbers of protons, neutrons and electrons


the same number of protons and neutrons but differing numbers of electrons

polyester

COOH

polystyrene

The type of linkage formed in the manufacture of nylon is referred to as:


amide linkage
B
ester linkage
A
C

peptide linkage

glycosidic linkage

58

Which of the following would NOT speed up the reaction between magnesium and
hydrochloric acid?
Adding concentrated acid
B
Heating the reaction flask
A
Using finely divided magnesium
D
Adding water
C

59

Which of the following is NOT as alternative source of energy?


Biogas
C
Crude oil
B
Wind energy
A

60

+02

the same number of electrons and protons but differing number of neutrons
the same number of electrons, protoiis and neutrons

The polymer formed by the reaction of compounds; HOOC X


is which of the following:
polyamide
B
polysacehande
A
C

57

2Cs

Isotopes of the same element contain:


A

56

3
2CsNO

Solar energy

The action of dilute nitric acid on copper can be represented by the equation:
0 + 2N0
2
x Cu(N0
2 + 4H
)
3
3
X Cu + y HNO
The values of x and y are:
B
x=3,y4
A

x2,y6

14

x3,y8

x=1,y3

CXC 0 Level Chemistry


Assessment Practice
Paper 02 Test 3
TIME:

1 hour 30 minutes.

INSTRUCTIONS TO CANDIDATES

Read the following directions carefully.

1. You must use this booklet when responding to the questions. For each question, write
your answer in the space provided.
2. All working must be shown.
3. There are five questions in this booklet. Answer all five questions.

vtl

You should NOT spend more than 30 minutes on Question 1.


A student was provided with two solutions.
3 and R.M.M.
3 containing 5 g/dm
BA is a substance XCO
1
.
3
hydrochloric acid containing 0.1 moll dm

(a)

2 is dHute
106. BA

1 in order to determine the values


2 against BA
The student is required to titrate BA
of n and X.
Figure I below displays the readings from the burette before and after each
1
25/cm of BA
titration against 3

30

28

29

29.

31

Titration 3

Titration 2

Titration i

30

Figure 1

(I)

Use the informtion if Figure 1 above to complete the following table of results.
TITRATION

Burette Reading

Final Reading
Initial Readtng

cm

1
-

2
Volume of solution BA
usedIcm

(..,

(ii)

2 should be used in the calculation?


What volume of solution BA

(iii)

3
1 in molt dm
Calculate the concentration of BA

(I)

How many moles are there in the average volume of HCI calculated in (i)?
.ib$

(ii)

J(O

How many moles of HCI react with 1 mole XCO


?
3

(iii) What is the most likely value of n?

.c-
(iv) What is the most likely value of X?
11
/,v
[R.A.M.

12, 0

16]
water out

(b)
tOC

water n

ethyl butanoeie

heat

The above figure shows the first stage in the production of ethanol by the
reaction of heat on a mixture of ethyl butanoate and sodium hydroxide.
(i)

What re the products of this reaction?

(ii)

Name the type of reaction involved.

(iii) Explain how the arrangement of the condenser might help to promote the
completeness of the reaction.

(iv)

Draw a diagram of the final arrangement of the apparatus needed to collect


the distillate of one of the products.

(v)

State a precaution which must be taken when performing this experiment.

2A Write the name of one example of each of the following:


a green solid which decomposes on heating to form carbon dioxide;
(I)
(ii)

a gas which turns moist red litmus paper blue;

(iii)

an acid that forms a white precipitate when mixed with aqueous barium nitrate;

(iv) a metal which is extracted by electrolysis,


r%.LLA

2B The table below shows a range of substances containing chlorine.


use
formula
substance
making pvc
2
Cl
chlorine
hydrogen chloride

HCI

making dyes and glues

sodium chlorate(l)

NaOC1

household bleach

sodium chtorate(V)

3
NaCIOO

weedkiller

sodium chloride

NaCI

food preservation

(i)

Which substances exist as covalent molecules?

(ii) In which compound does chlorine show an oxidation state of +1?

(iii) Household bleaches containing sodium chlorate(1) can be made by reacting chlorine
with sodium hydroxide.
0
2
NaCI+ NaOCI + H
2
2NaOH + Cl
Explain why this is a redox reaction.

(iv) Give another use of chlorine, Cl, not mentioned in the table.
[4]
3A

, burns in oxygen to give a very hot Ilame. The temperature is so high that the
H
2
Ethyne, C
flame can be used to cut metal.

c)
5

(a)

(I)

Name the two products of the complete combustion of ethyne.


(1)

(II)

(2)

..

Describe a chemical test for one of these products.


..

(b)

Calcium carbide, CaC


, is a greyish-white solid. Ethyne can be made by the reaction
2
of calcium carbide with water. Calcium hydroxide, Ca(OH)
, is the other product of
2
the reaction.
(I)

Construct an equation, inchjding state symbols, for the reaction between calcium
carbide and water. The molecular formula for ethyne is

(ii)

The mixture obtained after reaction was tested using Universal Indicator.
What was the final colour of the Universal Indicator? Explain your answer.
6]

3B The table below shows some data about the homologous series of alkanes.
alkane

number of carbon
atoms per molecule of
alkane
n

melting
pointiC

boiling
pointiC

energy released by
complete combustion
of one gram! kJ
E

number
of
isomers

methane

183_

162

55.6

ethane
propane

183
188

89

52

42

1
1

butane
pentane

2
3
4
5

138
130

0
36

49.7
49.2

2
2

hexane
heptane

6
7

95
91

69
96

48.8
48.5

5
9

(i)

Plot a graph E against n. Using the grid


provided below.

(ii)

Use your graph to estimate the value


of E for propane.
Put your answer in the table.

(iii) Calculate the energy released by the


complete combustin of one mole ot
methane.

(iv) Give the molecular formula of an alkane that is a liquid at roori temperature and
pressure.

(e)

Draw the structural formula for the two isomers of butane.

(f)

Explain the term homologous series.


[11]

4A

The element oxygen exists in two forms, 02 03


Both forms are gases.
(a)

b)

(1)

Ozone molecules, 03, decompose when heated into


Construct the equation for this decomposition.

(ii)

3 of ozone, O3 is decomposed,
What volume of oxygen, O2 is formed when 40cm
both volumes being measured at the same temperature and pressure?

molecules.

Ozone is an oxidising agent.


Describe the colour change you expect to see when ozone is bubbled into aqueous
potassium iodide.
[4]
Colour after
Colour before

4B The table below shows some data about some of the components of air.

component

melting pointiC

boiling point IC

argon
carbon dioxide
krypton
neon
nitrogen
oxygen
water vapour

-189
78
157
249
210
219
0

186
78
153

246
196
183
100

(a)

State the percentage by volume of nitrogen and oxygen in the air.


Oxygen
Nitrogen

(b)

Common pollutants of the air are not included in the table.


Name one atmospheric pollutant and state its origin.

(C)

Air is a source of nitrogen, oxygen and the noble gases. These are obtained by the
fractional distfllaiton of liquid air. Before air is liquefied, carbon dioxide and water are
removed.
(I) Suggest why air is dried before it is liquefied.

.Qi)

Liquid air at 200 C is fractionally distilled by allowing it to warm up gradually.


List the order in which the elements present in this liquid air will boil off.
[6)

5A

Use the list of elements to answer the questions that follow.


bromine

carbon

chromium

helium

magnesium

phosphorus

sodium

sulphur

In your answers, you may use an element once, more than once, or not at all.
Choose one element which
(i)

froms a basic oxide,

(ii)

has a high density and torms coloured compounds,

(iii) is composed of diatomic molecules,


(iv) is a liquid at room temperature and pressure,
(e)

exists as allotropes.
[5]

5B

The elements shown below are all in Group I of the Periodic Table.

element

symbol

lithium
sodium
potassium
rubidium
caesium

Li
Na
K
Rb
Cs

electronic structure
2,
2,
2,
2,
2,

1
8, 1
8, 8, 1
8, 18, 8, 1
8,18, 18, 8, 1

(a)

Which element reacts most vigorously with cold water?

(b)

Write the formula of a caesium ion.

(C)

How many protons are there in a rubidium ion?

(d)

Group I elements are very good conductors of electricity.


Use a simple model of the structure and bonding of a metal to explain this.
[5]

CXC 0 Level Chemistry


Assessment Practice
Paper 03 Test 3

TIME:

1 hour 10 minutes.

INSTRUCTIONS TO CANDIDATES

Read the Instructions on the answer sheet very carefully.

Candidates MUST answer THREE questions on this paper. They must choose ONE from
Section 1, ONE from Section 2, and ONE from Section 3.
All working MUST be shown for calculations.

Section I
Answer one (1) question from this section.
I

The sugar glucose has an empirical formula of CH


O and a relative molecular mass. Mr =
2
80.
(a)

Calculate the molecular formula of glucose.

(b)

If an aqueous solutin of glucose is heated with alkaline copper (II) sulphate, a brickred
precipitate is formed. The precipitate is an oxide of copper.

[1]

A 1.44 g sample of this oxide was founc n contain 1.28 g of copper.

(i)

Calculate the empirical formula of this oxide of copper.

(ii)

What is the oxidation state of the copper in this oxide?

(iii)

Why does this reaction show that glucose is a reducing sugar?

[4]

(c)

Describe the conditions needed for the fermentation of glucose to give ethanol.

(d)

(I)

Name an organic product formed when ethanol reacts with acidified potassium
dichromate (VI).

(ii)

Draw the structural formula of the organic product you have named.

[3]

[2]

Liquid hydrogen is used as a rocket fuel. It is used in space shuttles.


(a) (i)

Another gas must be earned by a space shuttle in order to burn the hydrogen.
What is the name of this gas?

(ii)

Suggest a reason why a space shuttle carries liquid hydrogen rather than
[2]
hydrogen gas.
.

(b) (i)
(ii)

Write the equation for the combustion of hydrogen.


One mole of hydrogen releases 282 kJ of energy when, it is burnt.
What is the amount of energy released when 10 tonnes of hydrogen bum?
[1 tonne is 1 million grams.]

(iii)

(c) (I)

In terms of the bonds broken and the bonds formed, explain why the burning of
[6]
hydrogen is exothermic.
It has been suggested that hydrogen could be developed as an alternative fuel for
the future.
Suggest one advantage hydrogen has over fossil fuels.

(ii)

Give one other ndustrial use of hydrogen.

[2]

Section II
Answer one (1) question from this section.
The table below showsthe results of an experiment using 0 lOg of magnesium ribbon
and
3 of 2.0 mol/dm
25cm
3 sulphuric acid. In the experiment, magnesium ribbon was mixed
with dilute sulphuric acid and the total volume of hydrogen produced was measured every
live seconds. After fifty seconds, the rection had stopped
time from start of

total volume of hydrogen

experiment/s

3
produc
edm

31

10

54

15

71

20

85

25

94

30

99

35

100

40

100

45

100

50

100

(a)

Draw a labelled diagram of suitable apparatus for this experiment.

(b)

The equation for the reaction of magnesium and dilute sulphuric acid
is given below
Mg +H
4
S
2
0

(c)

13)

2
H
MgSO

(i)

Calculate how many moles of magnesium and of sulphuric acid were


used in
this experiment.

(ii)

Hence explain why the reaction stopped

[2)

The experiment was repeated using the same mass of magnesium but
25 3
cm of 3 0
3 sulphuric acid.
mol/dm
State and explain how
(I)

(d)

(ii)

the inital rate of formation of hydrogen,


the total volume of hydrogen collected when the reaction had
stopped, compared
with the original experiment.

Give

two other methods of changing the initial rate of reaction.

(.

\
c

[5]

Silicon and carbon are both in Group IV of the Periodic Table.


(a)

Draw a diagram of an atom of silicon, showing how the electrons are arranged and
[3]
the numbers of protons and neutrons in the nucleus.

(b)

Carbon and silicon each form dioxides in which the bonding between the atoms is
covalent. Carbon dioxide has a simple molecular structure whereas silicon (IV) oxide
(silicon dioxide) has a macromolecular structure.
(I)
(j)

Draw a dot-and-cross diagram to show the bonding in a molecule of carbon


dioxide.
The structure of silicon (IV) oxide is drawn below.

0
Sirb

/ \

0\

Si
0

Silicon (IV) oxide has a high melting point whereas carbon dioxide is a gas.
Explain this in terms of the structures of the two oxides.
(iii) Does silicom (IV) oxide conduct electricity? Explain your answer with
[5]
reference to its structure.
(c)

Silicon (IV) oxide is an important raw material.


Give
(I)

one source of silicon (IV) oxide,

(ii)

one use of silicon (IV) oxide,

[2]

Section III
Answer ONE question from this section.
5

(a) Using illustrations, show the differences in the methods by which acid dyes and
basic dyes work on fabric.
[4]
(b) Give an example of one acid dye and one basic dye.

[2]

(c) Describe the process of mordant dyeing.

[6]

(d) Describe the differences between mordant dyeing and direct dyeing.

[3]

(e) Describe how one dye, e.g., alizarin, can be used to produce two different
colours in mordant dyeing.
[5]

6.

I
N

/
H

C\

0H

The above diagram represents that of the amino acid.


(a) State two important functional groups represented by the structure.
[4]
(b) Show by means of a diagram the kind of linkage formed when two amino
acids
link up to form part of the protein molecule. Name the type of linkage.
[4]
c) C 0 (NH
1
) urea, and (NH
2
2 SO
)
4
4 ammonium sulphate are two fertilizers used
in agriculture. Calculate the percentage of nitrogen in each fertilizer.
(R.A.M.

C=12

0=16

N=14

H=1

S=32)
[4]
(d) Discuss the use of organic manure versus commercial fertiliz
ers in terms of cost
and effect on the soil.
[4]
(e) Ammonium sulphate is a soluble salt. Ammonia is a soluble
base Suggest and
describe a method you would use to make a sample of ammo
nium sulphate. [4]

CXC 0 Level Chemistry


Assessment Practice
Paper 01 Test 4
TIME:

1 hour 15 minutes.

INSTRUCTIONS TO CAND
IDATES
There are sixty (60) quest
ions in this paper. Answ
er aH questions. For ea
there are four possible an
swers A, B, C and D. Ch
ch question,
oo
record your choice in soft
se the one you consider
correct and
pencil on the separate an
swer sheet.
Read the Instructions on
the answer sheet very ca

refully.

INFORMATION FOR CA
NDIDATES
Each correct answer wil
l score one mark.
answer.

A mark will not be dedu


cted for a wrong

Any rough working shou


ld be done in this booklet
.

th a mixture of
nsde a balloon tilled wi
s
cle
rti
pa
of
nt
me
ge
arran
Which diagram shows the
n?
helium and argo
D
key
C
B
A
...heiium atom

70

00 0

oQo

0.. .argon atom

/0

00

00000

t soluble in water.
ils at 12OC and is no
bo
C,
9O
at
lts
me
X
Compound
of X with water?
pure X from a mixture
n
tai
ob
to
ed
us
be
n
D
Which apparatus ca
B

the apparatus
onal distillation using
cti
fra
by
ter
wa
d
an
parate ethanol
A student tries to se
shown.
water out
condenser

water in

receiver

heat

Which error has the student made?


The condenser is at the wrong angle.
A
The thermometer is in the wrong position.
B

4
D

The top of the receiver should be open.


The water enters the condenser in the wrong place.

3 of water. They then each


cm
Five students each dissolved an indigestion tablet in 100
using the same indicator.
acid,
25.0cm of their solutions with dilute hydrochloric
titrated 3
The results are shown in the table.
5
4
3
2
1
student number

3
titration value 1cm

20.4

20.4

20.5

20.6

22.0

r 5?
t
Which statement could explain the result obtained by studen numbe
The burette was washed out with hydrochloric acid.
A
The pipette was washed out with the tablet solution.
B

4
O

The student measured to the top of the meniscus in the pipette.


The titration flask was washed out with the tablet solution.

Two particles X and V have the structures shown.

e=electron
p proton

Which term describes X and Y?


A
6

allotropes

ions

isomers

Element Phasan electromc configuration of 2,8,6


configuration of 2, 8, 8, 1
What is likely to form if P and R combine?
a covalent compound of formula PR
A
C

R
6
a covalent compound of formula P
an ionic compound of formula PR

2
an ionic compound of for mula PR

isotopes

Element A has an electronic

Which of the following ions contains


the smallest number of electrons?
Q2
A
B
K
C
Cl
D
2
Ca

The diagrams represent tour different


compounds
2

In which line are the compounds correctly named?


1
2
3
A
ammonia
sodium chloride
methane
B
methane
ammonia
sodium chloride
D

water
water

water

ammonia

methane

sodium chloride

water

methane

ammonia

sodium chloride

An element Eforms a hydride EH


3 which contains 90.0 % of E, by mass.
What is the relative atomic mass of E?
A

10

27

30

87

90

Which quantity is the same for one mole of ethanol and one mole of ethane
?
A
mass

C
P

number of atoms
number of molecules
volume at r.tp.

11

Which of the following, when added to water, makes a solution that is a good conductor of
electricity?

12

calcium carbonate

copper

ethanol

sodium hydroxide

Aqueous copper (II) sulphate is electrolysed using copper electrodes. The current is constant
and the cathode (negative electrode) is weight at regular intervals.
Which graph is obtained when the mass of the cathode is plotted against time?

0
13

time

time

time

The diagram shows a simple cell.


flow of electrons

....wire

metalX

metalY
dilute

sulphuric

For which pair of metals would electrons flow as shown in the


diagram?
x
y

iron

zinc

zinc

calcium

zinc

magnesium

zinc

iron

time

14

The table shows the energy released by the complete combustion of some compounds
used as fuels.
compound

formula

Mr

H, in kJ/mol

methane

4
CH

16

880

ethanol

H
C
O
5
H
2

46

1380

propane

8
H
3
C

44

2200

heptane

6
C
1
H
7

100

4800

Which fuel produces the most energy when ig of the compound is completely burned?
A

15

16

17

18

heptane

ethanol

methane

propane

Which change is an example of oxidation?


A

chloride ions to chlorine atoms

copper (II) ions to copper atoms

iron (Ill) ions to iron (II) ions

oxygen atoms to oxide ions

In which reaction is the underlined substance acting as a reducing agent?


iron (Ill) chloride

chlorine + iron (II) chloride


A
copper (H) oxide

hydrogen

hydrochloric acid

zinc oxide

water

copper

magnesium chloride

zinc

magnesium oxide

carbon monoxide

watei

carbon dioxide

Which process does not involve either oxidation or reduction?


forrnaUon of ammonium sulphate from ammonia and sulphuric acid
A
B

formation of nitrogen monoxide from ammonia

formation of sulphuric acid from sulphur

formation of zinc from zinc blende (ZnS)

A man suffering from an excess of acid in his stomach has no indigestion tablets.
7
Which substance could he now take to lower this acidity
A

aspirin (pH6)

bicamDnate of soda (pH 6

lrCi juice (pH5)

salt water (pH7)

19

Aluminium sulphate is sometimes used in water treatment to remove impurities. Aqueous


aluminium sulphate is acidic.
The table shows the results of tests on tour different samples of treated water.
To which sample had an excess of aluminium sulphate been added?

20

pH of
sample

reaction with an excess


of aqueous ammonia

white precipitate

no reaction

no reaction

11

white precipitate

Which ion, in aqueous solution, forms a white precipitate when dilute sulphuric acid is
added?
A

21

sample

Ag(aq)

aq)
Cu
(
2

aq)
Ba
(
2

Fe(aq)

Salt P0 is to be prepared by reacting the carbonate of P with the acid HO. The titration
method is to be used to carry out the preparation.
What are the solubilities of the carbonate, the acid and the salt?

22

the carbonate of P

the acid HO

the salt P0

insoluble

soluble

insoluble

insoluble

soluble

soluble

soluble

insoluble

insoluble

soluble

soluble

soluble

The structure of a compound containing an element Y and hydrogen can be represented


as shown in the diagram.
Only outer shell electrons are shown.
H

Key
x= Yelectron

H electron

To which group of the Periodic Table does element Y belong?


A

Ill

IV

VI

23

19

Fluorine,

F, is a halogen. The table shows some of the properties of other halogens.

element

number of
valency
electrons

melting
point/C

boiling
pointfC

colour of
vapour

chlorine

71

101

-35

yellow-green

bromine

160

+59

red-brown

iodine

254

+114

+184

violet

Which statement about fluorine is likely to be correct?

24

25

The M of fluorine is 19.

The melting point of fluorine is +212C.

The boiling point of fluorine is 188C.

At r.t.p., fluorine is a black solid.

Which line lists the electronic configurations of three metals?


A

2,8

2,8,8

2,1

2,8,1

2,88,1

2,7

2,8,7

2,8,19,7

2,8,3

2,8,4

2,8,5

An element R reacts in the following ways:


2R
R

HO

-_-

2HCI
+

2R0
H
2
RCI
no reaction

2
H

Which of the following could be R?


A

26

aluminium

calcium

copper

iron

What describes the solution formed and the gas evolved when potassium reacts with cold
water?

solution

gas

alkaline

al k all r. e

alkaline

neutral

neutral

akaIine

neutral

n E J I 1?

27

The diagram shows the cell used in the electrolytic extraction of aluminium

cathode

Which reaction takes place at the electrode where the aluminium is formed?
A
Atoms gain electrons and form ions.

28

29

Atoms lose electrons and form ions.

Ions gain electrons and form atoms.

Ions lose electrons and form atoms.

Which property most strongly suggests that an element Zis a non-metal?


A
Zforms an acidic oxide.
B

Zhas a low melting point.

Zhas more than one oxidation state.

Zreacts vigorously with chlorine.

The apparatus shown is used to determine the percentage by volume of oxygen in the air.
substance X

ng?t1syring
tube
Which of the following could substance X?

30

anhydrous copper(ll) sulphate

calcium oxide

carbon

copper

Which compound does not provide one of the essential elements needed for plant growth?
A
ammonium sulphate
B

calcium hydroxide

calcium phosphate

potassium chloride
9

31

Which diagram represents a tire-resistant polymer?

C-CO-bO--C----CO--EZ_O_

CCNL]N(__N_E]_N_

o LI o

LI

LI

LI

34

35

LI
I

LI

LI

Which of the following ions is not present in aqueous sulphuric acid?

H(aq)

A
33

o LI

OSiOSi----OSiO

32

0H(aq)

SO(aq)

SO(aq)

What are the formulae for lime, limestone and slaked lime?
lime

limestone

slaked lime

CaO

3
CaCO

2
Ca(OH)

CaO

2
Ca(OH)

3
CaCO

2
Ca(OH)

3
CaCO

GaO

3
CaCO

2
Ca(OH)

GaO

Which of the following is not a use for chlorine?


A

sterilising water

making domestic bleaches

making table salt

manufacturing plastics

Which structural formula represents an unsaturated hydrocarbon?


/H

/GC\
H

A
H\

H\
HC-- C\

Cl

/CGCH
H

H
10

HCC-ccH
H

36

37

, reacts with
CO
2
CHJCH
What is the formula of the ester formed when propanoic acid, H
ethanol?
A

O
H
3
CH
C
2

O
H
3
CH
C
2

O
H
3
CH
C
2

O
H
3
CH
C
2

Poly(carbonates) are synthetic polymers. Their structure can be as shown.


0

Which of the following has a type of linkage similar to that of a poly (carbonate)?
starch
poly(propene) D
C
B
a polyester
A
a polyamide
38

39

In the tractional distillation of crude oil, which product has the highest boiling point?
A

bitumen

lubricating oil

paraffin (kerosene)

petrol (gasoline)

The diagram shows the structure of a monomer.


Cl
F

c=c

H
H
Which polymer can be made from this monomer?

Cl

Cl

I I
CCCc
I I I I
F
Cl

40

Cl

Cl

Cl

Cl

I I
CCCc
H

Cl

C---C-C-C

Which statement is true about ethanol?


A

It is formed by the cata(1ic addition of steam to enene.

B
C

It is an unsaturated compound.
It is formed by the oxidation of ethanoic acid

It reacts with ethyl ethanoate to form an acid


11

Cl

Cl

CL

I I I
CCCC
I I I I
H
H

41

The volume of hydrogen produced at R.T.P. when 0.1 mol of sodium is


treated with an
excess atwater is
A

1 2 dm
3

2.4 dm
3

4.8 dm
5

7.2 dm
2

[1 mol of gas occupies 24 dm


3 at R.T.P.]

42

Which one of the following suggests that matter is made up of particles in continuous
motion?
A

43

44

46

neLitralization

diffusion

melting

The formation of ions in an electrovalent compound is thought to be due to:


A
The passage of an electric current through the compound
B

The transfer of electrons from metal atoms to non-metallic atoms

The sharing of electrons between a metal and a non-metal atom

The charging of atoms by friction as metallic and non-metallic atoms collide with
one another

A gas which decolourizes aqueous bromine is:


A

45

oxidation

ethane

ethene

methane

carbon dioxide

Diamond and graphite have different physical properties because:


A

They contain different isotopes of carbon

Diamond has a giant molecular structure but graphite has an ionic structure

They contain carbon atoms joined to one another in different ways

Diamond occurs naturally but graphite is a synthetic substance

To which group in the homologous series does the compound below belong:
H

HC

//
//
C

3
OH
A

a;Kanes

akenes

12

akar:oc acids

esters

47

48

48

acid but reacts with


A metal Y has no reaction with dilute hydrochloric or dilute sulphuric
dilute nitric acid. Metal V could be:
calcium
0
iron
C
copper
B
zinc
A
A typical non-metallic element:
A

forms positive ions

is a conductor of electricity

is brittle in solid state

forms a chloride with a very high melting point

80% by mass of
A compound containing only the elements carbon and hydrogen has
carbon. The empirical formula of this compound is:
A

2
CH

3
Cl-i

4
CH

,
C
H
2

[RAM. H=1, C=12)

50

ed that caesium.
Caesium (Cs) is a metallic element similar to sodium. It can be predict
A

has a low melting point

2)
forms a hydroxide of formula Cs(OH

reacts vigorously with water to produce hydrogen

forms an oxide of the formula CsO

Items 51-53 refer to the following:


Given below are the structural formulae of two hydrocarbons.
Compound B
Compound A

C=C/H
H-CC

H-CCC-H

Ii

13

51

Compound B belongs to the homologous series:


A

52

54

alkenes

alcohols

acids

Compound B undergoes this type of reaction with chlorine:


A

53

alkanes

substitution

hydration

reduction

addition

Compound A and B both have isomers. The number of isomers in A and B are:
Isomers in A

Isomers in B

Isomers in A

Isomers in B

The structure of a polymer is shown below:

HI

bCC
H

CH
J
3
n

The monomer from which this polymer is made is:


A

55

56

methane

ethane

ethene

propene

A solution containing lead (II) ions was added to a solution containing iodide ions and a
yellow precipitate was formed. Which of the following represents the equation of the
reaction that occurred?
A

Pb

Pb

21

Pbl
2
Pb1

t
2
Pb

F *

2
Pb

21

PbI
2
PbI

The general formula for an alkene is:


A

C, H,

C H
+
2

,
2
C, H

13

C H
1
,
2
_

57

58

59

60

Oxygen may be prepared in the laboratory by:


A

Heating water

Heating copper (II) carbonate

Passing steam over red iron

Adding manganese (IV) oxide to aqueous hydrogen peroxide

en?
Which of the following methods cannot be used for the production of hydrog
Add magnesium to dilute sulphuric acid
A
B

Add sodium metal to ethanol

Electrolyse dilute sulphuric acid

Add copper to dilute hydrochloric acid

When ions are discharged at the cathode, the process is one of:
oxidation
C
neutralization
B
reduction
A

hydration

when treated
A solid X is insoluble in water. It gives off a gas when heated and also
with dilute sulphuric acid. X could be:
copper (II) carbonate
B
calcium hydroxide
A
C

sodium carbonate

zinc oxide

CXC 0 Level Chemistry


Assessment Practice
Paper 02 Test 4
TIME:

1 hour 30 minutes.

INSTRUCTIONS TO CANDIDATES

Read the following directions carefully.

1. You must use this booklet when responding to the questions. For each
question, write
your answer in the space provided.
2. All working must be shown.
3. There are five questions in this booklet. Answer all five questions.

______

You should NOT spend more than 30 minutes on Question I.


Different volumes of 1 .OM silver nitrate and 1.0 M solution of a metal [Xl chloride were
mixed. After settling, the height of the precipitate was measured. The results are
given in the table below.
3
Volume of 1Dm silver nitrate in cm
chloride
in
metal
[X}
Volume of tOrn
3
cm
Height of precipitate in mm
(a)

10

12

12

10

10

15

20

10

(i) Draw a graph of height of precipitate against volume of I .Om silver nitrate.
3
(ii) From your graph, find the height of the precipitate formed by mixing 3.0cm
solution.
chloride
of metal [X]
nn
Is. 1

(i) Find the two volumes of silver nitrate solution that would give a precipitate
whose height is 12.5mm.
1.

2.

Li. 3
-

U-

(iv) Complete the following:


The maximum height of the precipitate is

mm.

cm of 1.Om metal chloride react with


3
solution of silver nitrate solution.

3 of i.oM
cm

,N

(v) Write the equation for the reaction of metal X chloride and silver nitrate
solution:
(b) Complete the table of results for tests carried out on a substance D which was
dissolved into water and divided into three portions.
(i)
(II)

(iii)

(iv)

TEST
To the first portion aqueous sodium
hydroxide was added
The contents of the tube form (1)
were allowed to stand for a few
minutes

OBSERVATIONS
a green precipitate
was formed

ft

ow

pr

1S

To the second portion dilute


hydrochloric acid was added
followed by barium chloride solution

precipitate
white
formed insoluble in
excess

To the third portion aqueous sodium


hydroxide was added followed by
hydrogen peroxide solution

the green mixture


formed turned brown

CONCLUSIONS

w
(2

zrt.I.A

-4.

nflj

Th

Th

_yc.-S

,
-.

re.t

$4

--P-

z.Ji

..-

-t i_.t..
.-... L J_....

.4..

.L-.-

2-p..

It

4--j
.-.

Ii-

..

Thtit__
L
--4

..L...

--.-L--.

1-,--

.
4-4-;

t_

-4-.

-rc-----:---;

i;L:

4
+

4-

,,i-__i.4-

-4_4_ I

+t r:L:

:4.

-if-

-.

;..:U;LZL

rTrt

-4

:-------

C
4

LZ :

:L1

---

::::

.r1$Tr1

t-.
.-

r-

.--

--.4-.--.

rr

rr-r

.4-

,;,
...

-,

-4.

4
-.-j-

-r-r

1.-lw-

-4-

.L..
4
t
I

I;;tt

ih

.._i._._._4.L
I

:.
_i z..z
Litr

111

ffr

s_-fl-i

Titlt

tt

--

-.

cC

14tII.

.______L_:,

..---

__

:.Jll4L
4
.
ZY
!7,jZu.rCf
.Z!-Lt iLt
htTt .UZ nil all
I
II-

ZtLI ifl

-fl-i

--

1ib t-l1- tf{


r
-h 1+4-.-
r---tLu4maffiLrHif
l
f[ -.--1-4t, tt...._
-+rr r-rr- i-.-r 1
-i ti-t-
-rWI
v-J.

-,

.-

LL

-r

I -

-it--r
J_...___ 1 -r

._....__L. ..L.L.....

O7fl.CtkI

-1 k-j-- -rn--i- -,-4-i--!


1z-nm-tz

-1--

lIlt rTI!fl !zt :


I

.yJ/

tsw

ian

CV\

(c) The red colouhng matter in a preserved fruit was extracted by a solvent. Two
drops of the red solution were placed at the centre of a circle of filter paper and
allowed to dry. Drops of solvent were slowly added to the centre of the filter
paper. After a time, four coloured circles were seen as shown in the diagram
below.

N filter paper
/

red circle
red circle

paper

orange cirde

(i)

What is the name of the process described above?

(ii)

Name a piece of apparatus that can be used to drop the solvent on the filter
paper.

(Ni) How many different dyes are present in the preserved fruit?
An alternative set of apparatus was set up as shown below for this
experiment.
r
beaker
strip of filter paper
position of red spot
solvent
___________

lid

(iv) Label on this diagram the results you would expect if this was set up and left
until the solvent reached the top.

2A

The following statements are taken from a students notebook. Each statement is incorrect
but can be corrected by changing one word. Underline one word that should be changed
and give a correct word in the space provided.

(a)

Terylene is a synthetic polyamide.


Correct word
-J

(b)

The relative molecular mass, Mr of water is seventeen.


Correct word

(c)

The reaction of bromine with an alkene is an example of substitution.


Correct word vt;.J

(d)

In an atom, the particles with a negative charge are protons and the particles with no
charge are neutrons.
Correctword

(e)

Esters are formed when acids react with alkalis.


Correciword

(f)

Oxidation is the g?in. of electrons.


Correct word

(&

Silicon is used in the manufacture of glass.


Correct word

(a)

[7]

The diagram represents the arrangement of the particles in a solid.

tt

4
4t
4
wjo
ir(..

pv+icIt-sa-.oC

0
c
t
c
d

fl.flt

pv.aavt

Ohv3

attt

t4F
-4

(i)
(ii)
(iii)

..

Describe the movement at the particles in the solio


How does this movement alter as the temperature of the solid is increased?
How does this moverneni alter as the solid melts
7
[3]

(iv)

(c)

the same
Samples of the gases carbon dioxide, M,= 44, and hydrogen, M= 2, are at
temperature.
Compare the speeds of the molecules in these two gases
[11
Equal masses of steam and of water contain the same number ci molecules.
Explain why the volume of the steam is much greater than that of the water
[1]

fertiliser
Phosphorus is one of the elements essential for plant growth. The effect of using a
table.
containing phosphorus on the yield of potatoes from a field is given in the

phosphorus added! kg

potato yield! kg
1300
2600
3100
3400 a4
3500
3500

0
10
20
30
35
55

sos)

(a)

Give the names of the two other elements essential for plant growth, which are
commonly present in fertilisers.
[1]
and

(b)

After studying the table, a farmer decided to use 15kg of phosphorus on his field.
Estimate the yield of potatoes the farmer expected.
(I)

(II)

(c)

Use the data in the table to suggest why the farmer decided to use only 15kg.
[3]
:
\.

The phosphorus was added to the field as superphosphate fertiliser. This contains
P0j
Ca(H
.
phosphorus as calcium dihydrogenphosphate (V), 2
Calculate the relative molecular mass, M,, of calcium dihydrogenphosphate (V).
(I)

(ii)

Calculate the mass of calcium dihydrogenphosphate (V) which musl he added


to the field to provide the required 15kg ci phosphorus.

[3]
6

4A

In order to make computer processing easier, a system using


proton (atomic) numbers
rather than names or formulae has been developed. The follow
ing examples illustrate this
code system.

name

formula

code

sodium chloride

NaCl

1117

iron (II) bromide

2r
FeB

26, 35(2)

aluminium oxide

3
0
2
A1

13(2). 8(3)

Use this system to complete the table below.

name
.

formula
.

code

KBr
29, 6, 8(3)

sulphur dioxide

16]
4B

(a)

The dot-and- cross diagram shows all the electrons in a


molecule.

key p

proton

(i)

Name this molecule.

(ii)

What type of bonding is present in this mole


7
cule

,x =

electron

[2]
(b)

Show in a similar manner a molecule of ammorna. NH.

IQ\

Y*4f

Qt

(3
7

(1

(c)

4 In the
H
2
Ammonia and carbon dioxide react to form water and a solid, urea, CON
3 of carbqtl dioxide at r.t.p. are converted to urea
reaction, 72dm
(I)
Write the equation, including the state symbols, for the tormation of urea
{ii)

Calculate the volume of ammonia at rip, which reacted.

(iii) Calculate the mass of urea formed.

[5]
5

(a)

The following terms are used in the description of organic compounds.


alcohol

alkane

alkene

amide

hydrocarbon

polymer

From this list, choose two terms which can be applied to each of the compounds
given below.
(I)

.L:r.i.

methane
and

(ii)

hexene

v.
and

(hi) poly(ethene)
and

.*:

p.dsptmt..r

(lv)nylon
18]

and
(b)

One of the general characteristics of homologous series is that all the members can
be represented by a general ormula.
(I)

9
What is the general formula of the alkene series

(ii)

Give one oilier general characteristic ol homologous series.


:....::

(:...

.....:

...

[2]

CXC 0 Level Chemistry


Assessment Practice
Paper 03 Test 4
TIME:

1 hour

INSTRUCTIONS TO CANDIDATES

Read the Instructions on the answer sheet very carefully.

Candidates MUST answer THREE questions on this paper. They must choose ONE from
Section 1, ONE from Section 2, and ONE from Section 3.
All working MUST be shown for calculations.

Section I
Answer one (1) question from this section.
Instructions for preparing hydrated crystals of magnesium sulphate are given, below.
Add one spatula full ot magnesium carbonate to 50cm
3 of dilute sulphuric acid. When it
has reacted, add further amounts until no more will dissolve. Then
ermixture.
Evaporate the filtrate to about half its volume. Allow the filtrate to cool. Filter off the crystals.
Dry them on filter paper Do not heat the crystals.

(a)

Explain the importance of the five instructions which are underlined

(b)

Because it has a larger surface area, powdered magnesium carbonate reacts more
rapidly than lumps of magnesium carbonate.

[5]

Explain why the larger surface area leads to a greater rate of reaction.

[1]

(c)

Name the gas

(2]

(d)

Name three substances, other than magnesium carbonate, which react with dilute
sulphuric acid to form magnesium sulphate.
(21

(a)

The diagram represents the electrolysis of aqueous copper (II) sulphate using copper
electrodes.

evolved in this reaction and give a test for the gas.

copper

copper

copper(ll) sulphate
(i)

Give the formulae of four ions present in aqueous copper(il) sulphate

(ii)

Explaiii why the concentratioi


electrolysis proceeds.

(iii)

Explain why solid copper (II) suiphate does not conauci an electric current

of the electrolyte does not change as

is

(b) Electrolysis is used for the manufacture of aluminium, chlorine and sodium hydroxide.
For one of these manufacturing processes, give
(i) the electrolyte used.
(ii) the materials used for the electrodes.

151

(iii) a commercial use of the product.

Section II
Answer one (1) question from this section.
3(a) For each of the following reactions.
(I) state the observations you would expect to make,
(ii) name the product(s) of the reaction(s).
(iii) explain the changes which take place.
1. Aqueous sodium hydroxide is added to aqueous iron (II) sulphate and the mixture
is allowed to stand.
2. Chlorine is bubbled into aqueous potassium iodide. The solid product is collected
[6]
and then heated.

(b) Hydrogen can be manufactured using the reversible reaction between methane and
steam. The formation of hydrogen is endothermic.
(i) Write the equation for this reaction.
(ii) Explain why this reaction is best carried out at a high temperature but at a low
pressure.
[4]
(c) Study the following reaction scheme.

Colourless Gas
+

Calcium
Hydroxide
Solution

White precipitate
C

Give the name or formula of:


(i) White solid A
(ii)

Yellow solid B

(Hi) White precipitate C


(iv) Gas D
(v) Write a symbol equation for the reaction of solid B with gas D.

(a) Nitrogen reacts with hydrogen to form ammonia.


31-12
2
N
; Lx H = 92 kJ/mol
3
2NH
In this reaction, some bonds are broken and others are formed.
(i) What is the total number of bonds in two molecules of ammonia?
(U)

Explain why this reaction is exothermic.

[3J

(b) Give the names or the symbols of two memrs of each of the follow
ing.
(i) the halogens;
(ii)

the noble gases;

(iU) the alkali metals:


(iv) the transition elements.

(c) Recent research has found several other allotropes of carbon,


named fullerenes.
One of these allotropes has a relative molecular mass. M of 720
(i)
(H)

Expiain what is meant by an allotrope.


Name the two common allotropes of carbon.

(iii) Calculate the formula of the fullerene with M = 720.

Section III
Answer ONE question from this section.
5. (a) Describe the composition alone type of cement.

131

quarried limestona
(b) Outline the process of the manufacture of cement from
glass.
(c) Show the structural differences between cement and

[6]

nt.
(d) Describe what occurs when water is added to ceme

[41

[4]

ess of cement and what is


(e) What type of reactions are involved in the setting proc
131
?
uced
responsible for the strength of the concrete prod

[4]
ystem.
6. (a) Briefly describe how energy flows in a soil ecos
Outline the nitrogen cycle
(b) One of the nutrient cycles in soil is the nitrogen cycle.
[6]
e.
giving illustrations and chemical formulae where applicabl

131
(c) Describe the importance of the use of fertilizers in soil.
nic manure as a fertilizer
(d) Describe how nutrients are released using orga
14]
compared to commercial inorganic fertilizers.
ed after repeated use of
(e) Show, using equations why soil should be treat
[3]
ammonium fertilizers.

CXC 0 Level Chemistry


Assessment Practice
Paper 01 Test 5
TIME:

1 hour 15 minutes.

INSTRUCTIONS TO CANDIDATES
There are sixty (60) questions in this paper Answer all questions. For each question,
there are four possible answers A, B, C and D. Choose the one you consider correct and
record your choice in soft pencil on the separate answer sheet.
Read the Instructions on the answer sheet very carefully.
INFORMATION FOR CANDIDATES
Each correct answer will score one mark.
answer.

A mark will not be deducted for a wrong

Any rough working should be done in this booklet.

What can be deduced about two gases which have the same molecular mass?
They have equal solubility in water at room temperature.
A
C

They have the same boiling point.


They have the same number of atoms in one molecule.

They have the same rate of diffusion at room temperature and pressure.

The diagram shows a chromatogram of several inks.

black

blue

brown

green

rect

yellow

Which statement is correct?


A
B
C
D
3

Slack ink can be made by mixing green, red and yellow inks.
Brown ink can be made by mixing blue and red inks.
Yellow ink can be used to make brown ink.
Yellow ink may be present in green ink.

in a
In an experiment to find the formula of magnesium oxide, magnesium metal is heated
covered crucible:
magnesium

heat

The otucible is covered with a lid to prevent

A
B
C
U

escaping from the crucible.


magnesium carbonate forming.
magnesium oxide escaping trom the crucible.
v;ater vapour escaping from the crucibie.

Equal volumes of twO liquids that mix


completely but do not react together are
placed in the apparatus shown. The
mixture is heated.
When the thermometer first shows a
steady reading, at which point will there
be the highest proportion of the liquid with
the higher boiling point?

LhemiotueIer-.

mixture

of liquic
heat

The atoms of element X have the electronic configuration 2, 8, 6.


Which statement about element Xis correct?
It forms an ionic compound with sodium
It forms an ion of charge 2+.

It has 6 protons ri the outer shell of an atom.


It only reacts with non-metals.

C
D
6

The table shows information about ions 0 and R

protons

neutrons electrons

16

18

18

17

18

18

What are 0 and A?


positive ions of the same element
A

positive ions of different elements

negative ions of the same element

negative ions of different elements

Which set of properties are those of an ionic comfround?

conductivity
of solid
A

good

good

S
o

poor
poor

conductivity
of mo/ten compound

conductivity
c aqueous so/ut/on
ad

good

toor

coca

) or

The bonding in sulphuric acid can be represented by the structural formula below.

0
HV 0
What is the total number of electrons in the covalent bonds surrounding the sulphur atom?
A
4
*6
CS
0
12
It is now known that carbon has an allotropic torm containing molecules of formula C.
What is the mass of one mole of these molecules?

A
10

11

12

12g

60g

360g

720g

What has a mass equal to that of one mole of water?


3 of water
24dm
A

one mole of steam

one molecule of water

two moles of hydrogen molecules and one mole of oxygen molecules

Which suiphide contains the greatest mass of sulphur in a log sample?

suiphide

formula

mass of one moleig

MS

90

2
FeS

120

MoS

160

PbS

239

The diagram shows the apparatus used


to electrolyse lead (II) bromide using
inert electrodes.

-k

V
N..

04

r
lb.

Why does the bulb only light up when


the lead (II) bromide is melted?
A

Bromine atoms in the lead (II)


oromide are converted to ions
when it is melted.

Electons flow through the lead (II)


bromide when it is melted.

4
D

Iead{tI) bromide

The ions in lead (II) bromde are


free to move only when the soild is melted.
There are no ions in solid lead (II) bromide.
4

13

purified ore by
The diagram represents a method for extracting aluminium from its
electrolysis.
waste gases

molten aluminium

Which substances are X and Y?


Y
x

14

aluminium

aluminium chloride

aluminium

aluminium oxide

carbon

aluminium chloride

carbon

aluminium oxide

The circuit shown was set up.


At first, the ammeter showed that no current was
flowing. A few drops of dilute sulphuric acid were
dissolved in liquid Z and the ammeter then showed
that a current was flowing.

What could liquid Z be?


B

aqueous sodIum chloride


aqueous sodium hydroxide

tetrachioromethane

water

15

The equation below shows an exothermic reaction.


I+H
(g)
2
aq)
MgC
(
2
Mg(s) +2HCI(aq)
Which statement about this exothermic reaction is incorrect?
.

Magnesium chloride is soluble in water.

B
C

Magnesium is above hydrogen in the reactivity series.


One mole cf magnesium produces ce mole of hydrogen cas

The total enerGy of the products is greater than that of the reactants.

liquid Z

16 Which of the following are true of


isotopes?
They have the same number of electrons
ii.

iv
A
C

17

They have the same number of neutrons


They have the same number of protons
They have a different arrangement of the
electrons
and U only
i and iii only
ii and iii only

i, ii and iv only

The rate of the reaction between a given mass of calcium carbonate and 1 mol/dm
3
hydrochloric acid was studied by collecting the released carbon dioxide in a graduated
syringe. The calcium carbonate completely reacted.
The results are shown in the graph.

lotal volumo
of carton

time/mn

How would the results change if the experiment was repeated using the same mass of
calcium carbonate and the same volume of hydrochloric acid but of twice the concentration?
maximum volume of carbon
dioxide collected

time at which the maximum


volume is first reached

3
90cm

less than 4 mm

3
180cm

less than 4 mm

3
180cm

exactly 4 mm

90cm

more than 4 mm

18

19

Which of the following is not an example of oxidation?


A

converting iron(Ill) salts into iron (II) salts

converting magnesium atoms into magnesium ions

dissolving of a copper anode during electrolysis

liberating chlorine from a chloride

Which element burns in air to form an oxide which, when shaken with water, gives a
solution with a p1-I greater than 7?
A

20

carbon

hydrogen

calcium

sulphur

An acid, X, was added to a solution of the nitrate of a metal, Y. A dense white precipitate
was formed.
What are X and Y2

21

acidX

metalY

hydrochloric

calcium

nitric

zinc

sulphuric

aluminium

sulphuric

barium

When aqueous sodium hydroxide is added to a solution, a white precipitate forms which
dissolves when more sodium hydroxide is added
Which ion could be present in the solution?
A

22

23

AP(aq)

Ca
(
2
aq)

Cu

2
(aq)

Na(aq)

A solution was tested with Universal Indicator and with aqueous potassium iodide.
Which results show that the solution contains both lead (II) nitrate and nitric acid?
Universal Indicator

aqueous potassium iodide

pH 3

yellow precipitate

pH 5

no reaction

pH 7

yellow precipa1e

pH 9

no reaction

Which statement about groups in the Perlod!c Table is correct?


A
All groups contain both metals and non-metals.
B
C
D

Atoms of elements in the same group nave the same total numoer of electrons.
In Group reacnvity decreases wrr increasing proton (atomic) lumber.
In Group VH me melting point of the eJe:ents increases with cro; (atomic) number

24

Indium, In, is an element in Group Ill of the Periodic Table.


What is the formula of indium chloride?
C/
In
3
B
Cl
3
ln
A

25

i
3
C
2
ln

3
InC!

The table gives the symbols or formulae of the catalysts used in some industrial processes.
catalyst

substance manufactured
hydrogen (from methane)

Ni

synthetic rubber

3
0
2
A1
3
0
2
Cr

polyesters

3
0
2
Sb

PVC

I
KCI
2
CuC
+

nitric acid (from ammonia)

Pt

Cr

Rh

How many different transition metals are included (as elements or in compounds) in the
above catalysts?
6
D
5
C
84
3
A
26

The diagram shows a blast furnace used to extract iron from iron ore.

hot gases

hot at
molten slag
molten iron

Which substance is added with the iron ore and coke at the top of the furnace?
oxygen
0
hn,estone
C
carbon
B
bauxite
A
27

chloride and
When zinc foil is placed in an aqueous solution containing both magnesium
7
cooper (II) sulphate, which element is displaced
magnesium
0
h;drogen
C
copper
B
chlorine
A

Discs of the same size and shape of three different metals were dropped into dilute
hydrochloric acid.
The diagram shows the results.
tube 1

tube 3

lube 2

SRII

Which metal is likely to have been placed in each tube?


tube 1
tube 2
tube 3
A

iron

copper

magnesium

iron

magnesium

copper

magnesium

copper

iron

magnesium

iron

copper

Two tests were carried out on a colourless liquid X.


X turned anhydrous copper (II) sulphate from white to blue.
X reacted with calcium, evolving a flammable gas.
What could X be?
1

dilute hydrochloric acid


ethanol

water

1 only

1 and2only

land3only

1.2and3

_rnoist iron filings

The apparatus shown was set up with 100cm


3 volume
of air in the tube. The volume of gas in the tube was
measured at intervals for six days.

air
graduated tube

Which graph best represents how the volume of gas


changes with time?
A

waler

I:H
123456

123456

Uniolday,

123c56

Iimeddays

brnoJdays

12345t
mo/dsys

31

32

33

Which fertiliser provides the most nitrogen per mole?


J
Q
(NH
S
2
4
C
B
3
NaNO
NO
4
NH
3
A

35

36

37

P0
3
)
4
(NH

sulphur

Four particles are present in dilute sulphuric acid.


Which particle is present in the lowest concentration?
A

the hydrogen ion, H(aq)

the hydroxide, 0H(aq)

the sulphate ion, SO(aq)

(l)
H
0
the water molecule, 2

Silicon is a common element in most rocks.


Which element is most frequently combined with silicon?
A

34

carbon

chlorine

oxygen

Which of the following is not an industrial use of chlorine?


A

making cooking salt

making domestic bleaches

making hydrochloric acid

making the plastic PVC

Which calcium compound would not increase the pH of acidic soils?


A

calcium carbonate

calcium hydroxide

calcium oxide

calcium sulphate

Useful fractions are obtained by the fractional distillation of petroleum oil.


Which fraction and use are correct?

f:action

use

bitumen

fuel in cars

oetrol (gasoline)

aircraft fuel

paraffin (kerosene)

for making roads

lubricating fraction

for making waxes and pohshes

9
V.iu:h compound has an aju;on reaction with chcr2c
HOH
7
C
C
C,H
S
A
to

CH,CQH

38

39

Which compound reacts with steam to form an alcohol?


H,
4
C
0
butane
A
B

ethanoic acid

O
2
C
3
CH
H

methane

4
CH

propene

5
H
3
C

te combustion ot
The table shows how many moles of products are formed by the comple
four hydrocarbons.
stion?
Which hydrocarbon requires 11 mol of oxygen gas, 2 for this combu
hydrocarbon

carbon dioxide/mo!

water/mol

e
C
H
3
H,
5
C
2

,
2
H
6
C

6
C
1
H
7

40

er?
Which of the following pairs shows the correct monomer for making the polym
polymer
monomer
H
H
H
H
H
H

HCCH

I-I

I-I

I-f

1-f

I-f

3 H
CH

CC

CC----O---C---C-O

C==C

i-f

Cl

I-I

CCCC

HCC-OH

CCCC

Cl

C?

3
3 CH
3 CH
3 OH
CH

CCCC

{
CH
F
3
11

I-f

41

Which pair of the following compounds has the same molecular formula 0
12
H
C
?
6
A
Ethanol and Sucrose
B
Fructose and Glucose
C
Sucrose and Glucose
D
Ethanol and Fructose

42

To which of the following may the term unsaturated be applied?


A
4
H
2
C
B
6
H
3
C
C
10
H
4
C

43

CH.

For an ionic compound, one of the following is UNTRUE:


A
It has a high melting point
B

44

It requires a lot of energy to convert to a liquid


It has high solubility

It is composed of neutral molecules

Which is the best sequence for preparing crystals of a soluble salt starting with an acid
and a metal?
A

Excess acid

Excess metal

acid

Excess metal

acid

Excess acid

metal

metal

_.

filter. evaporate _.decant----.dry crystals


filter, wash residue + dry crystals
filter_evaporate_cooL. decant dry
filter_. wash residue -* redissolve

recrystallize
45.

When the gas carbon dioxide is passed through an aqueous solution of W, a white
precipitate X is formed. This precipitate dissolves when more carbon dioxide is bubbled
through forming a colourless solution of Y. The formulae of these compounds could be

3
CaCO

x
i002
2
Ca(OH)

V
3
Ca00

GaO

2
Ca(OH)

2
)
3
Ca(HCO

3
CaCO

2
Ca(OH)

3
cabo

2
Ga(HGO)

2
)
3
Ca(HCO

12

46

47

When a dilute solution of sulphuric add is electrolysed using platinum electrodes


hydrogen is observed at the cathode. This can be explained by:
A

the oxidation of hydride ions:

the oxidation of hydroxide ions: 20H

the reduction of water molecules:H


0
2

the reduction of hydrogen ions:

49

2
Y
3
X

2
H

2e

2e

2
H

2
H

3
Y
2
X

3
XY

molten aluminium

aqueous copper sulphate

copper metal

4
XY

An oxide of sulphur contains 50% by mass of sulphur. What is the empirical formula of
this oxide?
SO
A
B
503
C
D
504
602
=

32, 0

16]

Which of the following elements requires the smallest number of Faradays for 1 mole of
atoms to be liberated during electolysis?
calcium

magnesium

sodium

aluminium

An isotope of element Y has 17 protons and 18 neutrons in its nucleus. Which is the
correct symbol for the ion of Y?
Y
18
17

0
17

17

1/

Which is the correct order in decreasing activity of the metals in the reactivity series
9
A
calcium, copper, zinc
B copper, zinc, calcium
C

53

graphite

52

2e

A
51

fr

Which one of the following conducts electricity entirely by the movement of ions?

[R.A.M. S
50

2H

2
H

_..

The atomic number of an element X is 6, the atomic number of an element Y is 17. The
probable formula of a compound formed between X and 1 is:
A

48

2K-

zinc, magnesium. copper

magnesium, zinc, copper

A metal X forms a nitrate of formula X(N0


. What is the formula of the sulphate of X?
)
3
A
4
XSO
B
S0
2
X
4
C
3
X(SO,)
0

13

54

Rubidium is in group I of the periodic table. The formula for rubidium sulphate is:
(S0
2
Rb
3
)
4
0
SO
2
Rb
4
C
2
)
4
Rb(S0
B
4
RbSO
A

55

12 was cracked to produce ethene and another gas V which had no


H
5
(C
Pentane )
reaction with aqueous bromine. What is the structural formula of Y?
H

H
I

HCCCH

HCCH

H
0

H
HCC=

HCH

56

Ammonium sulphate is used as a fertilizer to help plants::


produce proteins
B
fight disease
A
C

57

aid photosynthesis

Which of the following occurs in the production of sulphuric acid by the contact process?
A

sulphur dioxide is dissolved in water

sulphur trioxide is dissolved in water

sulphur dioxide is dissolved in dilute sulphuric acid


sulphur trioxide is dissolved in concentrated sulphuric acid

58

produce sugars

Which one of the following structures represents the linkage between units in both
proteins and nylon?
H
H
B
H
0
A
I
I
i
II
CC
C--C
I
I
I
H
H
H
C

0
Ii
C

H
I
N

Dro
II

LC

14

_
59

An element X reacts with the following substances as listed below:


2
2Xi-0
2X0
XO

2
H

X2HCI

no reaction
b

XC

2
H
I

Which of the following could be X?


A
iron
B
magnesium
60

calcium

copper

Oxygen gas was collected in a gas syringe by the catalytic decomposition


of hydrogen
peroxide solution. Which one of the following graphs represents the total volum
e of
oxygen measured at various intervals of time?

:H

15

CXC 0 Level Chemistry


Assessment Practice
Paper 02 Test 5

TIME:

1 hour 30 minutes.

INSTRUCTIONS TO CANDIDATES

s carefully.
Read the following direction
1.

h question.
ding to the questions. For eac
pon
res
en
wh
t
kle
boo
s
thi
use
You must
vided.
write your answer in the space pro

2.

All working must be shown.

3.

ns.
booklet. Answer all five questio
s
thi
in
ns
stio
que
five
are
ere
Th

1.

You should NOT spend more than 30 minutes on


Question 1.
(a)

A student performed an experiment to investigate how solubility of


potassium nitrate
crystals varies with temperature. The procedure carried out is as follows:
(I)

3 of water was added to a boiling tube containing 159 of potassi


10cm
um
nitrate.

(ii)

The contents were warmed in a water bath until the solid dissolved.

(i)

The thermometer was placed in the boiling tube and after a few minute
s. the
boiling tube was removed from the bath and allowed to cool.
The
temperature at which crystals first appeared was recorded.

(iv)

3 of distilled water was added and the above procedures (ii)


2cm
and (iii) were
repeated noting the temperature at which crystals first appeared.

(v)

The procedure was repeated adding a further 2cm


3
obtain a total of seven readings.

portions of water to

Complete the following table of results:


Reading

Volume of

Number

3
Waterl
crn

10

Teniperaturef

79

-_
_

16

Calculation of

Solubility

Solubility per

GmsIlOO

3 water
100cm

3 H
cm
0
2

15x100
10

56
-

18

51

20

48

22

45

00cm of water
nitrate in grams per 1 3
Plot a graph of solubHity of potassium
against temperature in C.
bility of potassium nitrate in grams per
(ii) Use your graph to determine the solu
100cm at 65C.
3

(a) (i)

Solublilty:
eriment.
(Hi) List two sources of error in this exp
1.
2.
the following reaction takes place:
(b) When potassium nitrate is heated,
KNO +02
KNO * 2
3
(i)

Balance the above equation.

(ii)

ed at R.T. P. if 10.1 grams of potassium


Calculate the volume of oxygen form
nitrate are heated.

ied out on solid Z.


(c) Complete the table below for tests carr
TEST

CONCLUSIONS

OBSERVATIONS

The gas
ammonia

To an aqueous solution of
Z sodium hydroxide was
mixture
and
added,
warmed

Pieces of red and blue


litmus were placed at the
top of the lest tube

To a second portion of an
aqueous solution of Z,
dilute nitric acid was added
(allowed by sliver nitrate

A white
formed

A fresh sample of Z was


heated in a dry test tube

Z tumed directly into gas.


Crystals reformed at the top
of the test tube

precipitate

was

evolved

was

(d) The following results were obtained in an experime


nt performed by a student:
HCI(aq) + Na OHcaq
I NaClaq + H2Oaq A H = 57KJ
3
HN
a
q
O

4
S
2
H
a
O.i
q)

Na QH
+

3
NaN
O

2Na OH() +

1
0
2
H

A H = -57KJ

7
N
S
2
+
1
O
2H
O
a A H =-114KJ

(i)

What type of reaction do these equations represent?

(ii)

Explain why Al-I for HCI, and HNO

(iN) Explain why AH for 2


H
S
O

4(aq)

3(aq)

are the same.

is twice that of HCI{aq).

(iv) Write the simplest ionic equation for the reaction


of hydrochloric acid and
sodium hydroxide.

-1 -ti 44-t
rrt

..:

r -r

.4.
.4

-I-

.4-.

.4

II,

4.
.1

.-i I.
.4.-

I
I
1

.1.

4-4..

4.

4-

4-

.4,
4

.4

.444
I,
*

j_.

1
1-

4.

,-:i-.:I.

44-

f.r 1,j

4-

.4

..

.4.

.4

4!

[4

h -ri: 4;.;;
t

-tr

I,J_L

4 i

ff
t
m

4.-.-t

.4.

4.?

.4

-a

4.

..

-i

.:

Hit

-:

d.t..-4--

TI
44 +4_

JI

4.

-14--- -t
-1 4- 1.. .4 .4 .4+4..
1

-.4.4..
+
1-

.4.

I.

44

it,

4 4

444

...

--

11

tl I

4,*
.4
.4..

H-ht
;+.i_4

-.

-,

.4- 4-

.4.4

.4,.

-fH

:tL:Lt:

4. .4-.
4..

Li:
4,,

.4--h tit
.t
4

k-

4.4.

4.

44

:t:tZtt

.4

II

.4.4_a.

.pr

IL

_.4.4.
_

qfr cLr
i4
ui4tLL____
J.Lt tLi
tt

tr n

.4-,

441*,

2A

This question is concerned with the (allowing oxides


aluminium oxide

carbon monoxide

copper(il) oxide

magnesium oxide

nitrogen dioxide

zinc oxide

Each oxide can be used once, more than once, or not at all.
Name an oxide from the list above which
is acidic,
(I)
(1]
is amphoteric,

(ii)

[11
contains an element in the +3 oxidation state,

(iH)

ill
(iv)

causes acid rain,


tt.P-

(e)

[1]

will react with dilute sulphuric acid to give a blue solution.

Ill
28 The diagram represents the structure of a metal.
(a)

Use the structure to explain why metals


conduct electricity.

0000
0i00
4 4N-ct) r 0

[1]
QUfr at)
Give two other physical properties of metals.
1

(b)

2
(c)

[2]

Give one chemical property of metals


[1]

JA

(a)

Complete the loliowing table which gves information about tvio Cins.
of
protons

numLer

ion
F

number of
neutrons

!;.;.;oer of
electrons

nucleon
number

10

24

10

2
Mg

(b)

F
.
2
(Mg
in magnesium fluoride )
Draw adot and cross diagram to show the bonding
Include all the electrons in your diagram.

[2]
38 The following question is concerned with ionic equations.
nic equation. The first one has been
(a) Rewrite each of the following equations as anio
done as an example.
2
MgSO + H
HS0 - 4
Mg + 4
equation
2
2 +H
Mg
ionic equation Mg + 21t
.

(I)

equation

Ca

2HCI

2I
CaC

equation

2
H

ionic equation
(II)

2NaBr

2NaCI
Cl

2
Br
[2]

ionic equation
(b)

ous iron (II) chloride.


The following reaction occurs when chlorine is bubbled into aque
2
2Fe
(I)
(ii)

4A

2Fe

2Ct

ced.
Explain, in terms of electrons, why chlorine has been redu
Name the salt formed in the reaction.
Za:.

[2]

ared by precipitation.
Barium sulphate is a white insoluble salt. It can be prep
precipitate of barium sulphate.
(a) Name two solutions which on mixing will produce a
[2]

(b)

of the precipitated barium


What three steps are required to obtain a pure dry sample
sulphate from the mixture in (a)?
[3]

4B Complete the following sentences.


dif1eren structural formulae are
(a) Compounds with the same molecular formula but
[ii
called
..::

(b)

Q conta:r the same number of


The acns Q and 8
The oxygen atoms O

but ddferent numbers of

[31

Q are called
and 8
7

4C

Use the following to answer the questions.


graphite
iodine
potassium
(a)

potassium bromide

silica

Which is a compound having a macromolecular structure?


)H

(b)

Which has a crystal lattice consisting of simple molecules?

(c)

Which substance conducts electricity when molten but not when solid?
[3]

5A

Saudi Arabian petroleum is a complex mixture of hydrocarbons.


The table below shows the fractions obtained and the amounts produced by fractio
nal
distillation compared with the amounts required.
traction

percentage produced
(supply)

percentage required
(demand)

Liquefied Petroleum Gases (LPG)


Petrol/gasoline
Naphtha
Paraffin/kerosene
Diesel Oil
Heavy oils and bitumen

3
14
9
15
10
49

4
25
3
7
22
39

(a)

S/

Define hydrocarbon
a

.:_..:

(b)

[1]

(I)

For which fraction does the demand exceed the supply by the greatest amount?

(ii)

For which fraction does the supply exceed the demand by the greatest amount?
[2]

(c)

Cracking involves the breakdown of large hydrocarbon molecules into smaller ones.
(i) Give the conditions needed for cracking hydrocarbons.
(ii)

Explain how cracking can match the demand for a fraction with its supply.

(iii) A hydrocarbon, F
12 is cracked to give only two products One of the products
C
is butane
What is the molecular formula of the other product?
[4]
8

5B

Aqueous potassium hydroxide is an alkali.


(a) Name the salt formed when aqueous potassium hydroxide reacts with ethanoic acid.
[11
(b)

Aqueous potassium hydroxide is gradually added to dilute sulphuric acid until it is in


excess.
(I) Write the ionic equation for this reaction.
(Ii)

What happens to the pH value of the mixture during the addition?


(2J

(c)

The temperature of the mixture initially begins to rise as the potassium hydroxide is
added to the sulphuric acid. It then reaches a maximum and eventually falls as more
potassium hydroxide is added.
The graph shows how the temperature of the mixture varies with the volume of
aqueous potassium hydroxide added.
(I)

Why does the temperature initially


rise?
2fi

.11

24

(ii)

What volume of aqueous


potassium hydroxide gives the
highest temperature reading?

72

20

--

tTTtzJ

r7#.v

iz

CI OflVn p,assn IIyOo.Ce ad

(ill) Explain why the temperature taIls ii more than this volume ot aqueous potassiul
hydroxide is added.

[4]

CXC 0 Level Chemistry


Assessment Pracfice
Paper 03 Test 5
TIME:

1 hour

INSTRUCTIONS TO CANDIDATES

Read the Instructions on the answer sheet very


carefully.

Candidates MUST answer THREE questions on


this paper. They must choose ONE from
Section 1, ONE from Section 2, and ONE from Sect
ion 3.
All working MUST be shown for calculations.

Section I
section.
Answer one (1) question from this

on dioxide.
ted they decompose to produce carb
hea
are
als
met
e
som
of
tes
ona
When carb
on into the rate of
show (he results of an investigati
Study the graphs below which
te was heated to
h experiment 0.010 mol of carbona
eac
In
a
ate
bon
of
car
n
itio
pos
om
dec
ry minute.
dioxide produced was measured eve
on
carb
of
e
um
vol
The
e.
atur
per
tem
the same
i_ti Iii

iUlilURlflhitlflInui

100

I,

801
ELI
C,
t

CuCO,

40

C,

20-

,r

0
0

itilili
11111 j_i_l
I

ri i rr,r,rr,rH4f1-44f4

lime! minutes

.
experiment the rate was very slow
Suggest why, at the start of each
the fastest rate?
(ii) Which carbonate decomposed at
onates
phs that the decomposition of the carb
(iii) How could you tell from the gra
was not complete?
used rather
of moles of carbonate had to be
(iv) Explain why the same number
[5]
a fair comparison
than the same mass of carbonate to get
121
calcium carbonate.
of
s
use
ial
ustr
ind
two
te
Sta
(b)
n products of
n they are heated. The decompositio
(c) Nitrates also decompose whe
gas nitrogen
wn
MgO, a bro
Mg(N0 are magnesium oxide.
.
2
)
magnesium nitrate, 3
a glowing splint.
, and a colourless gas which relit
2
dioxide, NO
Name the colourless gas.
(i)
[3]
n
ng slate symbols for the decompositio
(Ii) Construct the equation includi

(a)

(I)

chloride
by the electrolysis ot aQueous sodium
Sodium hydroxide is manufactured
duced
Hydroger and chlorine are also pro
e sodium hydro*:dc
in one of the processes j: mak
The diagram shows the cell useo

c1onne

hydtoii

-j
aqueous

hydra ide

positive
electrode
(anode)

pofous
inontnzto

negative
electrode
(cathode)

(a) Name a source of sodium chloride.


(b) Suggest a reason for the porous membra

[ij
ne

[11
(c) State a use for sodium hydroxide.
[1]
(d) 1 mole of sodium hydroxide, NaCH, and
0.5 mole of chlorine, CL
. are produced
2
from 1 mole of sodium chloride, NaCI.
(i) Calculate the mass of sodium hydr
oxide that can be produced from 117g of
sodium chloride.
(H) Calculate the volume of chlorine,
at room temperature and pressure, that
can be produced at the same time.
[4J
(e) Aqueous sodium hydroxide can also
be electrolysed in the laboratory using inert
electrodes.
(I) State the formulae of the ions presen
t in aqueous sodium hydroxide.
(H) Predict the product fan-ned at the ano
de.
(Hi) Predict the product formed at the cath
ode.

[31

Section II
Answer one (1) question from this sec
tion.
Natural gas is mainly methane, CH
. It is often contaminated with smal
4
l amounts of
hydrogen sulphide, H
5. Normally, the hydrogen suiphide
2
is removed before the
methane is used as a fuel.
(a) (i) Construct the equation to show the
combustion of hydrogen sulphide to form
sulphur dioxide and water.
(ii) A sample of methane was found to be
contaminated with I .70g of hydrogen
suiphide.
Calculate the mass of sulphur dioxide
produced when the contaminated
methane was burnt.
(iH) Suggest why it is important that hyd
rogen sulphide is removed from the
methane before it is used as a fuel.
[5}
3

t.
in the presence of ultra-violet ligh
(b) Chlorine reacts with methane
reaction.
of the substances formed in this
two
of
ula
form
or
e
nam
the
te
Sta
(i)
place?
(ii) What type of reaction has taken
t.
t with methane in uftra-violet ligh
(iii) Gaseous bromine will also reac
chlorine?
Is this reaction faster or slower with
[4]
Explain your answer.
ultra-violet
h chlorine in the presence of
wit
t
reac
to
e
pan
pro
ect
exp
you
(c) Would
light?
[1)
Explain your answer.

tion between
er process by the reversible reac
(a) Ammonia is made in the Hab
nitrogen and hydrogen.
liquid air for use in this process?
(i) How is nitrogen obtained from
er process.
and the conditions used in the Hab
lyst
cata
the
of
e
nam
the
te
Sta
(H)
[4]
3 of water, the temperature
ate dissolves in 50 cm
(b) When 4 gm of ammonium nitr
falls by 6C.
endothermic?
(I) Is the reaction exothermic or
for the water?
(ii) How much energy is absorbed
3 of water = 1 gm
Assume that 1 cm
Specific Heat capacity 4.2 J/gC.
for the reaction:
(iH) Draw an energy level diagram
)
H
O
aq
N
4
(
H + Water = 3
NO
4
)
5
(
3
(RMM of
moles of ammonium nitrate used
(iv) Calculate the number of
ammonium nitrate 80.
[4]
per mole of ammonium nitrate?
(v) What is the heat of solution
cture of an acid.
(c) One use of ethanol is the manufa
al formula.
Name the acid and draw its structur

[2J

Section III
Answer ONE question from this sec
tion.
& (a) What are fibres?
[2]
(b) List the three sources from whi
ch fibres can be obtained and give two
examples
of each type of fibre.
161
(e) Draw partial structures of the
three types of fibres in part (b)
showing the
chemical bonds responsible for ma
intaining the structure of each type
of fibre. [6]
(d) Briefly describe how synthetic fibr
es are produced from a polymer
solution or a
polymer melt.
[3]
(e) Using an example, describe
the benefits of the fibric produc
ed when a natural
fibre is blended with a synthetic
fibre.
[3]

6. (a) Outline three uses of sod


to plants.
[3]
(b) Discuss the advantage that
one plant has in soil containing
humus compared to
another plant in soil containing
no humus.
[4]
(c) Soil sample X needed to be
tested for the presence of am
monium ions, calcium
ions and phosphorous ions. De
scribe, giving equations, the che
mical tests which
needs to be carried out on Sam
ple X.
[6)
(d) Describe the technique of
hydroponics.
[2]
(e) Compare and contrast the
technique of hydroponics, which
is the normai method
of growing plants in soil.
[5]

CXC 0 Level Chemistry


Assessment Practice
Paper 01 Test 6

TIME:

1 hour 15 minutes

INSTRUCTIONS TO CANDIDATES
There are sixty (60) questions in this paper. Answer all questions. For each question,
there are four possible answers A, B, C and D. Choose the one you consider correct and
record your choice in soft pencil on the separate answer sheet.
Read the instructions on the answer sheet very carefully.

INFORMATION FOR CANDIDATES


Each correct answer will score one mark.
answer.

A mark will not be deducted for a wrong

Any rough working should be done in this booklet.

An inflated balloon will shrink if placed in a refrigerator.


This is because the lower temperature causes the gas particles in the balloon to move

faster and become closer together.

B
C

faster and become further apart.


slower and become closer together

slower and become further apart.

The diagram shows the colours of two indicators, methyl orange and metyl red, at different
pH values.
pH

colour of
methyl orange

red

5
yellow

colour of
methyl red

yellow

red

The table shows the pH of four solutions.


solution

pH

In which solution will both indicators be yellow?


A
3

WandX

XandY

YandZ

Zonly

The reaction between aqueous lead (II) nitrate and aqueous potassium iodide can be
represented by the following equation.
aq)
Pb(N0
(
2
)
3
colourless

2Kl(aq)

s)
Pb1
(
2
yellow

colourless

Which method could be used to separate the products?

chromatography

cryslaflisation

distil;ation

filtraUon

2KNO,(aq)
colourless

The diagram shows apparatus used to separate hexane (boiling point, lot) and heptane
(boiling point, 98t).
therrnomeler

/ water out
I

water in

distillate

heat

Which graph would be obtained if the temperature at point T was plotted against the total
volume of distillate collected?

ioo

tolal volume
of distillate

lo(aI volume
of distillate

of distillate

total volume
of distillate

An element Xforms a positive ion with the electronic structure 2. 8. 8.


What is the proton (atomic) number of X?
A

total volume

16

17

18

19

Element V has the electronic structure 2, 8, 2. Element 2 has the electronic structure
2,8,6.
The compound formed between Yand 2 will probably
A

conduct electricity when molten.

B
C

have a low boiling point.


have a macromolecular structureS

not conduct electricity when dissolved n water.

Which solid does not contain covalent bonds?


A
copper
B
diamond
C

02

ice

72Mg

0
2
2H

What is the mass of water formed?


A
9.Og
B
16.Og

10

A mixture of BOg ot hydrogen with 8.Og of oxygen is ignited.


2
2H

graphite

36.Og

The empirical formula of a liquid is C


0.
4
H
2
Which other information about the liquid is needed to work out its molecular formula?
A
boiling point
B

density

relative molecular mass

volume occupied by one mole

A solid deposit of element R is formed at the cathode when an aqueous solution containing
ions of R is electrolysed.
Which statement about element R must be correct?
A
A forms negative ions.
B

11

R ions gain electrons at the cathode.


A ions lose electrons at the cathode.

R is above hydrogen in the reactivity series.

All of the following substances produce carbon dioxide on complete combustion.


Which one will produce 1.0 mol of carbon dioxide?
A
B
C

o
12

2.0 mcI of graphite, C


1.5 mol of propane, 5
F-1
3
C
1.5 mol of ethene. 2
C
4
H
0.5 mol of ethanol, C
OH
5
H
2

Which statement is correct for the electrolysis of concentrated aqueous sodium chloride
using inert electrodes?
A
B

Chlorine is released at the cathode.


Oxygen s re[eased at the cathode.

Sodium

The pH .Df the electrolyte increases

is

re.eased at the cathoce

13

14

Electrical energy is produced by a simple cell as a result of


A

formation of covalent bonds between atoms.

formation of negative ions from atoms.

positive and negative ions coming together.

transfer of electrons from a more reactive to a less reactive eiement.

The thermometers show the temperature at which the reaction started in each beaker.
In which beaker will the reaction be fastest?

{2000L

Ig marble powder
In 200cm
3 moVdm F-IC!
15

ig marble chips
in 200cm
3
2 mol/dm Ho!

ig marble chips
in 200cm
3
3 HO!
2 mol/dm

Ammonia is manufactured by the Haber process, using an iron catalyst.


-i-3H 2NH
2
N
3
It is not possible to obtain 100% yield.
What is the reason for this?

16

lg marble powder
in 200cm
1 mot/dm HC1

A high pressure is used.

Nitrogen is reduced by the hydrogen.

The iron catalyst is used to alter the rate of the reaction.

The reaction is reversible.

In which reaction is the underlined substance reduced?


A

g2H

012
2Fe

Cct 42H

H
2
-C0
0

2r
Br

2Fe+2Ct

2Br+

12

17

18

19

What is the ionic equation for the neutralisation of aqueous sodium hydroxide with dilute
nitric acid?
A

H+OH

Nw

3 3
NwHNO
NaNO
i
-H

NaQH+H

NO;

-,

NaNO

0
2
Nw+H

Which element forms an oxide that reacts with water to give an acidic splution?
A

aluminium

sodium

sulphur

zinc

A certain metallic ion in aqueous solution produces


(I)

a white precipitate with aqueous sodium hydroxide, soluble in an excess;

(ii)

a white precipitate with aqueous ammonia, soluble in an excess.

What is the metallic ion?


A
20

21

2
Ca

IC

2
Zn

What is the best method of making copper (II) sulphate?


A

adding copper to aqueous zinc sulphate

adding dilute sulphuric acid to copper

adding dilute sulphuric acid to copper (II) oxide

adding aqueous sodium sulphate to copper (II) carbonate

Which element shows variable valency, can act as a catalyst and forms coloured
compounds?
A

22

A&

carbon

chlorine

iron

sulphur

Which statement correctly describes the changes in the elements from lett to right across
a period of the Periodic Table?
A

The ability to conduct electricity increases

The elements change from metals to non-metals.

The number ot neutrons in an atom decreases.

The numbe o protons in an atom decreases.

23

Lithium is the first element and rubidium is the fourth element in Group I of the Periodic
Table.
Which statement is correct?
B

Lithium has a higher melting point than rubidium.


Lithium has tewer electrons in its outer shell than has rubidium.

Lithium is a metal, rubidium is a non-metal.

Lithium reacts more rapidly with water than does rubidium.

24

Element U displaces element Vfrom the aqueous nitrate of V


Element Wreacts with cold water to give hydrogen.
Element U gives hydrogen only when heated with steam.
What could elements U, Vand Wbe?

25

calcium

copper

silver

copper

zinc

sodium

sodium

copper

calcium

zinc

copper

calcium

A solid X has the following properties:


(I)

is insoluble in water;

gives off a gas when heated;


(ill) gives off a gas when added to dilute sulphuric acid.
(il)

What could X be?

26

calcium hydroxide

copper (II) carbonate

sodium carbonate

zinc oxide

A1F is used in the extraction of aluminium


3
Na.
,
Molten cryolite, 6

Why is the cryolite used?


It acts as a catalyst.
A
B

It dissolves aluminium oxide.

It increases the purity of the aluminium.

It removes impurities as slag.

27

28

29

Which pair of gases explodes when ignited to form a product which, when mixed with
water, produces a solution of pH 7?
A

hydrogen and chlorine

hydrogen and nitrogen

hydrogen and oxygen

oxygen and carbon monoxide

Which substance present in car exhaust fumes, does not pollute the atmosphere?
A

carbon monoxide

lead compounds

nitrogen

nitrogen oxides

The apparatus was set up as shown.

solid

hydroxide
moist universal
indicator paper

heat

When the test-tube was heated, the indicator paper turned blue.
What could be solid Z?

30

aluminium oxide

ammonium sulphate

calcium hydroxide

copper (II) sulphate

Acid rain contains dilute sulphuric acid and has a pH of 5.


What effect dces limestone (calcium carbonate) have on acid rain?
A

It decreases the pH to 1.

It decreases the pH to 3.

It increases the pH to 7.

It increases the pH to 9.

31

The diagram shows, by percentages, the principal large-scale uses ol chlorine.


waler treatments
making
epoxypropane
5%
23%
making
chloro

7%

other uses

10%
making other
organic

alkanes

11%

making

compounds

20%

12%
1210

making hydrogen
chloride and
domestic bleaches

bleaching woodpulp and paper

What is the total percentage of chlorine used in making organic compounds?


A
32

34%

54%

61%

The diagram shows the structure of a naturally occurring compound.


This compound is used in the manufacture of glass.
Which two elements does it contain?
A

calcium and carbon

carbon and oxygen

chlorine and sodium

oxygen and silicon

/
33

The flow diagram shows how precipitate Y can be obtained from solid X.
colourles
so Id

h0at

solution

73%

What are X and Y?

34

3
CaCO

3
CaCO

3
CaCO

2
Ca(OH)

2
Ca(OH)

2
Ca(OH)

2
Ca(OH)

3
CaCO

What is the general formula of an organic acid?


A

35

2
CH

2
C
1
CH
H
O

O,
21
CH

CH,OH

The structures of three compounds P. 0 and R are shown.


P

R
H

i-f

HC COC H

I
I

I
I

HCC-CO-H
H

I
I

36

37

PandOonly

Pandflonly

OandRonly

P,QandR

Which pair of suostances both contain the


A

nylon

protein

nylon

Terylene

sugars

protein

Terylene poly(ethene)

Ethanol is produced from starch as follows


stage 1
stage 2
starch
simple sugars
ethan.i

Which terms are used 10 describe these two stages?

10

I
I

HCC CH

Which compounds are isomers of each other?


A

OH

Linkage?

38

stage 1

stage 2

cracking

oxidation

fermentation

distillation

hydrolysis

fermentation

C)

polymerization

hydrolysis

The diagram shows how useful products can be obtained by cracking long chain
hydrocarbons.
substance X
used as a
rocket fuel
long chain
hydrocarbons

_catalyst__J

other products

Which two substances could be X and Y?


substance Y
substance X

39

40

ethanol

propene

hydrogen

ethene

methane

ethane

steam

ethene

substance V
used to make
plastics

Which type of reaction occurs between propene and hydrogen?


oxidation
C
dehydration
B
addition
A
The liquid limonene can be extracted from oranges.
It has the molecular structure shown.

a polymer

a protein

an unsaturated hydrocarbon

H
HC H

Which type of compound is limonene?


a carbohydrate
A
B

substitution

H_C

H
HC

I-I

I-I

H:CcCc

11

41

Freezing
with ice

In the apparatus shown in the figure above


1 the thermometer reads -3C If the
experiment is carried out at atmospheric pressure, which one of the following is X most
likely to be?
A

a mixture of water and sand

Pure water

A mixture of ethanol and water

A solution of sodium chloride in water

Items 42 -43 refer to the following data:


The elements below are taken from the period sodium to argon in the Periodic Table
sodium
A
B

aluminium

silicon

chlorine

Select from the list above, an element which fits the following descriptions:
42

Forms a compound with hydrogen of formula XH


4

43

Exists as diatomic molecules

44

Reacts readily with cold water

45

What is the molecular formula of ethanoic acid?


A

0
2
0H

O
4
CH

12

0
3
H
2
C

0
4
H
2
C

46

The reaction between magnesium and hydrochloric acid can best be represented by:
Mg (aq)
Mg(s) + HCI(ao)
A
..

47

2H(aq)

Mg(S)

) 2H(aq)
5
Mg(

)
5
Mg(

cr (aq)

Mg

(Sq)

t
Mg

HCI(aq)

CF(aq)

Hz(g)

Mg Cl2(aqj

)
H
(
2

g)
(H
2

The equation for the burning of hydrogen in oxygen is:


0
2
2H
(g) 02(9)
2
2H
.

This equation indicates that:


A
B
C
D

48

51

strongly acidic

gold, silver, copper

slightly alkaline
strongly alkaline

with
When a nail is placed in the solution of a metal sulphate, the nail becomes coated
be:
could
ted
deposi
metal
the metal and iron ions are produced in solution The
sodium
ID
magnesium
C
B
copper
calcium
A
Which one of the following involves oxidation?
* Mg(s)
B
2 + 2e
Mg
A
C

52

magnesium, aluminium, iron

A solution has a pH of 2. This means that it is:


B
slightly acidic
A
C

50

2 moles of hydrogen combined with 2 moles of oxygen


2 moles of steam can be obtained from 0.5 moles of oxygen

list is
All three metals in one of these lists displace hydrogen from dilute acids. Which
correct?
silver, magnesium, aluminium
B
copper, aluminium, iron
A
C

49

2g of hydrogen combined with ig of oxygen


2 moles of steam can be obtained from 1 mole of oxygen

Br,+2e

s-2Br

211

)
9
(
2
H

2&

2Br-2e

Br,

Which one of the following is MOST likely to form ions with charges of +2 and +3?
Chlorine
ID
Iron
C
Carbon
B
Copper
A

13

During electrolysis, which of the following may occur?


I
The amount of the element discharged at the electrodes is directly
proportional to the quantity of electricity passed through the electrolyte

53

ii
iii
iv
A
54

The conductivity is due to the movement of electrons through the electrolyte


Reduction takes place at the cathode
A discharged ion becomes a neutral atom of an element
i and ii only
B
i and iv only
C
ii and iii only

i, iii and iv only

The following table gives the number of protons, neutrons and electrons in particles W,X,Y,
w

protons

17

17

20

20

neutrons

20

18

20

21

electrons

17

18

18

20

Which one of the following pairs of particles combine with each other to give and ionic
solid?
A
55

56

WX

XV

WZ

XZ

The oxidation state of chromium in Cr


7 is:
0
2
A
+3
B
4
C

+5

-i-S

Which one of the following is/are true?


i
Electrolytes conduct electricity by the movement of positive and negative ions
ii
Metals conduct electricity by the movement of protons
iii
A

Electrolytes conduct electricity by the movement of negative ions only


i only
B
i and ii only
C
ii and iii only
D
i, ii and iii

14

57

Which one of the following is representative of PVC.?


A

CH
II

Hm

rH

I-I

IC
I
LH

L[Jn
C

58

Ljn
U

V-c

3
Cl-i

C5H5n

If ethanol is dehydrated, what type of reaction will occur when the product formed reacts
with hydrogen in the presence of a nickel catalyst?
A

59

CH

Addition

oxidation

substitution

neutralization

Consider the following reversible reaction:


7
I

Which of these statements below is correct? At equilibrium:


A
N is being converted to Z as fast as Z is being converted to N
B
the forward reaction has stopped

60

the concentrations of N and Z must be the same

energy is constantly released by the system

Below is a list of polymers. Which pair consists of similar type linkages within their
structures?
A

Proteins and terylene

Proteins and Nylon

Terylene and Nylon

Polystyrene and Nylon

iS

CXC 0 Level Chemistry


Assessment Practice
Paper 02 Test 6

TIME:

1 hour 30 minutes.

INSTRUCTIONS TO CANDIDATES

Read the following directions carefully.

1.

You must use this booklet when responding to the questions.


write your answer in the space provided.

2.

All working must be shown.

3.

There are five questions in this booklet. Answer all five questions.

For each question,

You should NOT spend more than 30 minutes on Question I


I.

(a)

The table below shows a table of results for the formation of hydrogen in the reaction
of calcium with excess water at 20C.

Time (s)
Volumeof
3
hydrogen cm

0
0

10
6

20
14

I. I. t

30
26

40
41

5060
58
73

70
89

t t t I.
for

4 11790

110
137

120
140

t t I. t
of

calcium

and

130
141

water

Write

(H)

Draw a graph of volume of hydrogen (vertical axis) against time (horizontal


axis) on the graph sheet provided.

(Hi)

Sketch on it the lines you would expect if the water was kept at (a) 3000 (b)
10C.

(iv)

Why is the experiment slower at (a) the beginning, and (b) the end of the
experiment?

equation

reaction

100
131

(i)

an

the

80
102

H
- w
S
dsv.)v%
ts
Draw the apparatus you would use to carry out the experiment.
4

(v)

:y;;:,

._

h-----

I)

2\* ci
I

.r;

t
_.zt

.
(b)

The diagram shows an apparatus used to prepare dry ammonia in the laboratory

19JWL
Fig 1
2

Examine Figure 1 and then answer the following questions below in the spaces
provided.
(I)

Why does the gas jar have its open end downwards?

(ii) What reagents are heated in the flask?

and

(iii) What is the purpose of jar X?


(iv) Suggest the identity of X
(v) What do you see when you mix aqueous solution of barium chloride and sodium
sulphate?
L.

,Oo

2A

For each of the following reactions, describe what you would see and name the reaction
product which is responsible tar the observation.
(a)

Solutions of sodium chloride and silver nitrate are mixed.


observation
name of product

(b)

Solutions of sodium hydroxide and iron (Ill) chloride are mixed.


observation
-

nanieof product
(c)

Zinc is added to dilute sulphuric acid.


observation

ak

,.-

nameof product

I
(d)

Ethene is bubbled into aqueous bromine.


observation
nameof product

3A

(a)

The following terms are used for elements in the PeriodicTable. Name two examples
of each type of element.
alkali metals

(b)

[8]

and

..:

halogens

and

noblegases

and

transition elements

and

4.z

[4]

The element hydrogen has three isotopes. hydrogen, H. deuteriurn 0 and tritium, T.
The nucleon (mass) numbers of the isotopes are one, two and three respectively.

(I)

(II)

Complete the table to show the numbers of particles in the three nuclei.
neutrons
protons
nucleon (mass) number
name
hydrogen

deuterium

tritium

0 is 100.0C. Suggest a
2
0 is 1O1.6t but that of H
2
The bailing point of D
reason for this difference.

[1
(c)

4A (a)

D
0
Calcium reacts with cold water. Construct an equation for the reaction between 2
and calcium.
[2]
equation
Heptane is an alkane with seven carbon atoms per molecule.
It is a colourless liquid with a boiling point of loot.
(I)

Deduce the molecular formula of heptane.


formula

(ii)

Suggest two tests you would use to decide whether a colourless liquid was
heptane or water.
Predict the results of the tests for both heptane and water.
test 1
result with heptane
result with water
test2
result with heptane
15]

result with water


(b)

Fructose is a carbohydrate. One form of fructose has the following structure..


H
00

H0

H-CCCCccH
I
$
H
H
H
0 H
H
5

-n

(i)

Deduce the molecular formula of fructose.


lormula

(ii)

..

Suggest why the term carbohydrate is used for this type of compound.

4...
p
---

(iii) Name another carbohydrate.

name
4B

13]

The table gives some information about alcohols.


(a) Complete the table by naming the other alcohol, calculating the missin
g
predicting the boiling point of propanol.

and

name

relative molecular
mass, M

b.p.
PC

heat evolved on
burning one mole
Ad

methanol

32

65

715

78

1371

ethanol
propanol

60

2010

74

117

2673

[3]
(b)

Complete the equation for the combustion of ethanol.


C
O
5
H
2
H

(c)

[2]

Explain why the heat evolved increases as the M, increases.


[1]

SA

Cerussite is an ore containing lead as lead (II) carbonate, PbCO The


percentage ot lead
in the ore was measured as follows.
A weighed sample of the ore was crushed and then dissolved in
dilute nitric acid.
(a) (I)
Explain why the ore was crushed before adding the nitric acid.

(ii)

acid and describe


Name the gas evolved as the cerussite reacted with the nitric
a test for this gas.
name

[51

test

n obtained from the


An excess of aqueous potassium iodide was added to the solutio
ed, washed and
collect
was
iodide
reaction with nitric acid and the precipitate of lead (II)
dried.
ing the state symbols,
(b) State what you would see and give the ionic equation, includ
for the reaction between the solution and potassium iodide.
observation
[3]

equation
, was weighed. The results were as follows.
2
The Iead(ll) iodide, Pb1
mass of ore sample = 6.60g
mass of lead (II) iodide = 1.47g
(c)

(I)

.
2
Calculate the relative molecular mass, Mr of lead (II) iodide, Pbl

(ii)

Calculate the mass of lead in 1 .47g of lead (II) iodide.

(lii) Calculate the percentage by mass of lead in the ore.

1]

cxc Q Level Chemistry


Assessment Practice
Paper 03 Test 6

TIME:

1 hour

INSTRUCTIONS TO CANDIDATES

Read the Instructions on the answer sheet very carefully.

Candidates MUST answer THREE questions on this paper. They must choose ONE from
Section 1, ONE from Section 2, and ONE from Section 3.
All working MUST be shown for calculations.

Section I
Answer one (1) question from this section.
(a)

(b)

One use of sulphuric acid is in the manufacture of fertilizers.


(i)

name a compound, manufactured using sulphuric acid, which is used as a


fertilizer.

(ii)

Explain why the compound named in (I) will not be the only fertilizer used by a
farmer.
12)

Another use of sulphuric acid is the pickling of metals.


As an example of this process, the steel sheets needed to make car bodies are
soaked in dilute sulphuric acid in order to remove the surface layer of rust.

(c)

(i)

Write the equation for the reaction between the surface layer of rust and dilute
sulphuric acid.

(ii)

Explain why the steel sheets must only be left in contact with the sulphuric acid
for a short period.
[2J

One stage in the manufacture of sulphuric acid is the oxidation of sulphur dioxide to
sulphur trioxide.
For this stage

(d)

(a)

(I)

Write the equation for this reaction.

(ii)

State the essential reaction conditions.

(iii)

Explain why this reaction is described as an oxidation.

[3]

Sulphur dioxide is a reducing agent.


(I)

State a use of sulphur dioxide other than to make sulphur trioxide.

(ii)

Describe a test used to show that a compound is a reducing agent.

[3)

The diagram represents an experiment in which an electric current is being passed


through concentrated hydrochloric acid.

(b)

(I)

Give me names and formulae of all the ions present in this solution. Explain
what happens to each of these ions as it arrives at an electrode. Give ionic
equations for the reactions at the electrodes where appropriate.

(II)

9
Why would steel not be a suitable material for the anode

[61

Hydrogen chloride is a gas. The molecule contains a single covalent bond. When
hydrogen chloride is mixed with ammonia, a white solid is formed.
(I)

What is meant by a single covalent bond?

(II)

Construct a dot and cross diagram to show the outer shell electrons in a
molecule of hydrogen chloride.

Give the equation, including the state symbols, for the reaction between hydrogen
[4]
chloride and ammonia.
Section II
Answer one (1) question from this section,
The table gives some information about three metals.

(Ill)

(a)

(I)

aqueous metal sulphate


formula
colour

metal

order of
reactivity

zinc

colourless

4
ZnSO

nickel

green

4
NiSO

copper

blue

4
0uSD

Given samples of the three metals and of their aqueous sulphates, describe the
experiments by which you could confirm the order of reactivity of these metals.
In your account, clearly state the observations you would look for and explain
how the observations lead to the reactivity order.

(ii)

Give an equation to show the effect of heat on nickel carbonate. Suggest why
nickel carbonate decomposes at a lower temperature than zinc carbonate [8]

(b)

Name an alloy made from copper and zinc. Explain why a bar of the alloy is harder
[2]
and stronger than a bar of copper.

(a)

Hydrogen can be manufactured by the reaction between methane and steam


(I)

Name a source of the methane required for this reaction.

(ii)

Give the equation for this reaction between methane and steam

(iii) Calculate the maximum volume of hydrogen! measured at r.t.p, which can be
obtained from 16g of methane.
[3]
(b)

Margarine is manufactured using the addition reaction between hydrogen and a


vegetable oil.
(i)

State the conditions used for this reaction.

(ii)

What type of bond must be present in the vegeahic


place?

Dii

for this reaction

ae
[2]

(c)

The diagram represents the structure of a common plastic.


H

I
..._.___C
I
H

I
C
I

CL

CL

I
C
I

(I)

Name this plastic and draw the structure & the monomer from which it is
made.

(ii)

This plastic is non-biodegradable. Explain the meaning of this term and


describe the problems which this property causes.
[3]

(d) If this plastic is burned, a thick, black smoke and a very acidic gas are produced.
(i)

Suggest the identity of the black particles in the smoke.

(ii)

Suggest the identity of the very acidic gas.

[2]

Section Ill
Answer ONE question from this section.
5.

(a) List the three components of paints and give the uses of each component.

[6]

(b) Give the formula of two pigments used in paints and state the colour imparted by
each pigment.
[41
(c) Describe the difference between the components of an oil-based paint and a
water-based paint.
[2]
(d) Describe the two different processes which occur when oil-based and waterbased paints are applied to a surface.
[6]

6.

(e) What are the differences in the uses of oil-based and water-based paints?

[2]

(a) Describe, giving examples, how pests are controlled by biological means.

[41

(b) Describe, giving examples, how pests are controlled by chemical means.

[4]

(c) Discuss the advantages and disadvantages of controlling pests by biological vs.
chemical means.
[6]
(d) Descnbe the difference between a herbicide and a pesticide.

[2]

(e) Show how the use of one herbicide can be advantageous to a farmer while the
use of another herbicide may be disadvantageous to the farmer.
[4]

CXC 0 Level Chemistry


Assessment Practice
Paper 01 Test 7

TIME:

1 hour 15 minutes.

INSTRUCTIONS TO CANDIDATES
There are sixty (60) questions in this paper Answer all questions. For each question,
there are four possible answers A, B, C and D. Choose the one you consider correct and
record your choice in soft pencil on the separate answer sheet.

Read the Instructions on the answer sheet very carefully.


INFORMATION FOR CANDIDATES
Each correct answer will score one mark.
answer.

A mark will not be deducted for a wrong

Any rough working should be done in this booklet.

Which test could be used to show that a sample of water is pure?


A

It freezes at exactly 0C.

It turns anhydrous copper (II) sulphate blue.

It turns cobalt (II) chloride paper pink.

When it evaporates, it leaves no residue.

Which diagram shows the arrangement of atoms inside a balloon containing helium?
D
C
B
A

Which graph shows the temperature altering as steam at 110C is cooled to


0
C
B
A

ILI
time

time

time

N
bme

How can the rate of evaporation of water from an open beaker be decreased?
A by blowing air over the beaker
B

by cooling the beaker

by increasing the surface area of the water

by shaking the beaker

Which statement about the noble gases is correct?


A The number of protons in the atoms equals the number of neutrons.
B

Their atoms each have a stable arrangement of electrons.

Their atoms each have eight electrons in their outer shell.

They exist as molecules containing two atoms.

10C?

The number of outer shell electrons for the atoms of the first 12 elements in the
Periodic Table was plotted against the proton (atomic) number of the element.
Which graph was obtained?
0
B
A
-r

I,

4
I

4,

*4 II 4 U

I:

II I I

::K::
I :

1141111

2i4111

DflI4X4

Which substance has the highest melting point?


B lead (II) bromide C
A diamond

*W141)

,KKtQ

methane

ii

74447 I

4141111

p4o1 r.tTt.i

water

Element Z may be represented as


What is the structure of the ion -?
protons

neutrons

electrons

10

10

10

10

10

10

18

The table gives information about the ability of four substances to conduct electricity.
substance

does not conduct under any conditions

conducts only in aqueous solution

conducts when molten and when solid

conducts when molten and aqueous solution

What could these four substances be?

HCL

NaCL.

Pb

HCL

NaCL

Pb

HCL

Pb

NaCI

NaCL

HCI.

Pb

10

Ammonia gas decomposed according to the equation below.


(g)
3
2NH

(g)
2
N

(g)
2
3H

Which volume of hydrogen would be formed if 1 OOcm3 ammonia were decomposed?


(measurements at rip.)
A
11

12

3
25cm

3
75cm

3
100cm

3
150cm

Which compound does not have the empirical formula CH


O?
2
A

ethanol, 2
CH
3
CH
O

glucose, 2
H,
C
6
0

methanal, ECHO

methyl methanoate, 3
CH
2
1-ICO

The equation for the burning of hydrogen in oxygen is shown below.


(g)
2
2H

(g)
2
0

0(g)
2
2H

Which information does this equation give about the reaction?

13

36 g of steam can be obtained from 16g of oxygen.

2 g of hydrogen combine with 1 g of oxygen.

2 mol of steam can be obtained from 1 mol of oxygen.

2 atoms of hydrogen combine with 2 atoms of oxygen.

Two different hydrocarbons each contain the same percentage by mass of hydrogen.
Ii follows that they have the same

14

empirical formula

number of isomers.

relative molecular mass.

structural formula.

In which electrolysis experiment would there be no change in the concentration of the


solution?
electrolyte

electrodes

Aqueous copper (II) sulphate

copper

Aqueous copper (II) sulphate

carbon

Concentrated aqueous sodium chloride

carbon

Dilute sulphuric acid

platinum

15

Which element requires the smallest number of electrons for one mole of atoms to be
liberated during electrolysis?
A

16

17

18

aluminium

calcium

copper

sodium

What are the products when concentrated aqueous lithium chloride is etectrolysed?
at the anode
(positive)

at the cathode
(negative)

chlorine

hydrogen

chlorine

lithium

oxygen

hydrogen

oxygen

lithium

Which change occurs when a photographic film is exposed to light?


A

the formation of silver oxide from silver bromide

the oxidation of silver to a silver salt

the precipitation of a light-sensitive silver salt

the reduction of a silver salt to metallic silver

The dissolving of ammonium nitrate in water is an endothermic change.


Which graph shows how the temperature alters as the ammonium nitrate is added to
water and then the solution is left to stand?
A

tin.

tim.

D
temperature

temperature

t.nporatur.

IanleFatLSe

19

time

In which reaction is the pressure least likely to affect the rate of reaction?
A

C(s)

(g)
2
(g) 0
2
2S0

(g)
2
(g) 3H
2
N

NaOH

(g)
2
C0

2C0(g)

(g)
3
2S0

2NH;(g)

HCI(aq)

NaCI(aq)

+ H
0(I)
2

time

20

Dilute hydrochloric acid was reacted with magnesium ribbon and the volume of hydrogen
gas evolved was measured for the first 80s.

hr

total
volume of

N11 L
0

10

20

33

50

40

60

70

80

time from start of reaclioa!s

What was the average rate of production of hydrogen?


A
21

0,4 cmIs

s
2.5cm
/
3

s
4cm
I
3

s
40cm
/
3

The table gives information about three indicators.

colour at pHI

pH at which
colour changes

colour at pH 12

thymol blue

red

yellow

congo red

blue

red

colourless

10

red

indicator

phenolphthalein

Which colours would be obtained when each indicator was added separately to pure water?

22

23

thymol blue

congo red

phenolphthalein

red

blue

red

yellow

blue

colourless

yellow

blue

ted

yellow

red

colourless

Which of the following reacts with dilute sulphuric acid to give a gas and water as two of
the products?
A

zinc

zinc carbonate

zinc hydroxide

zinc oxide

Which oxide reacts tith an acid to form a saH but does not react with an alkali?
A

aluminium oxide

carbon dioxide

magnesium oxide

zinc oxide
6

24

The reaction represented by the ionic equation


2 + SO
Ba

4
BaSO

takes place readily when


aqueous barium nitrate is added to aqueous sodium sulphate.
A
C

molten barium sulphate is cooled until it solidifies.


solid barium chloride is added to dilute sulphuric acid.

solid barium sulphate is heated until it melts.

25

Which arrangement of electrons is that of a gas normally used to fill light bulbs?
2,8,8
D
2,8,2
2,6
C
2
A
B

26

Using the apparatus shown, chlorine was passed through the tube. After a short time,
coloured substances were seen at P, 0 and R.
potassium bromide

potassium iodide

OR

chlonne

____________

lit

gentle heat

What were these coloured substances?

27

azP

atO

at R

green gas

violet vapour

black solid

green gas

red-brown vapour

violet vapour

red-brown vapour

violet vapour

black solid

violet vapour

red-brown vapour

red-brown vapour

Which oxide can be reduced to the metal using carbon?


A

28

calcium oxide

sodium oxide

magnesium oxide C

zinc oxide

Three different beakers were set up as shown


rod of

rod of

metal Z

----5

observation:

metal W
aisplaced

aqueous
saltof
metalW

jJir

metal W
I

metal X
Js1aced

fl

rod of
metalX

aqueous

salt of
metal X

metal V
metal Y
aispiaced

What is the order of decreasing reactivity of the four metals?


most reactive

29

30

least reactive

Which statement about hydrogen is true?


A

It collects at the anode when dilute sulphuric acid is electrolysed.

It converts alkanes into alkenes.

It is obtained when dilute sulphuric acid reacts with copper.

It is used to make margarine.

An 80cm
3 sample of air is trapped in a syringe. The air is slowly passed over heated iron in
a tube until there is no further decrease in volume.
iron
Ill 111.11 blPIIIjIIIJ

,/lrr_7

3 of air
80cm

heat
When cooled to the original temperature, which volume of gas remains?
3
16cm
0
3
20cm
3
64cm
B
C
3
80cm
A
31

32

water
Which of the following would be true of a solution of chlorine in 9
It is a good oxidizing agent
It has no effect on potassium iodide solution
ii.
It does not form a precipitate with silver nitrate
0 H and iH only
C i and H only
Hi only
B
A. I only

The following scheme shows four stages in the conversion of sulphur to sulphuric acid.
In which stage is a catalyst used?
stage B

stage A

[suipnurj

air

air

concentratec
stage C

stage D
concentrated
suiphunc acj
B

water

sulphuric
acid

LLizJ

33

Carbon and silicon are both in Group IV of the Periodic Table.


Which statement is true for both carbon dioxide and silicon dioxide?

34

They are acidic oxides.

They are readily soluble in water.

They contain ionic bonds.

They have giant molecular structures.

What are different forms of the same element in the same physical state called?

*
35

36

allotropes

isomers

polymers

isotopes

Which statement about alkanes is correct?

They can be polymerised.

They have the same molecular formula.

They undergo addition reactions.

They contain carbon and hydrogen only

The equation shows a molecule of hexane being spilt into two smaller molecules by heating
to a high temperature.
HHHHH
I
I
I
I
HCC--C----CC---CH
I I I
I I

H H HH
I
I
I
HCCCCH
I
I
I
I

substanceX

What is likely to be the structure of substance X?


A
H

B
H

HCCH
H

37

H
C
/\

C
H
H

C=C

H
H-C
H

C=C

I
H

H
H

OH to
5
H
2
When C
OH is oxidised it forms substance Z. This substance reacts with C
5
H
2
produce an ester.
What is substance Z?
A

H
2
HCO

CO
3
CH
H
2

3
CH
C
H
2
H
O

3
CH
C
H
2
H
O

38

A condensation polymer is to be made from the two monomers below.


0

u.

RQCG0H

CH
N
2
NCHC

and

H-

H
Which diagram show the structure of the polymer?

It

II

II

11

II

II

II

Ii

l
CCjCH
-C-N
CHcN
2
C--O

II

II

II

IL

OH

HO

II

II

II

II

cO--cC---OCC-C

2
NH

39

2
NH

Which polymer has the empirical formula OH?


B

A
/HH\

7HH\

/HH

c-c
\\)fl

40

\HCH,Jn

\HC,I-s/fl

7HH

bc-c
\HG.%fl

Nylon (a polyamide) and Terylene (a polyester) are both man-made fibres


Which type of reaction is used to produce both of these polymers?
A

addition

condensation

10

fermentation

hydrolysis

The following are the electronic structures of four elements in the Periodic Table. Which one
represents a non-metal?

41

2.1

2,8,2

2,8,7

2,8,3

42 Which one of the following represents the ions in aqueous copper (II) sulphate?
A

Cue; S0
; H; OR
2
4

Cu; S0
; H; OF-f
2
4

2
4
CLC;S0

Q2-

Cut; S0
; H;
2
4

H;

Q2

43 Which one of the following displaces ion from its aqueous slution?
A

lead

copper

silver

zinc

44 Which one of the following ionic equations involves oxidation?


A

2W+2e

+2e
2
1

b
0

2
H

S
2
2e

2r

Mg+2e

Mg

45 Which type of particle is responsible for the movement of electricity through metals?
A cations
C protons
B anions
0 electrons
46 Which of the following are true of ionic compounds?
i
ii
iv

They are insoluble in organic solvents


They consist of oppositely charged ions

i and ii only

iii

47

I and iii only

ii and iii only

ii and iv only

An element Y forms an oxide of formula Y


. In which group of the Periodic Table is V likely to
3
0
2
be round?
A

48

They are good conductors in molten state or in aqueous solution


They have low melting and boiling points

Groupl

Groupll

GrouplIl

GrouplV

In which one of the following formulae does sulphur show an oxidation state of +4?
I.

F-fSO

ii.

3
NaSO

iii.

2
SO

iv.

i and ii only

i and iv only

ii and iii only

49 A sample of ethane and ethene can be distinguished by the use of:


A

calcium hydroxide

litmus solution

silver nitrate solution

aqueous bromine

11

4
S
2
Na
O
ii and iv only

50

51

A concentrated solution of sodium chloride is electrolyzed using inert electrodes. The gas collected at
the anode:
A

has no effect on blue litmus paper

relights a glowing splint

bleaches damp litmus

13

lights with a pop sound

Which one of the following is formed when ethanol is oxidized using acidified potassium dichromate?
A

52

54

55

COOH
5
H
2
C

COOH
3
CH

COOK
3
CH

Propene reacts with hydrogen in the presence of phosporic acid as a catalyst to form:
A

53

ONa
5
H
2
C

8
H
3
C

COOH
3
CH

OH
7
H
3
C

8
CH

Which of the following is NOT TRUE of soapy detergents?


A

They are biodegradable

They contain phosphate

They do not lather in hard water

They are manufactured from a renewable resource: fats and oils

Which of the following substances reduce iron (Ill) oxide to iron in the blast furnace?
A

carbon dioxide

carbon monoxide

coke

limestone

A sample of ammonium sulphate was warmed gently with aqueous sodium hydroxide. The gas
produced:
A

turned lime water cloudy

ignited with a pop sound

gave a white smoke with hydrogen chloride fumes

relit a glowing splint

56

A soapy solution is obtained by boiling a solution of concentrated sodium hydroxide with


a fat
0
a protein
C
B
a carbohydrate
an acid
A

57

The process by which an alcohol is converted to an acid is:


oxidation
C
reduction
B
fermentajion
A

12

neutralization

58

A certain quantity of electricity liberated 4,Og of calcium from a fused calcium salt. What mass of
sodium would be liberated from a sodium salt by the same quantity of electricity?
[R.A.M. Ca
A

59

60

2.3g

40, Na

23]

1.159

11.59

23g

Which of the following pairs does NOT consist of two compounds of the same homologous series?
A

methanol and ethanol

methanoic acid and ethanoic acid

methyl methanoate and methyl ethanoate

ethene and ethane

Metal A reacts more readily with water than metal B. It would be expected that:
A

A would displace B from a solution of its salt

A would react less readily with 6xygen than B

The oxide of A would be easier to reduce than the oxide of B

A would be less reactive with dilute hydrochloric acid than would B

13

CXC 0 Level Chemistry


Assessment Practice
Paper 02 Test 7

TIME:

1 hour 30 minutes

INSTRUCTIONS TO CANDIDATES

Read the following directions carefully.

1.

You must use this booklet when responding to the questions. For each question write
your answer in the space provided.

2.

All working must be shown.

3.

There are five questions in this booklet. Answer all five questions.

1.

You should NOT spend more than 30 minutes on Question 1.


(a) The following experiment was performed by a student to determine the heat of
combustion of ethanol. The following procedure was carried out and results set
out below
(I)

A spirit lamp was weighted empty and then reweighed with some ethanol.

(H)

200cm of water was placed in a soft drink can and the initial steady
3
temperature recorded.

(Ni) The spirit lamp was used to increase the temperature of the water and this
final temperature was recorded.
(iv) The spirit lamp was removed then reweighed after putting out the flame.
Record of Results:
Mass of empty spirit lamp
Mass of spirit lamp + ethanol
Mass of lamp + ethanol after burning
Mass of ethanol burnt
Initial temperature of water
Final temperature of water
Temperature rise (At)
(i)

=
=
=
=

=
=

22.5g
63.OOg
62.20g
31C
51C
CC

Based on the results above, calculate the heat change AH for the
combustion of ethanol. Assume that the heat capacities and densities of the
solutions are about the same as those of water).
1 C
Cspecific heat capacity of water = 4.2Jg
Density of water: lg = 1cm
3
[Enthalpy change AN = mass of solution x 4.2 x At

(N)

Calculate the number of moles of ethanol burnt.

[R.AM.: C

12,0

16, H

1]

(IN) Hence, calculate the heat given out when 1 mole of ethanol burns in air

(b)

The insoluble salt lead (II) chloride was prepared by the reaction between lead (II)
ions and chloride ions.

(i) Write the ionic equation for this reaction:


(h) What solution could have been used to provide Cr(,?:
:
Pb
?
(hi) What solution have been used to provide 2
(iv)

%AA

j&

What would have been observed on mixing these two solutions?

tia- ctfEy
c4pk4oQai.. coey.

h;:Lkctt

1-

(v)

How could lead chloride have been separated from the mixture?:

(vi)

How could a pure sample of lead (H) chloride have been obtained from the
sample in (V)?:

.
0
Ja

(c)

An experiment was set up as shown in the diagram below to investigate other


reaction between marble chips and hydrochlone acid.
plug
acid
balance

The following results were obtained.

(I)

Write a suitable aim for this investigation:

(ii)

Write a balanced equation for the reaction between marble chips and
hydrochloric acid:

(Hi)

Identify from the data:


(a) One variable that was held constant:
(b) One variable that was changed
(c) One variable that responded:

(iv)

Calculate the number of moles in 25cm


3 hydrochloric acid.
3 of 1 mol/dm

Ca

40. C

12,0

16, H

1, Cl -35.5

2A

This question is concerned with chlorides. Use the following chlorides as the answers.
aluminium chloride

ammonium chloride

copper(ll) chloride

hydrogen chloride

silver chloride

sodium chloride

Each chloride can be used once, more than once, or not at all.
Name a chloride which
(a)

is a diatomic, covalent molecule,


[iJ

(b)

is prepared by precipitation,
[1]

(c)

contains an element in the +3 oxidation state,

Iii
(d)

will react with hot aqueous sodium hydroxide to produce an alkaline gas.
[1]

(e)

dissolves in water to form a green solution.


[1]

2B

Complete the table which shows the names of reagents needed to produce some gases.
reaction
number

reagent 1

reagent 2

gas produced

concentrated sulphuric acid

ethanol

ethene

dilute sulphuric acid

hydrogen
carbon dioxide

[2}
3A

One of the isotopes of an element M has a proton (atomic) number of 20 and a nucleon
(mass) number of 39.
(a)

What is meant by the term isotopes?

[1

Complete the table about the isotope of

(b)

2o

number of protons
number of neutrons
number of electrons
electronic structure

131
(c)

To which Group of the Periodic Table does element M belong?


(11

(d)

M reacts with chlorine to form a white crystalline solid.


What is the formula for the white crystalline solid?
[1]

38

The displayed formulae of three organic compounds are shown below. Name the
homologous series to which each belongs.
H

(a)

H.j
H\
/
0=0

homologous series

1
N-

(b)

HCC----COH

homologous series

(c)
H

homologous series

C
H

3
4A

C C
H

0H

Hydrogen reacts with chlorine to produce hydrogen chloride This reaction is exothermic
During the reaction covalent bonds are broken and new covalent bonds are formed
H
(
2
g)+ C4(g)
(a)

(I)

2HCI(g)

AH

lS4kJ / mel

What type of energy change occurs when bands are broken?


a

(ii)

--

Whjt type of energy charge occurs when bonds ac

formed?

(Ill) Which is the larger of the energy changes taking place in the reaction above,
bond breaking or bond forming?
Explain your answer
[3]
(b)

When 1 mole of chlorine gas reacts the energy change, tH, isi 84kJ.
Calculate the energy change when 1 42g of chlorine reacts completely with hydrogen.

Energy change
(c)

(I)

[3]

In forming ionic bonds electrons are transferred between atoms.


How is a covalent bond formed?

(II)

Draw a dot and cross diagram to show the bonding in a molecule of hydrogen
chloride. You need only draw the electrons in the outer shells.
[21

48 Experiments were carried out to compare the effects of four different catalysts on the
decomposition of hydrogen peroxide.
0
g)
0(l)+
2
2H
aq) (
2H
(
0
2
3 of O.5omolldmD aqueous hydrogen
Each time 0.01 mol of catalyst was added to 10cm
peroxide. The volume of oxygen produced was measured at regular time intervals.
The results were used to draw the graphs below
140

WfjffjE1E:thi J:I 1ff41t


Eflf4fftW&u
JIuLIIItII;IHtLfltL;L[rt

:i,...,.

I20

100

WIIIIfl1IID11$ThWMI l t
I IIIMIIH*llllffiIllJ*JL

a
l5

33

(a)

Draw a labelled diagram of the apparatus that could be used tar these experimentsj2]

(b)

Which was the best catalyst?


[11

(c)

The experiment using iron (Ill) oxide was left until the reaction had stopped.
Predict the final volume of oxygen produced.
cm

(d)

[1]

A fifth experiment was carried out. This time 10cm


3 of 1 .OOmoIfdm
3 aqueous hydrogen
peroxide and 0010 mol of manganese (IV) oxide was used
On the graph, sketch the curve you would expect to obtain and label it p.

SA

[2]

In an experiment to determine the mass of iron in steel, a sample of the steel was
reacted
with excess dilute sulphuric acid. The iron in the steel was converted to iron (II)
sulphate.
(a) Write the equation, including state symbols, for the reaction of iron with dilute
sulphuric
acid.
[2]
(b)

The aqueous iron(II) sulphate produced was titrated with aqueous potass
ium
manganate (VII), KMnO
. The ionic equation for the reaction is written below.
4
MnO, (aq)

8H(aq)

5Fe
(
2
aq)

5F&(aq) Mn
(aq) + 4H
2
O(O
2
In the experiment, 0.020 mol of potassium manganate (VII) ions
was used.
(I)
Calculate the number of moles of iron(lt) ions that reacted.
(ii)

Calculate the mass of iron in the sample.

(Iii) What is the name given tc the type of chemical change that
converts Fe
(aq)
2
into Fe
(aq)?
3
(Iv) The colour of the aqueous potassium manganate (VII)
change during the
reacnon. Describe the colour change.
from
to
(c)

[4]

Give one use of stainless steel.


[1]

P.

5 The atmosphere is a mixture of gases including nitrogen, oxygen, carbon dioxide, water
vapour and the noble gases.
n in clean air?
(a) What are the percentages by volume of oxygen and nitroge
[1]
nitrogen
oxygen
(b)

st how respiration,
The composition of the air does not vary very much. Sugge
of the air fairly constant.
combustion and photosynthesis help to keep the composition
[4j

(c)

The air also contains pollutants. Complete the table.


pollutant

source of pollutant

environmental effect of pollutant

carbon
monoxide

incomplete combustion of
fossil fuels such as petrol
burning fossil fuels such
as coal at a power station

corrosion of buildings

lead compounds

causes brain damage

[1

CXC 0 Level Chemistry


Assessment Practice
Paper 03 Test 7

TIME:

1 hour

INSTRUCTIONS TO CANDIDATES

Read the Instructions on the answer sheet very carefully.

Candidates MUST answer THREE questions on this paper. They must choose ONE from
Section 1, ONE from Section 2, and ONE from Section 3.
All working MUST be shown for calculations.

Section I
Answer one (1) question from this section.
Many millions of drink cans made from aluminium are produced each day. Much research
into the design of these cans has been carried out to minimise the amount of aluminium
used per can, while still maintaining its strength. A typical drink can uses 1 3.5g of aluminium.
(a)

A(
3
0
Aluminium is manufactured from aluminium oxide. 2
Outline the manufactured of aluminium from pure aluminium oxide.
(I)
What is the mass of aluminium oxide required to make a typical aluminium soft
drinks can?
(iii) Suggest why the manufacturers of aluminium cans should minimise the amount
(6]
of aluminium used per can.
(II)

(b)

Aluminium is used in drink cans because it will not corrode easily.


Explain why aluminium will not corrode easily, even though it is high in the reactivity
[2]
series.

(c)

Give one use of aluminium, other than making containers for food or drinks, and
[2]
state one physical property of aluminium that makes it suitable for this use.

2
One of the components in crude oil is undecane, C,,H
(a)

One of the reactions of undecane is shown by the equation below. This reaction
takes place during one of the processes used in an oil refinery.
+

(I)

8
H
4
C

What is the name of this type of reaction?

Suggest two conditions required for this reaction to take place


. gives only 21 2g
74
(iii) When the reaction is carried out, 1.00 mol of undecane. C,H
[5]
heptane
of heptane. .
16 What is the percentage yield of
H
7
C
(ii)

(b)

(i)
(ii)

(c)

Butene, C
. can exist as isomers. Draw the displayed formulae of two of
6
H
4
these isomers.
[3]
Suggest a use of butene, other than as a fuel.

Heptane is a saturated hydrocarbon and butene is an unsaturated hydrocarbon.


Explain how aqueous bromine can be used to distinguish between heptane and
[2]
butene.
Section II
Answer one (1) question from this section.

Water is a very important resource. Water needs to be purified for domestic and industrial
use.
(a) State two processes used to purify river water so it can he used as drinking water
Give ensons for the processes you mention.
(b)

The water quality of drinking water can vary. The table gives information about Inc
concentration of ions in drinking water from lour ditferen ocations.
2

location

A
B
C
D

concentration of ion, in mol per


metre of water

cubicl
pH

2
Ca

Na

C1

SO

N0

4.2
2.7
0.35
0.25

2.4
06
0.3
cL4

20
4A
0.4
0

25

2.0

71

0.1

0.2
0.4

76
8.2
67

0.2

(I)

In which location is the drinking water acidic?

(II)

In which location would the drinking water not give a white precipitate with
acidified barium nitrate?

(lii) Write the formulae of two compounds that give rise to the ions found in the
drinking water at location 0.
(lv)

In which location is the water most likely to be. polluted by fertiliser Explain
your answer
[5]

(c)

Describe one chemical test for water

(a)

A composite material is a mixture of two ditferent substances Reinforcing fibres are


often used in a plastic to give the plastic extra strength. The table below gives some
information about tour different fibres that are used to make composite plastics
fibre carbon

(2]

density in kg/rn
3

force needed to break the fibre in N1m


2

174

31

glass

257

3 7

kevlar

145

3.6

polyester

138

Keviar and carbon fibre composites are used in the building of spacecraft
Suggest reasons why.
(I)

polyester composites,

(Ii)

glass fibre composites,


are not suitable for this purpose

(Ill)

Terylene is a polyester

Draw a section of the structure of this polyester showing clearly the nkages
between the small unils that make up the polymer
(iv) Give the name of a raw material used in making glass
(b)

In Brazil, ethanol is addeo to petrol to form aaschci


fermentation of sugars obtained from sugar cane
(i)
(ii)
(iii)
(iv)

(c)

[5]
The ethanol

is

made by the

How is petrol obtained from crude oil?


Name another fuel obtained from crude oil.
What must be added to a solution of sugar for fermentation
to occur
7
Write an equation to represent the fermentation reaction.

Name

two

other alternatve sources of energy and state


one Important use of each

Section III
Answer ONE question from this section.
5.

(a) Give a definition for an alloy.

[2]

(b) How are alloys made?

[2]

(c) State the names and compositions of two alloys.

[5]

(ci) List the properties of two alloys. Relate these properties to the use of each alloy.
[6]
(e) Describe the advantages that a given alloy has over the components from which
[5]
it is made.

6.

(a) What are fertilizers and what are their major components?

141

(b) Outline a situation showing the need for a fertilizer in soil.

[2]

(c) Describe the differences between slow-release and quick-release fertilizers.


[6]
Give the advantages and/or disadvantages of using each type of fertilizer.
(d) Organic manure and commercial inorganic fertilizers are the two classes of
141
fertilizers available. Give four characteristics of each type of fertilizer.
(e) Describe which of the above two classes of fertilizers [part (d)] you would use if
your farming land is located on a steep hill and in close proximity to a major
[4]
water source.

CXC 0 Level Chemistry


Assessment Practice
Paper 01 Test 8
TIME:

1 hour 15 minutes.

INSTRUCTIONS TO CANDIDATES
There are sixty (60) questions in this paper. Answer all questions.
For each question,
there are four possible answers A, B, C and D. Choose the one you
consider correct and
record your choice in soft pencil on the separate answer sheet.
Read the Instructions on the answer sheet very carefully.
INFORMATION FOR CANDIDATES
Each correct answer will score one mark.
answer.

A mark will not be deducted for a wrong

Any rough working should be done in this booklet.

.7

Bromine has a melting point of 2C and a boiling point of 59C.


At which temperature is bromine a liquid?
D
C
B
A

I.

50

25

25

50

75

100

125

150

temperature/C

A beaker of nitrogen was inverted over a porous pot containing carbon monoxide as shown.
The water level did not change.

beaker
nitrogen

carbon monoxide
porous pot

water
What is the reason for this?

Both gases are diatomic.

B
C

Nitrogen is an unreactive gas.


The gas particles are too large to pass through porous pot.

The two gases have the same density.

The graph gives the melting points of mixtures of lead and tin.
400

400

m.p

04

P4Jt 4&d

300

300

200

200

100

100

melting
point/C

50

0
0%

tEn in the rruxture

100

The graph shows that any mixture of lead and tin must have a melting point
A
above that of tin.
B

below that 01 lead.

below that of both tin and lead.


between that of tin and lead.

0
4

Which property of an ester could be used to check its purity before use as a food flavouring?
A
boiling point
B

colour

smell

solubility in water

The table shows the number of protons, neutrons and electrons in four ions.
For which ion is the data correct?
number of
ion

protons

neutrons

electrons

2
Ca

20

20

20

10

10

12

Na

11

12

10

The atomsP and S have the same


A

nucleon number.

number of electrons.

number of neutrons.

number of protons.

Which statement explains why magnesium oxide has a very high melting point?
A
Magnesium atoms and oxygen atoms are joined by strong double bonds.
B
C

The crystal lattice of magnesium oxide esumbles that of diamonD


The magnesium ions are strongly attracted to the oxide ions.

The reaction between magnesium and oxygen is strongly exotherniic.

Which solid does not contain covalent bonds?


C
B
diamond
A
copper

graphite

ice

Y.
2
The elements Xand Yform the compound X
What is the correct electron arrangement of the atoms X and Y?
electron arrangement:
atom of X

atom dY

2,7

2,2

2,7

2,2

2,6

2,1

2,6

10

What Is the maximum mass of chromium, Cr, that can be extracted from 76g of chromium
?
3
0
2
(Ill) oxide, Cr
152g
0
104g
C
52g
S
A
48g

11

When 7g of iron reacts with 4g of sulphur, 1 ig of iron (II) sulphide is produced.


What will be produced if 7g of iron is reacted with 7g of sulphur?
1 ig of iron (II) sulphide and 3g of unchanged iron
A
B
1 lg of iron (II) sulphide and 3 got unchanged sulphur
C
D

12

13

14

1 ig of iron (II) sulphide only


14g of iron (II) suiphide only

same
1249 of phosphorus vapour has the same volume as 71g of chlorine gas at the
temperature and pressure.
What is the formula of a molecule of phosphorus?
P
0
2
P
C
B
4
P
5
P
A

Which of the following is the best conductor & electricity?


liquid oxygen
A
pure ethanoic acid
B
C

sodium

solid sodium chloride

und
Which technique could be used to obtain the elements X and V from the ionic compo
XY7
filtration
D
electrolysis
C
distillation
B
A crystallisation
4

15

When the experiment shown was set up, the bulb lit, but there were no decomposition
products at the electrodes.
b lb
What is
X?

rH ij

sodium chloride

bromine

molten sodium chloride

0 mercury

L.__..ve

electrodes
16

The formation of hydrogen iodide from hydrogen and iodine is an endothermic reaction.
HHll-H--lH--l
What may be deduced from this information?
A
The number of bonds broken is greater than the number of bonds formed.
B
The formation of H I bonds absorbs energy.
C
The products possess less energy than the reactants.

D
17

15

The total energy change in bond formation is less than that in bond breaking.

Which industrial process uses enzymes as a catalyst?


A
conversion of ethene to ethanol
B

cracking of hydrocarbons

fermentation of sugar solution

synthesis of ammonia

Copper (II) oxide catalyses the decomposition of aqueous hydrogen peroxide to give oxygen
and water.
In a laboratory experiment, aqueous hydrogen peroxide was shaken with copper (II)
oxide in a test-tube.
What did the test-lube contain when the reactIon was complete?
A
a black solid and a colourless liquid
B

a blue liquid only

a light blue solid and a colourless liquid

a pink-brown solid and a colourless hquic

19

When a metal atom becomes an ion, it


A
B
C
U

20

C
D

dilute hydrochloric acid and zinc


dilute sulphuric acid and aqueous sodium carbonate
aqueous silver nitrate and aqueous sodium chloride

Which of the following does not react with dilute sulphuric acid?
A
magnesium hydroxide
C

magnesium metal
magnesium nitrate

magnesium oxide

22

loses electrons and is oxidised.


loses protons and is reduced.

Which two substances react to form a salt and water only?


dilute ethanoic acid and aqueous sodium hydroxide
A
B

21

gains electrons and is reduced.


gains protons and is oxidised.

3 aqueous copper (II) sulphate was mixed with


3 of 1.0 mol/dm
In an experiment, 4.0cm
3 of 1.0 mol/dm3 aqueous sodium carbonate.
8cm
3
SO + CuCD
2
Na
4
CO
2
Na
4+3
CuSO
What did the reaction vessel contain when the reaction was complete?
-.

a colourless solution only


a green precipitate and a blue solution
a green precipitate and a colourless solution

a white precipitate and a blue solution

A
B

23

24

Selenium, Se, is in the same group of the Periodic Table as sulphur.


What is the formula of potassium selenide?
P
SeO
2
K
4
C
4
KSeO
B
Se
2
K
A
Fluorine is the first element in Group VII of the Periodic Table.
Which statement will not be true of fluorine?
A

Fluorine exists in diatomic molecules.

Fluorine forms negative ions.

Fluorine is less reactive than chlorine.

Silver fluoride will be sensitive to light.


6

FCSeO,

25

In the Periodic Table, how many periods include the elements of atomic numbers
8
D
6
C
2
3
S
A

26

Which metal has the least tendency to form positive ions?


magnesium
iron
C
B
calcium
A

27

28

29

sodium

Which reducing agent is used in the production of iron in a blast furnace?


A

air

carbon dioxide

carbon monoxide

limestone

From which reaction is a gas produced?


A

adding calcium to water

B
C

adding dilute hydrochloric acid to silver


adding dilute sulphuric acid to copper

electrolysing aqueous copper (II) sulphate, using copper electrodes

The apparatus is used to show the reaction between zinc and steam.
mineral wool
saturated with water

zinc

Which equation represents the reaction taking place?


2
ZnO-i-l-1

0
2
Zn-i-H

Zn

0
2
2H

2
+H
2
Zn(OH)

0
2
Zn+-4H

)
Q
j-3F-1
Zn(OH
+
2

2Zn 3H
0
2

ZnO 7
+2H
2
Zn(QH)

-18?

30

The scheme shows some reactions of a compound X.


What could the compound X be?
A

aluminium sulphate

calcium carbonate

copper (II) carbonate

zinc carbonate

excess
(aq)
3
HNO
colourless
solution

fNaON(a)

r
31

white precipitate
which dissolves in
excess NaOH(aq)

The steel bodies of cars can be protected from rusting by spraying them with zinc.
Why is zinc used?

32

33

Zinc does not react with acidic exhaust fumes.

Zinc forms a stable compound with iron.

Zinc has a high melting point.

Zinc is higher in the reactivity series than iron.

Which use of a sulphur compound is not correct?

sulphur compound

use

sulphur dioxide

a fuel

sulphur dioxide

bleaching wood pulp

sulphuric acid

making detergents

sulphuric acid

making fertilisers

Which naturally occurring substance is mainly silicon dioxide?


A

34

haematite

limestone

mica

quartz

When calcium carbonate is heated the following reac;on lakes place.

(s)
3
CaCO

CaD(s)

CO(q

Which volume of carbon dioxide at rip, would be produced, if 50g of calcium carbonate,
M= 100, is heated?
A

3
12dm

3
22dm

C
8

24 dm
3

44 drn
2

35

Which pair of compounds are isomers of each other?


HH
H
A

HCCH

and

HCH
H
H

HCCCl
H

36

HR

HCCCl

and

C=C

Cl

I-I
Cl

Cl

HCC--H

and

HCCCl

Cl
and

Cl

Cl

C=C

H
H

The displayed formulae of two compounds are shown.


H

H
0H

0CH

What are the similarities and differences between the two compounds?

37

similarities

differences

molecular formula

reactions

molecular formula

relative molecular masses

structure

molecular formulae

structure

relative molecular masses

O and reacts with sodium hydroxide?


2
Which compound has the empirical formula CH
A

ethanoic acid

ethanol

ethyl etharioate

methanoic acid

38

39

What is the catalyst used in the preparation of ethyl ethanoate from ethanol and ethanoic
acid?
A

concentrated sulphuric acid

manganese (IV) oxide

nickel

yeast

The diagram represents part of a nylon molecule.


0

II

II

II

N 0N

C 0C

N flN
H

What is the simplest repeat unit in the structure of nylon?

II

0NCC

NON---C0C

II

II

H
0

C--0N---C-0N

II

II

CN---0NC---

40

The equation represents the conversion of starch to a simple sugar.


1
H
6
(C
)
5
0
+
0 nH
O
2
This reaction is an example of
condensation.

hydrogenation.

hydrolysis.

polymerisation.

12
H
nC
6
O
simple sugar

starch
A

10

The element Y has an electronic structure, 2,82. The element is therefore in:
Period 1 group I
A
B
Period 3 group II

41

Period 3 group II

Period 1 group II

Items 42 45 refer to the following diagrams which represent structural formulae


of four organic compounds.
-

H
I

HHH

ill

H-C-OH
H

A
42

H-C-C-C-H
Iii
HHH

H
B

H-C--C
H

0H
D

A reversible reaction takes place between B and D. The forward reaction is:
esterification
A
B
polymerization C
hydrolysis
synthesis
D

43 The reverse reaction is called:


A

44

esterification

polymerization

synthesis

hydrolysis

Which of the compounds is used as a monomer in the production of polythene?

45 Which of the compounds is least likely to produce carbon dioxide and water when burnt?
46 The following substances are all members of the same homologous series EXCEPT:
10
H
4
C
A
B
16
H
8
C
C
H:
6
C
5
0
C
.
1
H
6
47 Which of the following list describes the solution formed and the gas evolved when calcium
reacts with cold water?
Solution

Gas

Alkaline

Neutral

Acidic

Neutral

Neutral

Alkaline

Alkaline

Acidic

11

12 neutron?
48 Which of the following contains 10 electrons 11 protons and
I) Ne
C Ne
B Na
Mg
A 24
10
10

11

12

with some solid sodium chloride.


49 A bottle of copper (II) oxide has been contaminated
oxide.
How can the sodium chloride be removed from the copper (II)
mixture and allow it to cool
A Place the mixture in a separating funnel B Heat the
D Add water to the mixture and filter
C Add dilute acid to the mixture and filter
gas on heating?
50 Which one of the following forms a black residue and a brown
B Potassium iodide
A Copper carbonate
C
51

I)

Potassium nitrate

Copper nitrate

tate on mixing?
Which of the following aqueous solutions would form a precipi
Potassium chloride and barium chloride
i
ii
iii
iv
A

Potassium sulphate and barium chloride


Silver nitrate and potassium chloride
Potassium chloride and barium nitrate
B i, ii and iii
i and ii

andHi

i and iv

52 A compound which has a similar structure to that of protein


53 A compound which when heated evolves a brown gas.
to a solution of barium chloride, a
54 When an aqueous solution of the compound is added
white precipitate is formed.
of oxygen atoms?
55 Which one of the following would be true of one mole
en molecules
It contains the same number of atoms as one mole of hydrog
I
ii
iii
iv
A

It has the same mass as one mole of carbon atoms


tant
It contains Z atoms of oxygen where Z is the Avogadros cons
of carbon
It contains twice the number of atoms as there are in 0.5 mol
13 iii and iv only
C i and iv only
B iv only
I only

12

56

Which of the following islare addition polymers?


i Nylon
H Polythene
iii Starch
iv P.V.C.
B i and Hi only

A i only

57

58

c ii and iii only

D ii and iv only

te (VII) is placed at the bottom 01 a


A crystal of the purple compound potassium mangana
change. One may then
beaker of water and the beaker left until there is no further
observe a:
colourless liquid with the purple crystal unchanged
A
purple layer below a colourless layer
B
C

uniform purple solution

a colourless layer below a purple layer

An electrolyte is:
A
C

a covalent substance of low melting point


any compound soluble in water

a highly conducting metal


an ionic substance dissolved in water

ene reacts with hydrogen in


Items 59 60 refer to the following information: Prop
the presence of a nickel catalyst at 200C.
-

59

The product formed in this reaction is:


A

60

4
H
2
C

H
3
C

8
H
4
C

0
C
1
H
4

hydrogen is called:
The type of reaction that occurs between propene and
dehydration
D
addition
C
hydration
B
substitution
A

13

CXC 0 Level Chemistry


Assessment Practice
Paper 02 Test 8
TIME:

1 hour 30 minutes,

INSTRUCTIONS TO CANDIDATES

Read the following instructions carefully.

1.

You must use this booklet when responding to the questions.


write your answer in the space provided.

2.

All working must be shown.

3.

There are five questions in this booklet. Answer all five questions.

For each question.

________

I.

You should NOT spend more than 30 minutes on Question 1.


(a) Hydrogen peroxide solution may be decomposed in the presence of a catalyst to
produce oxygen and water.
(I)

What is a catalyst?

(ii)

Name the catalyst suitable for this reaction.

An experiment was carried out at room temperature to investigate the


decomposition of ml/dm
3 hydrogen peroxide in the presence of the catalyst.
The following results were obtained.
TimeinMinutes
Volume of oxygen cm
3

80

140

185

220

230

240

240

240

(iii) Draw a graph of volume of oxygen (vertical axis) against time horizontal axis
on the graph sheet provided.
(iv) What is the maximum volume of oxygen given off?_____________________
(v) How long did it take for the hydrogen peroxide to decompose?
(vi) Explain why the graph for the first four minutes is a curve and not a straight
line.

(b) (i)

A few cm
3 of potassium dichromate (VI) solution were added to a gas jar
containing sulphur dioxide and the mixture was shaken.
What would have been observed?____________________________________

(ii) An excess of aqueous sodium hydroxide was added to a test tube


containing aqueous iron (II) chloride and allowed to stand for a few minutes.
What would have been the outcome?

C.,

..

,-

..

t;-

4.,

4-

4.

r:::

-t
.

.4..

.-

.t
1
1

-4q

I-

.1,1

4-

:.

..

4.4

4-

.._._

..

.
4

.::::::

.4

...

-.

4-

..4

-t
4

.11

.
1

*I

....4

i....-.

4.

-.
-i-

..

,-,

i_I_I

::::n

4--s
11.14

-1

.:

Lt ..

I-.---*

ILj.
41

-I.

4441.

njt
i
t
4

.. ._t.
4.11
.
a..i
i__f.4.i
I

4-.--

r--

..4...-l.T...
.4.JI

-T

-rihH*r:4-i.2t.

4.

.::;

1t
t

,i

tL
-

_I
4...
.
.1

.:zt: .-_.JL..J.

.4.
.*4__l...
.4.--

4..

.:;:tf4

.:.

.-

..,:..

.r-.--

..

.4

-.

...

4.i

F.-FI
,-fri-

..

--

m:

I.

L_.i

-.

}.j.j_..L._.J

...*_i

i...

..

41

4.

tz -Lfi

.4:4)4.2

.:.

-4-.__

.4...

:tL Ir

._
T
LiZ.
1

E_

4..

.:

.-

--r---i

: .4:+ i-: 4

Ht

-.

I-!-j.

.-...i

4.

.:

4
r- 4-
+
.. 44
I
I,,

-*444_TI

J,,

.4-..

..

...

-----

.i..p
4...,...

4
t

*4

,...

I..

__

__________
______

(iii)

A solid was heated with sodium hydroxide. The gas evolved turned moist red
litmus paper from red to blue. Name the gas produced.
The gas was also allowed to mix with hydrogen chloride.
What would have been observed.

Write an equation for the reaction:


An organic liquid propanone mixes in all proportions with water, 50 cm
3 of the
mixture were fractionally distilled and distillate collected.

(c)

The diagram below show parts of the stem of the thermometer as each
temperature reading was taken.
After Scm

After 10cm

After 15cm

After 20cm

TI
(I)

Read and record the temperatures in the table below and suggest values which
would have been obtained when the total volume of distillate collected was
3 and 30 cm
25cm
.
3
flilume of distillate/cm

flmeraturerc
(ii)

I 10

15

20

25

30

(a)

Which student at experiment 1 and 4 is not using the correct technique for
reading the thermometer.

(b)

State two precautions which must be taken when reading the


thermometer

Section I
Answer one (1) question from this section.
2A Consider the following list of common substances.
air

brass

glass

salt

sugar

water

limestone

Choose from this list a substance which


(a)

is a mixture containing both elements and compounds.

(b)

is a mixture of compounds.

(c)

is a mixture of elements.

(d)

is a compound containing only two elements.

(e)

is a compound which contains carbon.

2B The table gives some information about the noble gases.


name

symbol

proton
number

relative
atomic mass

b.p.
1K

density at rip.
3
in gldm

helium

He

0.167

neon

Ne

20

27

0.833

argon

Ar

18

40

87

Kr

36

84

xenon

Xe

54

131

radon_[

Rn

86

222

(a)

Name the gas with the symbol Kr.

(b)

What is the proton (atomic) number of neon?

121

3.50

165

5.46
9.25

(c) Calculate the density of argon at r.t.p.

(d) Predict the boiling point of radon.

(e) How are these gases obtained commercially?

(f)

Suggest why argon costs less to produce than any of the other noble gases.
[81

3(a) (I)

An alkene has the molecular formula C


. This alkene reacts with bromine to
8
H
4
form a compound with the displayed formula shown below.
(I) Draw the displayed formula of C
.
8
H
4
H H H H

I
I

I
I

HCC-C C-H

(ii)

Br

Sr

Write the equation for the reaction between C


9 and bromine.
H
4

(iii) Give the colours of

this alkene.
bromine
the reaction product
(iv) Explain why this reaction is an example of an addition reaction.
f7J
(ii) The molecular formulae of the first three members of a homologous series
of
hydrocarbons are:
H
6
C
9
H
7
C
H,
0
C
(i) Give the molecular formula of the next member of this series.
(ii)

Suggest a physical property of these hydrocarbons which change


s as the
number of carbon atoms in the molecule changes.
[2)
6

3(b) (I)

,
O was heated in the
5H
2
CuS
.
4
A sample of hydrated copper (II) sulphate, 0
apparatus shown below.
crystals

heat

cold
water

indicator
paper
On gentle heating, the blue crystals changed to a white solid P and
liquid Q was collected in the tube cooled by cold water.
Name the white solid and the neutral liquid.

neutral

white solid P
[2J

neutral llquid Q

(ii) On stronger heating, the white solid P changed to a black solid R and a gas was
evolved which dissolved in liquid Q to form solution S. The indicator papei
turned red as solution S was formed.
Addition of aqueous barium chloride to solution S gave a white precipitate.
(i)

Name the white precipitate.

()

Give the formulae of the two ions present in solution S.


and

(i) Name the black solid R.


[4]

4A

(a)

A gas jar at room temperature contains a mixture of equal volumes of hydrogen,


44,= 2, aridoxygen, M,=32.
(I)

Which molecules, hydrogen or oxygen, are moving faster? Give a reason for
your answer.
fastermolecules
reason

(II)

If the gas jar is warmed to 50C, what happens to the speeds of the molecules?
[3]

(b)

The hydrogen and oxygen in the gas jar in (a) are allowed to react together.
(I) Write the equation for the reaction which takes place.
(II)

Name the substances which wilt be present in the gas jar after the reaction.
and

(c)

[3]

Nitrogen dioxide, NO
, is an atmospheric pollutant.
2
One source of nitrogen dioxide in the air is from the gases from car engines.
Explain how nitrogen dioxide is formed in a car engine and indicate how its emission
can be limited.
formation

limiting its formation


[2]
(d)

Name another atmospheric pollutant.


[1]

5A

The diagram represents a simple cell for the production of electrical energy.
One
electrode is copper and the other is one of aluminium, copper, iron, magnesium,
or
zinc.

copper elrode

metal electrod
electrolyte

(a)

Suggest a suitable solution to use as the electrolyte in this cell.

(b)

Give an equation for the reaction taking place in this cell when zinc is used as the
other electrode.

(c)

Explain how the reading on the meter is linked to the position of the metal in the
reactivity series.

(d)

Complete the table by inserting the names of the metals, aluminium, copper, iron,
magnesium, and zinc.

meter reading
N

metal

2.72
2.00
1.10
0.78
0.00
[8]

CXC 0 Level Chemistry


Assessment Practice
Paper 03 Test 8

TIME:

1 hour

INSTRUCTIONS TO CANDIDATES

Read the Instructions on the answer sheet very carefully.


Candidates MUST answer THREE questions on this paper. They must choose ONE from
Section 1, ONE from Section 2, and ONE from Section 3.
All working MUST be shown for calculations.

Section I
Answer one (1) question from this section.
(a)

An electric current was passed through dilute sulphuric acid. The reactions at the
electrodes are shown below.
at the anode

40H

at the cathode

2H(aq) 2e H
(g)
2

0(l)
2
2H

(g)
2
0

4e

(I)

Explain why the volume of hydrogen formed in this electrolysis is twice the
volume of oxygen formed.

(II)

Give the formula of the other ion present in dilute sulphuric acid.

(b)

Predict the products which will be obtained at the anode (ve), and at the cathode
[2]
(ye), when an electric current is passed through aqueous sodium sulphate.

(c)

Aluminium is manufactured from purified aluminium oxide.

(d)

(I)

Describe the extraction of aluminium.

(II)

Give one use of aluminium.

[4]

Aluminium oxide is an amphoteric oxide.


[1]

Explain what is meant by amphoteric.


2

j3]

(a)

Aqueous sodium chloride can be prepared by titrating aqueous sodium hydroxide


with dilute hydrochloric acid.
The equation for this reaction is shown below.
NaCH(aq)

I-ICI(aq)

NaCI(aq)

0(I)
2
H

(I)

Name the two pieces of apparatus used to measure accurately the volumes of
the solutions in this titration.

(II)

Name a suitable indicator for this titration.


Give the expected colour change of this indicator.

(Ill) Explain the meaning of the symbols (aq) and (I) in this equation.
(b)

(c)

[5]

3 sodium hydroxide was used.


In this titration, 25.0 cm
3 of 1.0 mol/dm
(I)

Calculate the volume of 2.0 mol/dm


3 hydrochloric acid needed to neutraliseihe
alkali.

(II)

Calculate the mass of sodium chloride formed.

[3]

.
2
An element, symbol E, forms an ionic oxide, P0
Predict, giving your reasons,
(I)

whether Eis a metal or a nonrnetal

(II)

in whicn group of the Periodic Table element & is found

[2]

Section II
3

(a)

(b)

Answer one (1) question from this section.


Tin cans are often used as containers for food. In fact a tin can is
made from a steel
alloy with a thin coating of tin on it.
To recover the tin tram used cans, they are crushed and then warmed with
chlorine.
The tin forms tin(tV) chloride, b.p. 1 14C, which distils from the mixtur
e.
On stronger heating, the tin(IV) chloride decomposes into molten tin
and chlorine,
which is re-used.
Suggest why the steel alloy is coated with tin for use in food contain
(I)
ers.
(II) Give the formula of tin(lV) chloride.
(Ill) Is the bonding in this compound ionic or covalent? Give an
explanation for your
answer.
(Iv) Construct the equation for the decomposition of tin(lV) chlorid
e. Explain why
this decomposition is an example of a redox reaction.
(5
Carbon can exist as either of two allotropes.
(I)
Name the two allotropes of carbon.
(II)

Give one use for each of the named allotropes and explain
how this use is
linked to the structure of the allotrope.

[51

4 (a)

Margarine is manufactured by the catalytic hydrogenation


of unsaturated vegetable
oils. The diagram represents a laboratory investigation of this
reaction.The apparatus
was filled with hydrogen before the start of the reaction.

(I)

Explain why the catalyst is used as a fine powder,


rather than as lumps.
(II) What observations would enable you to measure
the rate oi the reaction?
(Ill) Giving a reason for your answer, suggest
why nickel is more likely to be an
effective catalyst than magnesium.
[4)
(b)

(c)

A O.50g sample of a vegetable oil reacted with 60cm


3 of hydrogen, measured at r.t.p.
Assuming that one molecule of the oil contains only
one double bond, calculateS
therelative molecular mass, M,. of the oil.
[3)
A vegetable oil is an ester which can be hydrolysed by boiling
with an alkali.
(I)
Name the hydrolysis products obtained from vegetable
oils.
(II) Name a synthetic polymer which is also an ester
(3]
3

Section III
Answer ONE question from this section.
5.

(a) Give a definition of a dye.

[2]

(b) Give four desirable properties of an ideal dye.

[4]

(c) Describe the four categories by which dies can be classified.

[4J

(d) Illustrate the differences in the structures of natural animal fibres and natural
[4]
vegetable fibres.
(e) Describe why natural animal fibres can be dyed directly by natural vegetable
[61
fibres cannot be dyed directly.

6.

(a) Lime is frequently used by farmers. Why is lime added to the sail?
(b) Lime is mainly impure calcium carbonate.
reaction of calcium carbonate with acid.

Write an ionic equation for the

(c) What do you expect to happen to the pH of the soil when lime is added to the
soil?
(d) List three effects on plants when soils are too acidic.
(e) The garden shop attendant advises a farmer not to add lime and the ammonium
fertilizer he purchased at the same time. Why should this precaution be taken?
(f)

Write a chemical equation for the reaction taking place in (e) above.

CXC 0 Level Chemistry


Assessment Practice
Paper 01 Test 9

TIME:

1 hour 15 minutes.

INSTRUCTIONS TO CANDIDATES
There are sixty (60) questions in this paper. Answer all questions. For each question,
there are four possible answers A, B, C and D. Choose the one you consider correct and
record your choice in soft pencil on the separate answer sheet.

Read the instructions on the answer sheet ve,y carefully.

INFORMATION FOR CANDIDATES


A mark will not be deducted for a wrong

Each correct answer will score one mark.


answer.

Any rough working should be done in this booklet.

Which of the following would diffuse most rapidly?


A
B
2
CO
0/2
C
CH
4

2
N

The reaction scheme shows how hydrated capper (II) sulphate, 2


.5H
4
CuSO
0
, changes
when heated.
90C
2
.
4
CuSQ
0
5H

250C
2
.
4
CuSO
0
5H

4
CuSO

A little water was accidentally spilled into a dish containing hydrated copper (II) sulphate.
What could be done to remove the water, leaving pure, dry 0
2
.
4
CuSO
5H
?
Heat the dish over a boiling water-bath.
A

Heat the dish to constant mass.

Heat the dish with a Bunsen burner.

Let the dish stand in direct sunlight.

Compound X melts at got, boils at 120t and is not soluble in water


Which apparatus can be used to obtain pure X from a mixture of X with water?
A

jr
4

The table shows information about particles X and Y.


number of
DQ!Qn5

number of
neutrons

electronic
structure

10

Z8

20

2,8,8

x
Y

17

v:H!crI statement is correc: cr both X andY?


2

They are atoms of metals.

They are atoms of noble gases.

They are isotopes of the same element.

They are negative ions.

In which set do the three particles each have the same total number of electrons?
A

Ct

Br

F-

Ne

Na

2
Ca

Br

Li

Nr

Which substance in the table could be ethanol?


ability to conduct electricity

substance

melting
point/C

boiling
point/C

when liquid

114

85

none

114

78

none

none

180

218

none

insoluble

808

1465

good

good

Which of the following is a compound?


carbon
B
air
A

-in aqueous solution

oxygen

good

steam

Electrolysis of molten lead (II) bromide gives lead at the cathode.


Why does the lead form?

Lead (II) bromide decomposes on heating.

Lead has a low melting point.

Negative lead ions are discharged at the cathode


Positive lead ions are discharged at the cathode

0
9

In which oxide does X have the same oxidatioon sta:e as in the chloride, XC;:
2
X0
0
0
2
X
x
a
2
C
A
0
3
X
B

10

1-1 at r.t.p. has a mass of 20g.


2
C
,
A volume of ethane, 5
What is the mass of an equal volume of propene, C
, at r.t.p.2
5
H
3
20g
A
C
28g
B
21g
D
3

429

11

Rubidium, Rb, is in the same group of the Periodic Table as sodium.


Which products are obtained from the electrolysis of concentrated aqueous rubidium
chloride?

12

cathode product

solution produced

hydrogen

acidic

hydrogen

alkaline

rubidium

acidic

rubidium

alkaline

Aqueous copper (II) sulphate is electrolysed using copper electrodes.


Which observations will be made?

13

14

at anode ( ye)

at cathode

anode dissolves

pink solid forms

blue colour fades

anode dissolves

no change

colourless gas forms

pink solid forms


colourless gas forms

no change

colourless gas forms

pink solid forms

blue colour fades

the electrolyte

What is the equation for the change that takes place at the cathode (negative electrode)
during the electrolysis of molten magnesium chloride?
A

2
Mg-2e
-Mg

2
M
2w-M
g
g

Mg-t-e--Mg

Mg

2
2e- Mg

Which process is exothermic?


burning petrol in a car engine
A
B

cracking of oil fractions

tractional distillation of oil


melting bitumen for roads

o
15

( ye)

Which indstrial process uses iron as a catalyst?


making ammonia from nitrogen and hydrogen
A

o
C

makwg ethanol from ethene and steam


making steel

making sulphur trioxide from suiphur dioxide and oxygen

16

The rate of the reaction between a given mass of calcium carbonate and an excess of
hydrochloric acid was studied by collecting the released carbon dioxide in a graduated
syringe.
The results are shown in the graph.

P IfEHIHU !I

1tIII1T1IiI PI1I I 11U


1
tot9l VOUfl1I
of caiton
dioxide!cm
3
40

L.

tim&min

How much time was required for halt the calcium carbonate to react?
A
17

1.5 mm

2.0 mm

3.0 mm

10N
3
N
2

In which of the following changes is the nitrogen reduced?


A

18

0.95 mm

toNQ
3
NH

toNO
NH
3

I0NH
2
N
3

When gas X is passed over heated copper (II) oxide the products are copper and steam
only.
What is gas X?
A

19

ammonia

carbon monoxide C

hydrogen

methane

Acidified potassium dichromate (VI) can be used to detect the presence of ethanol vapour
in the breath of a person who has consumed alcohol.

7 -f.-...

filter paper moistened with


3cidified potassium dichromate(vi)

A colour change from orange to green is observed if ethanol is present.


This shows that ethanol is

20

a catalyst.

an alkalL

an oxidising agent.

a reducing agent.

A white compound produces a mixture of gases when heated This mixture turns moist
Universal Indicator paper red and relights a glowing splint.
What does this mixture contain?

21

an acidic gas and hydrogen

an acidic gas and oxygen

an alkaline gas and hydrogen

an alkaline gas and oxygen

When two liquids are mixed, a solution with a pH value 017 is formed.
Which of the following are the pH values of the two liquids?

22

23

first liquid
pH

second liquid
pH

12

14

Which of the following describes a step in the preparation of insoluble barium sulphate
from aqueous barium chloride and dilute sulphuric acid?
A

Add dilute sulphuric acid until no more gas is produced.

Add the indcator methyl orange.

Collect the precipitate of barium sulphate by filtration.

Evaporate the filtrate until it crysta!ses.

Which pair of elements will combine to lorm an ionic compound


2
A

carbon anc chlorine

B
C

fluorine and sodium


hydrogen arid oxygen

oxygen and carbon


6

24

Which properly decides the order of the elements in the Periodic Table?
A
B
C
D

25

the masses of their atoms


the number of electrons in the outer shell
the number of neutrons in the nucleus
the number of protons in the nucleus

The proton (atomic) number of indium, In, is 49.


What is the most likely formula for the oxide of indium?
A

26

0
2
1n

3
0
2
1n

28

2
mO

From knowledge of the manufacture of aluminium and of iron, what is the order of chemical
reactivity of aluminium, carbon and iron towards oxygen?
least reactive

most reactive

27

mO

aluminium

carbon

iron

aluminium

iron

carbon

carbon

aluminium

iron

carbon

iron

aluminium

AIF used in the extraction of aluminium from aluminium oxide?


3
Na
,
Why is cryolite, 6
A

to dissolve aluminium oxide

to prevent the anodes from burning away

to prevent the oxidation of the aluminium

to remove impurities from the aluminium oxide

Copper (II) carbonate, calcium carbonate and zinc carbonate decompose when heated.
The temperature at which decomposition takes place depends upon the position of the
metal in the reactivity series.
What is the correct order for their decomposition?
highest temperature

lowest temperature

copper (H) carbonate

zinc carbonate

calcum carbonate

calcium carbonate

zinc carbonate

copper (II) carbonate

zinc carbonate

copper(fl) carbonate

caIcir carbonate

zinc carbznate

calcium carbonate

ccpe: ) carbonate

29

Deuterium, D, is an isotope of hydrogen.


Which formula is incorrect for a deuterium compound?
A
4
CD
B
OD
5
D
2
C
C
CaOD

30

13

DCI

Experiments are set up to investigate the sacrificial protection of iron.

iron
nail

water

iron
nail

water

copper

water

Iron
nail
zinc

magnesium

In which test-tubes will the iron rust?


A
31

Xonly

Yonly

The fertiliser ammonium nitrate 3


NQ M,
4
(NH
,
(Mr = 17) by a two-stage process:
stage 1
stage 2

3
NH

XandZonly

13

YandZonly

80) is manufactured from ammonia

3+H
2O2-* HNO
3 + NH
HNO
3
3
N0
4
N
+

What is the madmum mass of fertiliser that can be made if only 17 tonnes of ammonia is
available?
A
32

33

34 tonnes

40 tonnes

80 tonnes

13 97 tonnes

Which reaction does not involve neutralisation?


A

S
2
H
(
4
aq)
0

(g)
3
2NH

S
2
H
(
4
aq)
0

(aq)
2
BaCI

S
2
H
(
4
aq)
0

CuC(s)

13

(aq)
4
HSO

C
2
Na
(
3
aq)
O

S
2
)
4
(NH
(
aq)
0

(s)
4
BaSO

(aq)
4
CuSO

2HC1(aq)

0(I)
2
H

S
2
Na
(
4
aq)
O

(g)
2
C0

0(!)
2
H

The formula of china day (aluminium silicate) was shown in an old book as
A1 25i0
.
3
0
2
. 2H
2
0.
2
This formula is shown in a modem book as Ab(OH)Si
O.
2

What are the values of x and yin the modern formula?

34

35

Which of the following pairs of substances are both manufactured using calcium carbonate?
A

aluminium and iron

ammonia and cement

ammonia and glass

cement and iron

To reduced atmospheric pollution, the waste gases from a coal-burning power station are
passed through powdered calcium carbonate.
Which waste gas will not be removed by the powdered calcium carbonate?

36

37

carbon monoxide, CO

nitrogen dioxide, NO
2

phosphorus (V) oxide, P


5
0
2

sulphur dioxide, SO
2

Which of the following is not made from crude oil?


A

aircraft fuel

diesel oil

furniture polish

margarine

Which of the following describes the ways in which isomers are identical and different?

identical

different

chemical properties

molecular formulae

molecular formulae

structures

physical properties

chemical properties

structure

molecular o:mulae

38

Which structure shows a compound that reacts with ethanol to give a sweet-smelling liquid?
A
H

C
H

1111/
C=C
Hcc-cc
/
\
\
II
H
H

I/I

II

HCc

0H

0
H

III

Which of the folLowing is an ester?


A

40

HocCCC--H

?1

39

PVC

soap

starch

Terylene

Starch can be broken down into smaller molecules by heating with a dilute acid.
What is this type of reaction called?
A

cracking

fermentation

10

hydrolysis

D reduction

.4

45

22

44

27

50

20

Cs

Fr

C....r

64

34

Ri

224

IS...,

51

257

La

Ky

.ICwrr rr

b. v14(n SIcrI rilfitfi

1.100 rYS

72

HI

90

73

U
0wrr

PI51flCr

29

30

77

Np

564,

Pm

93

4*Otfl%t

SI

94

L.IWS

Pu

S.,a,o.r,,
07

Sm

50

Pd

Pt

Oe...n

T.,.

Gd
C...
54

Eu

64

Dy

ISa

SI

T%S-.r

Th
(rgo.r

47

64

Cr..,

44

Cl
C..gs

Bk
aM

Cm
Am
I4tSjt

CI

64

III4I

Hg

201

Ti

064

44

64

Fm
I3

No
.a4

110

Md
CI

M.a b

10

Is

ta.t.
71

7,.
70
C...

Na.w.
OP

75

Lu

73

Yb

105

Tm

07

Er

Sr,..,

05

.54

Rn
.

At
4fl

Po
c._,-

Di

Es

frIe..

Ho

43

ew

a#.14r

54

III

k454

Xe

24

bsrn

127

24

1.

179

kIrtr

52

34

54

Kr

64

Br

79

S.

IS

Ar

C san.,
00

47

52

L4

Pb

*,ur,

I.
cn.,.

07

74

Au

07

S4.4

Sb

51*

Sn

115

In

Il?

a,..

I?

33

Cd

53

4er_

Ag

fl

te,.I,,.

31

G.f I

Cso

In

P..,

75

24

75

As

73

Ge
Ga

64

Zn

54

Cu

55

Ni

70

ct_I

IS

IS

I3

255

31
24

Si

17

Al

3 at room temperature and pressure (r.t.p.)


The vohjme of one mole of any gas is 24 dm

01

735

Pa

222

Tb

r,,It

Nd

Pr
rs,,.r.

Ce

I.

70

Ir
nn

Os
0.I.

ft.

Re

Cr..r

92

75

,SAr.,,.

44

l4.r

344

74

W
.wl

141

540

Ta

.,,nr

44.,

42

r,s....r

t.r,.r..
.....

r5_,

I1..o.rs

4.t.a.,.

__.z

103

Rh

101

Ru

Tc

44

Mo

43

Nb

Zr

54

C.4.4

64

54s

Co

Sr

75

4n.

54

Fe

55

Mn

64

a,.
23

r.....
On,,e,
14

52

Cr

5?

Ti

Rb

S.ssw,
15

Sc

58-71 Lanthanoid series


t90-103 Actinod series

57

55

C.al

20

4....

II

40

Ca

Mg

50

an
4

4_.

!__.

24

II

Na

--_-__

50

Ne

F
0

Be

54

II

54

ImINIv[hI0
II

ii

Group

DATA SHEET
The Perio6c TabI. of the Elecn.nts

41

Which one of the following provides the best evidence for the particle theory of matter?
A

42

Crystallization

Diffusion

Chromatography

Dehydration

When gases are heated their particles:


A

do not move

move more in one direction

move more rapidly

melt

43

Which of the following metals reacts with water to give a solution which forms a white precipitate with
carbon dioxide.
copper
U
magnesium
C
B
sodium
calcium
A

44

3 is:
The oxidation state of nitrogen in N0
A

45

Copper (H) sulphate crystals can be separated from sand using the four processes listed below. The
order in which these processes should be used is:
4th

3rd

2nd

1st
A

filtration

solution

crystallization

evaporation

solution

evaporation

filtration

crystallization

filtration

solution

evaporation

crystallization

solution

filtration

evaporation

crystallization

46 Which one of the following elements burns in air to give a compound which in water gives a solution of
pHi
hydrogen
D
sulphur
sodium
C
B
aluminium
A
47

48

Which of the following is true of all acids in aqueous solution.


A

They liberate carbon dioxide from carbonates

They give hydrogen with any metal

They have a pH greater than 7

They turn red litmus paper blue

Which of the following is least affected chemically when heated in air.

49

Fe

Mg

Cu

Na

A mixture of powdered charcoa and potassium chloride can best be separated by


12

50

51

52

53

chromatography

subhmation

Filtration after shaking with water

Fractional distillation

Which one of the following represents an acid with a basicity of 2:


A
COOH
3
CH
B
H
2
HCO
C
S0
2
H
4

3
HNO

ii and iii

Which two of the following acid salts?


i

Hydrochloric acid

Methanoic acid

Hi

Sulphuric acid

iv

Carbonic acid

iand ii

Hiandiv

i and iv

Which of the following would be true of both fats and polysaccharides?


i
They are produced by condensation polymerization
ii

They can be hydrolysed

lB

They have the same elements

ionly

i,HandiH

iHonly

iandiionly

What are the products given off during the electrolysis of molten magnesium bromide between carbon
electrodes?
Anode

Cathode

magnesium

bromine

oxygen

magnesium

bromine

magnesium

oxygen

hydrogen

54

Moist ammonia gas may be dried by passing it through:


A
calcium chloride
B calcium carbonate
C
calcium oxide
D concentrated sulphuric acid

55

When ethanol is converted to ethanoic acid, the process is one of:


hydration
A
B
dehydration
C reduction
0

oxidation

The number of pairs of shared elections in a methane molecule is:


A
1
B
2
C 3
0

56

57

Which one of the following best describes the reaction of butene with bromine:
A
unsaturated
B
addition
C saturated
0
substitution
13

58

59

at red heat. Which one of the


A metal Z reacts very slowly with water but decomposes steam
following could be Z?
Magnesium
D
Potassium
C
Lead
B
Silver
A
Which one of the following is true of an exothermic reaction?
Heat is absorbed from the surroundings
A
B
C
D

60

Heat is given out to the surroundings


The products have more energy than the reactants
The products have the same energy as the reactants

?
Which one of the following produce an amphoteric oxide on heating
D
zinc nitrate
sodium nitrate C
calcium nitrate B
A

14

copper nitrate

CXC 0 Level Chemistry


Assessment Practice
Paper 02 Test 9

TIME:

1 hour 30 minutes.

INSTRUCTIONS TO CANDIDATES

Read the following directions carefully.

1.

You must use this booklet when responding to the questions.


write your answer in the space provided.

2.

All working must be shown.

3.

There are five questions in this booklet. Answer all five questions.

For each question,

1.

You should NOT spend more than 30 minutes or Question 1.


of a
(a) A student was provided with two solutions to investigate how the rate
reaction depends on concentration. Solution X was sodium thiosuiphate 0.15
. In the reaction of X and
3
. Solution Y was hydrochloric acid 2.0 mol/dm
3
mol/dm
cross marked on a white
pencilled
e
a
obscur
Y sulphur is formed which can
piece of paper
The following procedure was carried out:
3 flask.
3 of the thiosulphate were measured and placed a a 250cm
(1) 50cm
3 of hydrochloric acid were added to the solution andlhe time noted.
(2) Scm
(3) The mixture was swirled and the flask placed over the cross.
(4) The time taken for the cross to disappear when viewed from above was
noted.
(5) The procedure was repeated with other concentrations as indicated in the
table below.

Experiment
No.

Volume of
3
Aci&cm

Volume of
3
thiosuiphate cm

Volume of
water/crn

concenfration

lime t
Sec.

50

0.15

44

40

10

0.12

55

30

20

0.09

74

20

30

0.06

115

0.03

245

40

10

(i)

Complete the table.

(ii)

Plot a graph of concentration of volume of thiosuiphate against lit.

lit.
secs

(iii) Assuming lit is a measure of rate of reaction, how does rate depend on
concentration of thiosulphate?______

(iv) Which variable is manipulated?


(v) Which variable is responding?
(vi) Which were the controlled variables?.

__________

(b)

_____

Complete the table below for tests carried out on V which were dissolved in water
and divided into five portions.
TEST
To the first portion diluted hydrochloric
acid was added followed by barium

(i)

OBSERVATIONS
white precipitate formed

CONCLUSIONK1
-

chloride

(iii)
(iv)

(c)

To a second portion dilute nitric acid was


added followed by silver nitrate solution
To a third portion sodium hydroxide was
added and warmed gently
To the fourth portion ammonia solution
was added until in excess
To the first portion potassium iodide
solution was added

no precipitate formed
-

white precipitate soluble,


pungent gas given off
white precipitate formed,
soluble in excess
no precipitate formed

4)

Acqueous ammonia is the active ingredient in a number of household cleaners,


namely window cleaners. You have been given two brands of window cleaners.
Plan and design an investigation to find out if each brand contains the same amount
of ammonia.

11
(d)
L

,
r

tpp 4111

4t#-f -4-L4t

it?.

--t-t

I-

i-i

Hi
4-H-i-

r:
1
t
!

.,

.:

4..

..

tr:,

-.:;

.4

;..

;.u;

i.
4
4

__

..

4.

..

;4

.:

...

..

_.

rirtt

-.,

:tr
-;

-.

--

..

t-----r-t

i-

,.
.

rrr

--._

ii_.iL.Li_Lr

!J

,:L,z--

.
-

-.

----t-

------.
-I,
*

..

---

-*----s.4---4+--

--

--

_L-

-:-

-cI-

2A

Choose from the following elements to answer the questions below


aluminium

bromine

calcium

chlorine

hydrogen

iodine

iron

magnesium

nitrogen

oxygen

Each element can be used once, more than once, or not at all.
Name an element which;
(a) forms two different positive ions,
[11
(b)

has an amphoteric oxide,

11]
(c)

is the most reactive metal in the list,


[1]

(d)

forms a diatomic molecule with a triple bond,


[1]

(a)

is the most reactive halogen in the list.

11)
28 The diagram shows how silicon is manufactured from sand.
caon monoxide

(a)

Sand is a compound of silicon. Name the compound.


[1]
5

(b)

silicon
Use the information in the diagram to explain why the manufacture of
involves a redox reaction.

[2]
(c)

Silicon is used to make silicones.


(I) Silicones are synthetic polymers. The repeat unit contains silicon-oxygen bonds.
Draw the structure of a silicone showing at least two repeat units.

(II)

Give one other use for silicon.


[2]

3A

Glues can stick surfaces together

A chemical reaction takes place when the glue sets solid. An experiment is carried out to
(the
find out how changing the temperature changes the time for the glue to set solid
setting time).
The graph below shows the results of the experiment.

1I[F
T1
IW}ilfi1llhifJ}MII1IIE 1iiiIi!1

Trru1nannnutn qw.!

setting
time?
minutes

..

30

rtt1JiI1i!JIfI1

lfllflEfl4jIjflfljfl[I. H
J ttHHNflflHftN-l

flhltllhtlltflhltll

11111111Ff 1111 [hfI iiiti

U Ililhlifli H

III ttl I t
o ml lit
10
5
o

2
15

20

iomperatur&C

25

30

35

(a)

What is the setting time at 20t7


minutes

(b)

[1]

(I)

What happens to the rate of the setting reaction


as the temperature increases?

(II)

Explain your

answer in terms of reacting particles.

[3]
(c)

3B

The experiment is repeated using glue containin


g a small amount of a catalyst. Draw
a curve on the graph to show the results you would expe
ct. Label this curve catalyst.
(1]
The table below give some properties of the
elements in Group I.
element

atomic
symbol

electronic
structure

melting
point/C

boiling
point/C

density
in g/cm
3

atomic
radius4m

lithium

Li

2, 1

180

1330

0.53

152

sodium

Na

2, 8. 1

98

892

0.97

186

potassium

2, 8, 8,1

64

760

0.86

231

rubidium

Rb

2, 8, 18, 8, 1

(a)

The properties of rubidium are missing from


the table.
(1) Which one of the properties of the elem
ents does not show a detinite trend?
(II)

Predict numerical vaiues for the other three


properties of rubidium.
Write your answers in the table.

[4]

(b)

What common feature of the electronic


structures of the Group I elements is
responsible for their chemical properties bein
g similar?
[1]

(c)

Uthium reacts with water as shown in the


equation.
2U(s)
(I)

2H
0
2
(l)

2LiOH(aq)

H
(
2
g)

A smah piece of lithium of mass 0.35 g is


added to cold water The resulting
solution is titrated with 2.00 3
mol/dm hydrochloric acid
7

What volume of hydrochloric acid is needed to neutralise the solution?

(ii)

Rubidium also reacts with water.


Name the products of the reaction between rubidium and water

Which will react faster, lithium or rubidium?

[61
4A

(a)

.
2
The alkanes are a series of hydrocarbons. They have the general formula CH
(I) Give the name of the alkane having n = 3.

(II)

Draw the structure of two isomers of the alkane with n

5.

[31
(b)

The alkenes are a series of unsaturated hydrocarbons.


Give the general formula for an alkene,
(i)

(ii)

Explain how the molecular structures of alkanes and alkenes differ.

,
(iii) Describe a test you can use to d:sflnguish between an aikane and an alkene
and give the results you would expect.
reagent
observahon with an alkane
observation with an alkene

[5]

BA

CFCs are compounds that contain only carbon, chlorine and fluorine They are
atmospheric
pollutants and destroy ozone in the upper atmosphere.
(a)

CFCl 1 has the following composition, by mass:


C, 8.7%; F, 13.B% Cl, 77.5%
Calculate the empirical formula of CFC1 1.

12]
(b)

CFC1 2 has the molecular formula 2


CI It can be made by the reaction of hydrogen
CF
.
fluoride, HF, with tetrachloromethane, CC!
.
4
4
Cd

2HF

CC
+
2
F
2HCI
1

What is the maximum mass of CFC12 that can be made from


lOOg of hydrogen
fluoride?

[31
58

(a)

Name two aqueous solutions which will react to form a precipitate of silver
chloride,
AgCI.
and

(b)

11]

Write an ionic equation, including state symbols, for the reactio


n in (a).
[2]

(c)

Describe how you would obtain a pure, dry sample of the silver
chloride precipitate
from the mixture in (a).

[31

CXC 0 Level Chemistry


Assessment Practice
Paper 03 Test 9

TIME:

1 hour

INSTRUCTIONS TO CANDIDATES

Read the Instructions on the answer sheet very carefully.

choose ONE from


Candidates MUST answer THREE questions on this paper. They must
Section 1, ONE from Section 2, and ONE from Section 3.
All working MUST be shown for calculations.

Section I
Answer one (1) question from this section.
Hydrogen is used as a fuel in some space rock
ets.
(a) Describe how hydrogen is manufactured from
(b)

a named hydrocarbon source.

[3]

A highly exothermic reaction occurs between hydrogen


and oxygen to form water
0
HH
0
H
H
+
*0
HH
0
/\
H
H
(I)
Suggest why liquid hydrogen, rather than hydr
ogen gas, is carried by space
rockets.
(II)

Explain what is meant by an exothermic reaction.


(hi) Which bonds are broken in this reaction?
(lv)
(v)

(c)

What type of energy change occurs when bond


s are formed?
When 1 mole of hydrogen molecules reacts with
oxygen, the energy change,
!iH, is 285kJ. Calculate the energy change which occu
rs when 1 COg 01 hydrogen
reacts with oxygen.
[6]

Other space rockets use hydrazine. N


, as a fuel. It reacts with hydrogen peroxide.
4
H
2
to form nitrogen and water
Construct a balanced equation for this reaction.

[1]

[Total 10]
2

Ammonia is used to make lertilisers, dyes,


explosives and plastics. Ammonia is
manufactured in the Haber process by the reve
rsible reaction between hydrogen and
nitrogen.
(a)
(b)

Describe the essential conditions needed in


the Haber process.

[3]

Ammonium nitrate, 3
NO
4
,
H amrnonium sulphate, 4
(NO
S
2
,
Hj and urea, 2
(N
C
)
0,
H
are all fertilisers that can be made from amm
onia. The three fertilisers all provide
plants with nitrogen.
(I)

Name the two other essential etements that are


needed by plants.
(II) Construct an equation for the making of
ammonium sulphate from ammonia.
(Ill) A shop sells ammonium nitrate, ammoniu
m sulphate and urea in 1 kilogram
bags.

Which bag contains the most nitrogen? Explain


your answer
(c)

Draw a dol ad cross diagram to show the bond


ing in ammoni2

[5]
(2]

[Total :10)
7

Section II
Answer one (1) question from this section.
of food They can all be
Carbohydrates, fats and proteins are the three main constituents
s
alkali.
aqueou
hydrolysed, either by aqueous acid or by

(a)

What is formed by
(I) the acid hydrolysis of carbohydrates,
(Ii)

(b)

The hydrolysis of proteins gives a large number of products.


s are
What is the name of the class of compound formed when protein
(I)
hydrolysed?
y the products
(II) Describe how chromatography can be used to separate and identit
[4]
formed by the hydrolysis of proteins.

(c)

Poly (ethene) and nylon are synthetic polymers.


proteins.
Nylon can be hydrolysed because it contains the same linkage as

(d)

[2]

the alkaline hydrolysis of fats?

(I)

Name this linkage.

(Ii)

Poly (ethene) cannot be hydrolysed. Outline one environmental problem caused


[2]
by this lack of hydrolysis.

, is a colourless liquid. It is rapidly hydrolysed by water


3
Phosphorus trichloride, PCI
P0 and hydrochloric acid.
H
.
to form an aqueous mixture of phosphoric acid, 3
[2]
Construct an equation, including state symbols, for this reaction.
[Total :101

electrodes or copper
Aqueous copper (II) sulphate can be electrolysed using either carbon
electrodes.
[1]
(a) Give the formulae of the ions present in copper (II) sulphate.
(b)

(c)

electrodes, a
When aqueous copper (II) sulphate is electrolysed using carbon
colourless gas is formed at the anode (positive electrode).
What is the name of this gas?
(I)
[21
(II) Describe a test for this gas.
des.
Aqueous copper (II) sulphate is electrolysed using copper electro
Explain why the anode decreases in mass.
(I)
(II) Explain why the cathode increases in mass.
(III) Write an equation for the reaction at the cathode.

[3]

(d)

drop by drop, until is


Describe what you would see when aqueous ammonia is added
[3]
in excess, to aqueous copper (II) sulphate.

(a)

Give the name of an alloy that contains zinc and copper

11]

[Total :101
3

Section III
Answer ONE question from this section.
5.

(a) Give a word equation for the formation clay.

[21

(b) Describe how clay particles are able to hold water molecules to their surface
s.
What is the purpose of these water molecules?
[5]
(c) List three properties of clay.

[3]

(d) Describe the basic process of producing pottery from clay, showing how
the
properties listed in part (c) are utilized in the process.
[6]
(e) List three products of clay and state one similarity between clay produc
ts and
glass products.
[4]

6.

(a) Give a balanced chemical equation for the process of photosynthesis in


green
plants.
[2]
(b) Distinguish between the following types of elements needed for the
normal
growth of green plants; non-mineral elements, primary nutrients, second
ary
nutrients and micro-nutrients.
[4]
(c) Give six elements which are essential to plant growth. State which elemen
ts are
non-mineral, primary, secondary or micronutrients.
[6]
(d) For each element listed ri part (c), state its source.

[3]

(e) Describe how a deficiency of three named elements affects green plants.

[5]

CXC 0 Level Chemistry


Assessment Practice
Paper 01 Test 10

TIME:

1 hour 15 minutes.

INSTRUCTIONS TO CANDIDATES
There are sixty (60) questions in this paper. Answer all questions. For each question,
there are four possible answers A, B, C and D. Choose one you consider correct and
record your choice in soft pencil on the separate answer sheet.
Read the Instructions on the answer sheet very carefully.

INFORMATION FOR CANDIDATES


Each correct answer will score one mark.
answer.

A mark will not be deducted for a wrong

Any rough working should be done in this booklet.

The cover-plates were removed from the gas jars shown in the diagram. After several
days, the colour of the gas was the same in both jars.
-

oxygen

cover-plate

bromine

Which statement explains this change?

Oxygen and bromine gases have equal densities.

Oxygen and bromine molecules are in random motion.

Oxygen and bromine molecules diffuse at the same rate.

Equal volumes of oxygen and bromine contain equal numbers of molecules.

, is 160.
4
The relative molecular mass, M of copper (II) sulphate, CuSO
The relative molecular mass, Mr of water is 18.
?
5H
2
.
4
CuSO
What is the percentage by mass of water in copper (II) sulphate crystals 0
A

18x100
160

5xlBxlOO
160

18x100
160+18

Sxl8xlOO
160+(5x18)

3 of water They then titrated


Five students each dissolved an indigestion tablet in 100 cm
using
the same indicator.
acid,
hydrochloric
dilute
with
solutions
3 of their
25.0 cm
The results are shown in the table.

student
utration value/cr&

20.4

1-

2
20.5

204

20.6

-f

Which statement could explain the result obtained by siudent 5?


A

The burette was washed out with hydrochloric acid

The pipette was washed out with the tablet softjiicn


he student measured :o :he top of the meniscus in the pipette

The titration flask was wsned out with the iaoc: Diuiion

sI
22.0

From which mixture can the underlined substance be obtained by adding water, stirring
and filtering

calcium carbonate and sodium chloride

copper (II) sulphate and sodium chloride

ethanoic acid and ethanol

iron and magnesium

What can be deduced from the symbol

An atom of helium contains two electrons.

An atom of helium has two protons and tour neutrons in its nucleus.

Helium has a proton (atomic) number of 4.

Helium occurs as a diatomic molecule

Which of the following ions has the same number ol electrons as a krypton atom, atomic
number 36?

A
7

He?

chlorine

rubidium

xenon

Which substance could be sodium chloride

conduction of electricity
substance

melting point

when liquid

PC

sodium

in aqueous
solution

114

none

none

114

none

good

180

none

insoluble

808

good

good

The diagram shows the arrangement of electrons in the outer shells ol the atoms in the
.
2
compound YZ
Which pair of elements could be V and 7

calcium

fluorine

carbon

sulphur

oxygen

hydrogen

sulphur

chlorine

I
electrons of Y
x

electrons f 7

What is the formula of iron (Ill) nitrate?


A

10

Fe(NQ,)

Fe.](N0J)]

3
FeN0

,
(N0
3
Fe
)

70g

7
What is the mass of oxygen contained in 72 g of pure water
[Relative atomic masses: H
A

11

lSg

1:0

16]
C

32g

64g

Sodium reacts with water according to the equation below


2Na

0
2
2H

2NaOH

2
H

Which volume of hydrogen is produced at rip, when 02 mol of sodium reacts?


3
9.6dm
0
3
48dm
C
3
Z4dm
S
3
12dm
A
12

Which element is liberated at the cathode (negative electrode) by the electrolysis of an


aqueous solution containing its ions?
oxygen
0
hydrogen
C
chlorine
B
bromine
A

13

The heat-reflecting shields of some space rockets are gold-plated, using electrolysis.
Which electrodes and electrolyte would be used to gold-plate the heat shield?

14

negative electrode

positive electrode

electrolyte

carbon

heat shield

gold compound

gold

heat shield

copper compound

heat shield

carbon

copper compound

heal shield

gold

gold compound

shown
A current was passed through concentrated aqueous lithium chloride, LiC1, as

concentrated aqueous
lithium chiorci

Which entry in the table is correct


2
ions moving towards:

15

16

the cathode (-ye)

the anode (+ve)

Lh only

Ct and 0H

LV only

C! on!y

LV and H

C/ only

Lfr and H

Ct and 0H

Which of the following is an endothermic process?


A

the addition of water to anhydrdus copper (II) sulphate

the combustion of ethanol in air

the formation of a carbohydrate and oxygen from carbon dioxide and water

the oxidation of carbon to carbon dioxide

Dilute sulphuric acid reacts with copper (Ii) oxide to form copper (Il) sulphate and water
What would not alter the rate of this reaction?

17

the concentration of the sulphuric acid

the pressure at which the reaction takes place

the size of the particles of copper (II) oxide

the temperature of the reacting mixture

Magnesium reacts with hydrochloric acid.


Which solution would give the fastest initial rate of reaction
2
A

40 g of HC! in

1000 cm
3 of water

20 g of HC! in

3 of water
1000cm

lOg of HOt in

100 cm
3 of water

g of HC1 in

3 of water
50cm

D
18

Ethanol is produced by the


fermentation of sugar During the
reaction carbon dioxide is
given off The graph shows bow
the volume of carbon dioxide
produced per minute varies
With :eroerature.

pcnducedper
minuio

20

O
emperaIiuerC

40

&

50

Using the graph, decide which statement is correct?

19

20

21

22

The rate of reaction always increases with temperature

The rate of reaction reaches a max:mum at about 40C

The reaction is slowest at OC

The reaction takes a long time to begin

Which compound, when added to aqueous iron(ll) sulphate, takes part in a redox reaction?
A

ammonia

barium chloride

potassium manganate (VU)

sodium hydroxide

Samples of polluted air are passed through two reagents, acidified potassium dichromate(Vi)
and aqueous potassium iodide.
Which colour changes will be seen if the polluted air contains sulphur dioxide?
acidified potassium
dichromate (VI)

aqueous potassium
iodide

green to orange

brown to colourless

green tc crange

no change

orange to green

colourless to brown

orange to green

no change

In an accident at a factory, some nitric acid was spilt.


Which substance, when added in excess, would neutralise the acid without leaving an
alkaline
A

aqueous ammonia

aqueous sodium hydroxide

calcium carbonate

water

The diagram shows some ot the stages in the manufacture of ammonium sulphate
2
From wnc connecting pipe would a major leak most increase the pH value of rain
nitroce-
ann

Ernon I________________

hydrooc

ammonia
PIPE S

PIPE A

.-

N
ammonurn
sulphate

__
Th
ion
rca

so,

fl_
reaction -il
3
2
S
2
H
)

yessel
6

PIPE C

23

Which compound in solution produces a precipitate with aqueous ammonia that does not
dissolve when an excess of ammonia is added
2
A

copper (II) sulphate

iron (N) chloride

potassium hydroxide

zinc chloride

Element X forms a diatomic molecule. An atom of X requires one electron to form a stable
X- ion.

24

What can be deduced about X?

25

It isa Group I metal.

It is a halogen.

It is a noble gas.

It is a reducing agent.

The bar chart shows the period of elements from lithium to neon

prope

Li

BeB
element

Which prcerty of these elements is shown on the char


A
the number of electrons used in bonding
B

tne number of orbits hoding eectrons


the nmton) atomic number

the reiative atomic mass

CXC 0 Level Chemistry


Assessment Practice
Paper 03 Test 10
TIME:

1 hour

INSTRUCTIONS TO CANDIDA
TES

Read the Instructions on the


answer sheet very carefu

lly.

Candidates MUST answer THRE


E questions on this paper. They
must choose ONE from
Section 1, ONE from Section 2,
and ONE from Section 3.
All working MUST be shown for
calculations.

Fr

Ra

226

I,

Ac

227

Key

I X
[_J

5ITWC

a 9ifl sin)

XaMonlC.vnlbCi

2S42a.#!

Hf
72

La

L.n.-

Ba

9..

Cs

C,n.,.,

40
dl

94

58

71

IS,,

41

PttIsII

Pa

P,,00,I,

34

,n

Nd

ISA

6..

Re

946

92

82

t5

Ii

ISA4rI,

75

To

M.4n,.

75

Pm

Os

94

Tha,

Ru

CI

98

33

Sm

350

,n,,

Ir

97

d.

Rh

53

Sr,-fl
C

fl

45

77

Ca.,

1!!

NpPu
6SAII-

PIw 6,.

76

44

24

Am

Eu

52

P98,,r

Pt

IZ

63

74

44

00

Pd
.,tO.,

26

NtkI

Group

04

73

43

29

C .0,

Cm

Gd

57

C06

Au

97

Ag

00

Clo..

Bk

1n

Th

150

Ma
f
3
l

Hg

201

Sa,tr

82

44

Cd

117

2c

CO

20

Cl

Dy

IC

TI

C$,.,,I
94

60

01

204

.e.,

In

115

CJA..,

Es

Ho

IC

Lno

Pb

207

6,

I ,ws.
49

6)

62

Sn

119

C.,4,.,.n
32

4a%.,,,

:4

71

Si

27

Cst

Al

49

2:

B
901a

ii

ns

..n.

112

Fm

tm,,4

Er

67

Sn.,

Bi

200

Ate.,

Sb

f.,,..,
903

53

51

7)

A,t

Md

Tm

leO

Dt,.,

Po

Te

179

3.

9r

32

2fl

101

9Seab.1

94

64

52

34

94

C154,

Cl

355

F%A

Yb

172

Ann

At

.,,..

12

Ba

No
152

f.w,

70

94

5,3

35

20

Is

Lu

75

Rn

Xc

wb

A9al

&

40

Na

903

t,s.,.I6,,

21

90

54

30

10

IiiiIivlvIviIvii;

The volume of one mole of any gas is 24 3


dm at room temperature and pressure (rtp.)

Th

232

0...-

149

Pr

Ia

54

ItO

74

47

Ce

1.a*

Ta

191

*2t44p

Mo

Nb

24

V
Zr

2)
94

27

70

39

22

58

36

11

37

fl

Sr

t*.

Rb

94

p-so.

*9)

Mg

I4._es,

Na

24

58-il LintriaroirJ series


t90-1O3 Aclinoid series

SAO.n

23

o
9

Be

I%

Li

Lc_

II

DATA SHEET
Th. Periodic T&,Ia of the EImn.nts

41

-1 is:
10
1
4
The number of isomers of butane C
C
B
2
Al

n
42 Which of the following nitrates does NOT decompose with the evolution of nitroge
dioxide gas when heated:
(NO
2
)
Pb(NO) 13 Cu 3
2
C
3
KNO
B
A MgNO
3
43

The hydrogen atom:


only forms positive ions

only forms negative ions

C always loses an electron

may gain or lose an electron

neutral

44 Classify ZnO as an oxide:


basic
B
acidic
A

45

amphoteric

When ions are discharged at the anode the process is one of:
hydration
oxidation D
neutralization C
B
reduction
A

ally changed:
46 Which one of the following will conduct electricity without being chemic
solid sulphur
B
solid magnesium chloride
A
C

molten magnesium chloride

solid copper

47 Which solid hydroxide Is used usually in the preparation of ammonia:


Mg(OH)
2
13
NaOH
C
Ca(OH)
2
B
KOH
A
uents:
48 Crude oil can be separated by fractional distillation because its constit
are chemically similar
B
are compounds
A
C

have different boiling points

hve different densities

49 Covalent compounds are usually:


A

crystalline solids

scUds with high melting points

soluble in water

13

non electrolytes

12

50

From the symbol DNa, it can be deduced that:


A

SI

an atom of sodium has 11 protons and 23 neutrons


an atom of sodium contains 11 electrons

sodium has an atomic number of 23

an atom of sodium contains 11 neutrons

Fluorine is in a group(VlI) of the Periodic Table. Its chemistry will most closely resemble
that of:
A

52

56

iodine

sodium

11

20

oxygen

carbon dioxide C

argon

chlorine

A solution has a pH of 5. This means that it is:


A

55

boron

Which gas is used to fill electric light bulbs and prevent burning away of the filament
7
A

54

The element with atomic number 4 is likely to have similar chemical properties to the
element with atomic number:
A

53

neon

slightly acidic

strongly acidic

slightly alkaline D

strongly alkaline

Which one of the following undergoes sublimation?


A

sodium chloride

ammonium chloride

calcium carbonate

lead nitrate

In which pair do the elements combine to form a compound with an ionic attica
A

nitrogen and hydrogen

carbon and hydrogen

carbon and oxygen

sodium and hydrogen

57

The atomic numbers of three elements, P0 and Rare 17, 18 and 19 respectively.
Which of the following is a correct statement regarding P, 0 and R?
A

Q is a noble (inert) gas

2
P and R Combine to give an ionic compound PR

0 exists as diatomic molecules


R can form an ion of charge +2

D
58

The empirical formula of ethanoic acid is:


A

59

60

O
2
CH

O
4
CH

0
4
H
2
C

0
6
H
2
C

Caesium is an element in the same group of the Periodic Table as sodium. It can be
predicted that caesium:
A

reacts vigorously with water

has a low melting point

2)
forms a hydroxide of formula Cs(OH

forms a solution with water with a pH of 2

Which of the following is not a polymer?


A

Polythene

Ethene

14

Nylon

P.V.C.

CXC 0 Level Chemistry


Assessment Practice
Paper 02 Test 10

TtME:

1 hour 30 nilnutes.

INSTRUCTIONS TO CANDIDATES

Read the following directions carefully.

1.

You must use this booklet when responding to the questions.


write your answer in the space provided.

2.

All working must be shown.

3.

There are bye questions in this booklet. Answer all five questions.

For each question.

(a) A student was provided with two solutions.

1.

3 of solution.
X is a solution potassium manganate (VII) containing 3.2 gmsldm
.
3
V is a solution hydrated iron (II) sulphate containing 22.5 gmsldm
The student is required to titrate X against V.
In this reaction a redox reaction takes place according to the following equations.
Mn

4 5Fe
Mn0

2 +

or

5Fe
2+

4 ag reacts with 5 MoIs Fe aq)


1 Mol Mn0

5 MoIs Fe

3+

(aq)

(i)

Identify the oxidation reaction taking place.

(ii)

What is the oxidation number of manganese in Mn04?

Figure 1 shows the reading in the burette before and after each titration against
3 of solution V.
25cm

23

25

24

2t

Titration 1

Titration 3

Titration 2

Use the information presented in Figure Ito complete the table.


Burette Reading
3
1cm

11TRATION
2

Final Reading
Initial Reading
Volume of solution x used/cm
3

(iii) What volume of solution X should be used in the calculation?

(iv) Calculate the concentration of solution X in mol/dm


.
2

(v) Calculate the number of moles in the average volume calculated in (ii).

(vi) Calculate the number of moles in 25 cm


3 of solution V.

(vh) Calculate the number of 3


mol/dm of solution Y.

(viii) Calculate the reactive molecular mass of the hydrated iron II sulphate.

If the formula of Vs FeSQ x il2O. Find the value of X.

RAM.

K=39

Mn=55

0=16

Fe=56

S=32

H=l

(b) The diagram beiow illustrates a meihod of converting glucose Ic ethanol.

Solution of glucose
and subitancc
w

(I)

What is the name of this process?

(ii) What is the substance W?___________________________________


(iii) Write an equation for the formation of ethanol in this reaction.

(c)

Ov

Name gas Y.

(v)

Describe what happened to the solution of lime water.

(vi)

WnEe an equation for the formation of precipitate.

(vii

Expain why the precipitate redssoIves as the reacfian proceeds.

(viii)

n
When the mixture was allowed to stand for a week, thepHof the solutio
result.
this
dropped to about 5 How would you expiain

The following apparatus can be used to separate each of the following


mixtures. Match each apparatus with the mixtures below.
A

(i)

A mixture of ethano! plus water

(ii)

A mixture of caThon plus copper oxide

(i)

The dyes in ink

(tv)

Oily water

ZA The Periodic Table will help you to answer


this question.
Give the symbol of
(a) a non-metal used to sterilise water,
[1]
(b) an element which consists of diatomic molecules,
[1]
(C) the most reactide element in Group VII,
[1J
(d) an element which reacts with water to give an
alkahne solution,
[1J
(e) an element which forms an ion of the type
2
X
[1]
(f)

the main element present in steel.


111

28 Use the information in the table to answer the follo


wing questions.
substance

conducts e!ecthcity
when solid

melting point/V

dissolves in water

sodium chloride

no

808

yes

sulphur

no

113

no

tungslen

yes

3377

no

wax

no

35-50

aiunithiuni

no

yes

660

no

(a) (I)

Name two metals from the table.


and

[1)

(ii) How can you tell from the table that the subs
tances you have chosen in (I)
are metals?

ru
(b) How can you tell from the table that wax is a mixture?
Lii

(c)

(I)

Name a compound from the tabe.

(ii)

Explain the meaning of the ward compound

111

[II
(d)

of particles in aluminium change as


Describe how the arrangement and movement
the temperature rises from est to 66fC
121

(e)

hur and sodium chloride to produce


Suggest how you could separate a mixture of sulp
a sample of each.
[3)

ZA

by fractional distillation. The table


Crude oil, a mixture ol hydrocarbons, can be separated
obtained
below lists some ol the fractions which can be

(a)

fraction

basting range/C

2570

70115

115200

200380

What is meant by hydrocarbon?

11
(b)

?
Which trachon corflains the ongesl chao molecules

(c)

, and one oilier hydrocarbonX.


4
H
5
. can be cracked to produce hexane, C
0
t-i
1
Decane, C
Deduce the molecular formula of X
(I)
(ii)

r1

ars of nydrocarbon X
Draw aDrans to show the structures at two isom

sed
(Ill) Explain why hydrocarbon X can be nolymeri

141
6

38

Space rockets use, as a fueL a mixture of hydrogen and oxygen.


(a) Write an equation br Ihe reaction between hydrogen and oxygen
!1]
(b)

The reaction ri

(a) is

an example of a redox reaction Eplan what this means.

12]
(c)

The value of the enihalpy change icr this reaction is given as:H
Is this reaction exothermic or endothermic
7 Explain your answer

285 kJ/nio[

[1]
(d)

Suggest one reason why burning hydrogen may cause tess envitOnmenaI damage
than burning hydrocarbons.

Ill
(e)

A pressurised gas cylinder contains 50 got hydrcgen. Catcuate the volume that the
hydrogen would occupy at room temperature and pressure.

[3]
4

Given below is a list of metals in order of reactivity.


calcium
magnesium
aluminium
zinc
copper
silver
The fcltowing experiment was set up
metal

AgNO
(
3
aq)
A

Ca(N0
(
2
)
3
aq)
B

M(tO)
(
2
aq)
C

CuQQ
(
2
)
3
aq)
D

(a)

(I)

in which beaker was there no reaction?

(ii)

Construct an equation for the reaction in beaker A


[2)

(b)

The experiment was repealed Zinc strips were used instead of the metal strips in all
four beakers
From looking at the reactivity is!. in which beakers would you expect to see a reaction?
[11

(C)

The experiment was repeated a third time, using strips of aluminium in all four beakers.
No reachons happened
Explain why ajuminium does not react.
[21

SA

Hydrogen. H, deuterium. D, and tritium, T, are isotopes of one anoiher. Their atomic
siructures are shown below.

deulerium

hydrogen

(a)

tritium

Use the information above to complete the tabte


symbol

particle

proton

charge

mass
1

.
)C

(b)

[21

Explain why the three atoms are isotopes of each other.


ji]

(c)

What is the relative molecular mass of a molecule

of

titium. T,

[1
(d)

yd:1gen can lorm a hyriie

on.

(i)

What is the formula of catchim hydride?

(i

Drawadiagram, sinna. lotheonesabove, Ic


In a hydride ion.
B

si

thearrangernentof pariIes

31

All ammonium salts on heating with sodium hydroxide produce ammonia gas.
From which ammonium salt can the greatest mass of ammonia be obtained?

32

P0
3
)
(NH
0.5 mol 4

),S0
(NH
0.5 mol 4

CI
4
1.0 mol NH

1.0 mol 3
NO
4
NH

Compound X reacts with some metals to liberate hydrogen and is used to make fertilisers.
It gives a white precipitate when added to aqueous barium nitrate
What is X?

33

34

35

animonium sulphate

hydrochloric acid

potassium nitrate

sulphuric acid

Which of the following is the main constituent of sand?


A

calcium carbonate

calcium silicate

silicon (IV) oxide

sodium silicate

Which statement about graphite is not correct?


A

It burns to torm carbon dioxide.

It is a carbon compound.

It is a macromolecule.

It is used as a lubricant.

Waste water trom a factory was found to have a pH value of 2


Which substance could be used to neulralise the waste water before it is released into a
river?
A

ammonium sulphate

lime

oxygen

sulphur dioxide

36

Octane is an alkane containing eight carbon atoms per molecule


Whal is its molecuiar lormula?
A

37

H
1
C

6
C,I-l

H
8
C

The diagrams show tour structures


1

2
H

&

I
/
HCCCC
I
H

I
I
I
HC-CC--CH
I
I
i
H

HCCH

HC--CCH

HCC-H
I-I

1
/\

Which structures represent alkenes with the molecular lormula 0


?
H
4
A

38

1and2ony

1,2and4orily

12sand4

3and4only

An organic compound X reacts with sodium hydroxide to give a compound with formua
C
0
5
H
3
,Na
Wia; is compound X?
A

39

ethano

prcpano:c acid D

propane

propanol

iich diagram represents the structure ol

A
B

2
0H
-CH-
CHCH
01

Cl

0
U

40

I-I

What is prcstjed when ethanol is boieo with an excess of acjd,ied potassium dichromate
(VI)

eihane

ethanc c ace

10

ether.e

ehyt ethanoat

Section I
Answer One (1) question from this section.
Sodium phosphate, 4
PO is a soluble salt, used as a water softener in washing
3
Na
,
powders. 1 is made by reacting dilute phosphoric ad. HP0. with an aikalL
(a) Give the formulae of the ions present in sodium phosphate.

[1]

(b) (i) Name the alkali which reacts with phosphoric acid to make sodium
phosphate.
Write
(ii)
an equation for this reaction.
[21
(cj

Given solutions of phosphoric acid and alkali, a suitable indicator and standard
laboratory apparatus, explain how you would obtain crystals of sodium
phosphate.
[31

(d) Sodium carbonate also reacts with phosphoric acid to make sodium phosphate.
(i) Name the gas formed during this reaction.
(ii) Describe the test for this gas.
[2]
Ce) Calcium phosphate is used to make fertilizers.
(i) Deduce the formula of calcium phDsphate.
(ii) What essential plant element does calcium phosphate provrde

121
[Total: 10]

Wines which contain ethanol, CH


OH may have a sour taste ci vinegar after long
5
storage.
(a) (i) Give the name and draw the structure of the aciu which causes the sour
taste.
(ii) Explain how this acid is formed from the ethanol.
[3]
(b) Ethanol can be made for industrial use by the cataytic addition of steam to
ethene,
(g) + -420(9)
4
H
2
C
OH(g)
5
H
2
C
(i) Name the catalyst and give the necessary conditions for this reaction
() Calculate the maximum mass of ethanol wtch can be made from 72 dm
3 of
ethene at room temperature and pressure.
[5]
(c) Suggest why most of the ethanol used in industry is not produced by
fermentation.
[1]
(d) Give an industrial use for ethanol, other than in alcoholic drinks.
[Total

[1]
10]

Section II
Answer one (1) question from this section.
3

A student carried out three experiments using lithium and water.


3 of
150 cm
In experiment 1, A cleaned piece of lithium of mass 0.2 g was added to
water.
(g)
2
2LiOH(aq) + H
0Q)
2
2Li(s) + 2H
The volume of hydrogen formed was measured at intervals. The following graph
was obtained.
.

--

j
400
300
tnta
volume of
200

TI

rH

Frn-T-1]J.1

-t
-44

L4L44

.1

F i-i-I-,

nitH
0
m

Irn

4
1

4
L
time/seconds

(a) (i) State two observations which would be made after adding lithium to the
water.
(ii) Name a 5ubstance which could be added to the water to show that a
[3]
solution containing hydroxide ions had been produced.
(b) (I) State how the rate of reaction changes during the experiment,
(ii) Explain why the reaction stops,
(C)

HOW long does it take for the lithium to react?

[2]
1]

(d) The student carried out two further experiments.

Experiment 2 was the same as experiment 1. except 0.1 9 lithium were used.
Experiment 3 was the same as experiment 1, except the water temperature was
raised by 1OOC.
(i) Use the graph to find the total volume of hydrogen produced in experiment
1
(H) Deduce the tota] volumes of hydrogen produced for each of experiments 2
and 3.
(iii) Deduce how the rates of reaction for each of experiments 2 and 3 wou:d be
[4]
different from experiment 1.
Fotal 101

When sodium bums in chlorine, sodium chloride


is forrne&

sium
chlorine

(a) What chemica test would show that, at first, the


gas jar contained chionne gas
[1]
(b)

State two observations which would be made durin


g the reaction

(c) Write an equation for the reaction.

[2)
[1)

(d) These diagrams show atoms of sodium and


chlorine.

sodium

chlorine

Draw similar diagrams to show the arrangement


of electrons and the charges on
the ions present in sodium chloride.

[31

(e)

Explain why the bonding in sodium chlo


ride gives rise to the following
properties
(i) A very high melting point.

(ii) High electrical conductivity when molten.

[1
[Total :1OJ

Section LU
Answer ONE question from this section.
[2]

5. (a) Give a basic definition for glass.

the arrangement of aloms in the


(b) State the major component of glass and show
[41
structure formed from this compound.

13]

(c) List lhree properties of glass

and describe (he effect of one


(d) State the components of two types of glass
[6]
each case.
component on the quality of the glass produced in
rent. Explain how this
(a) Give one property of two types of glass which is diffe
[5]
of glass.
difference is responsible for the uses of each type

fixing* of (a) nitrogen?


6 (a) What do you understand by the tern,
(b) How is nitrogen fixed in nature?
is so much nitrogen available Ti the
(c) Why is there a nitrogen problem when there
atmosphere?
to farmland where crops are
Cd) Explain why it is necessary to add fertilizers
regularly harvested
onium nitrate. Calculate the
Ce) One of the most widely used fertilizers is amrn
H = 1 ,O = 16
percentage of nitrogen iii ammonium ni(rae IN = 14,
as a fertilizer.
(f) Suggest one disadvantage of using pure ammonia

Anda mungkin juga menyukai